2011 Inservice

अब Quizwiz के साथ अपने होमवर्क और परीक्षाओं को एस करें!

31. What is the 5-year survival rate for patients with stage IV thymoma, based on the Masaoka staging system? A. 10% B. 30% C. 50% D. 70%

Both options B and C were scored correct. RATIONALE: The Masaoka staging system is useful for management and determination of prognoses for patients with thymoma. Patients with stage I to stage III thymomas have a 5-year survival rate of about 70%. Patients with stage IV disease have a 5-year survival rate of about 50%. REFERENCE: Park HS, et al. Thymoma. a retrospective study of 87 cases. Cancer. 1994;73:2491-98.

33. A proliferating population of mammalian cells maintained at 37°C is irradiated with 4 Gy, and then a second dose of 4 Gy is delivered at various times thereafter. The cell surviving fraction as a function of the time between doses: A. remains unchanged as the length of the interval between doses increases. B. increases as the length of the interval between doses increases. C. increases as the length of the interval between doses increases, but levels off once the interval exceeds 2 hours. D. increases as the length of the interval between doses increases, but then oscillates once the interval exceeds 2 hours.

Both options B and D were scored correct. RATIONALE: By splitting the total 8 Gy dose into 2 equal fractions with a time interval in between, sublethal damage from the first exposure would be repaired over time and therefore would become unavailable to interact with damage produced by the second dose. In the case of rapidly dividing mammalian cells, however, cell synchrony effects will be superimposed on the repair response, in that most of the cells surviving the first dose would be in the radioresistant S phase of the cell cycle. For an interfraction interval in excess of 2 hours, however, these cells start to enter the radiosensitive G2 and M phases, so a second exposure at this time will produce a drop in cell survival. Later, survival will increase again as the surviving cells go through the M phase and double their numbers.

156. Which of the following findings is associated with pancreatic cancer? A. High (>75%) incidence of KRAS mutations B. High (>75%) incidence of HER2/neu overexpression C. Diagnosis confirmed by an elevated serum CA 19-9 level D. Low incidence of positive lymph nodes

answer is A. RATIONALE: A KRAS codon 12 mutation frequency of 70% to 100% is the highest reported for any tumor type. Ca 19-9 serum levels may be elevated due to biliary obstruction. Seventy to eighty percent of patients have node-positive disease. Exam answers and rationales Radiation Oncology In-Training Exam 2011 47

145. Which of the following additional staging procedures is NOT indicated for a 10-year-old boy with clinical stage IIB classic histology Hodgkin lymphoma? A. Bone survey B. Bone marrow biopsy C. CT scan of the chest, abdomen, and pelvis D. PET scan

answer is A. RATIONALE: A bone survey is not indicated for staging of a 10-year-old boy with stage IIB classic histology Hodgkin lymphoma. A bone marrow biopsy is indicated, except for stages IA and IIA. REFERENCE: Orkin, et al. Oncology of Infancy and Childhood. 1st ed. 2009. Exam answers and rationales 44 American College of Radiology

138. Which of the following proteins activated in response to DNA damage is phosphorylated first? A. ATM B. Chk1 C. Chk2 D. H2AX

answer is A. RATIONALE: A cell's response to DNA damage requires activation of the ATM signaling pathway. ATM, a member of the phosphatidyl-inositol-3-OH kinase-like kinase (PIKK) family, auto-phosphorylates itself first in response to the presence of DNA strand breaks, and then proceeds to phosphorylate Chk2, followed by histone H2AX (to form H2AX). Other downstream substrates are then phosphorylated (including BRCA1, p53 and others), which in turn activate proteins involved directly in DNA repair and cell cycle checkpoint control. REFERENCE: Hall and Giaccia. Radiobiology for the Radiologist. 6th ed. Chapter 17. Figure

108. What common cytogenetic abnormality is found in patients with Ewing's sarcoma? A. t(11;22) B. LOH 16q C. Codeletion 1p and 19q D. N-myc amplification

answer is A. RATIONALE: A fusion protein resulting from t(11;22) (q24;q11.2-12) fuses the Ewing's sarcoma (EWS) gene of chromosome 22 to the FLI1 gene of chromosome 11, which functions as a master regulator. Other translocations that are also found are t(21;22) and t(7;22). LOH 16q is found in Wilms tumor, codeletion 1p and 19q is found in oligodendroglioma, and N-myc amplification is associated with neuroblastoma.

130. Which method of analysis would best determine the simultaneous effect of smoking and age on the risk of lung cancer? A. Multivariate B. Kaplan-Meier C. Cox proportional hazards D. t-test

answer is A. RATIONALE: A multivariate analysis is used when the effect of more than one variable needs to be considered when predicting an outcome.

282. Which of the following rationales may be used to justify irradiation of radiographically (PET or CT scan) uninvolved lymph nodes in a patient with a primary solid tumor that tends to spread to the lymphatics? A. A 1 mm tumor may contain up to a million cells, and a typical CT scan slice is only 2.5 mm thick. B. Chemotherapy is ineffective for cytoreduction of subclinical lymphatic disease. C. Eradication of subclinical disease with a >90% probability requires <50% of the total radiation dose needed for primary tumor control. D. The negative predictive value of a PET or CT scan revealing negative lymph node disease is typically <50% (e.g., in breast or lung cancer).

answer is A. RATIONALE: A radiation dose-response relationship exists for subclinical disease, and it typically requires the delivery of 45 to 50 Gy to achieve tumor control. It is important to realize the sensitivity limits of radiographic studies such as PET or CT scans. Exam answers and rationales 86 American College of Radiology

208. A person who receives 10 Sv of whole-body irradiation most likely will die from: A. gastrointestinal syndrome. B. cerebrovascular syndrome. C. bone marrow syndrome. D. prodromal syndrome.

answer is A. RATIONALE: A whole-body irradiation dose of 10 Sv would cause sloughing of the epithelium of the GI tract and death within about a week (most often due to septicemia). This would occur before the individual had a chance to develop the bone marrow syndrome, which typically takes a month or two to cause lethality. Death due to the cerebrovascular syndrome would occur even sooner, however this syndrome has a much higher threshold dose, on the order of 50 Sv or more. The prodromal syndrome consists of a set of (usually) transient gastrointestinal and neuromuscular symptoms that occur within minutes to hours of whole-body irradiation. The shorter the time of onset, the longer the duration, and the greater the severity of prodromal symptoms is usually indicative of the individual having received a lethal dose of radiation. REFERENCE: Hall and Giaccia. Radiobiology for the Radiologist. 6th ed. Chapter 8.

286. According to AAPM TG-40, the output of a radiation therapy beam should be verified: A. daily. B. weekly. C. biweekly. D. monthly.

answer is A. RATIONALE: AAPM TG-40 recommends daily output verification within 3%, with tighter thresholds on the less frequent tests (2% on monthly validation, 2% on annual absolute dosimetry). Exam answers and rationales Radiation Oncology In-Training Exam 2011 87

1. According to the 2008 National Cancer Institute (Buchholz, et al.) report regarding postmastectomy irradiation, which of the following treatment recommendations is most appropriate for a patient who has clinical stage IIB (T2N1) breast cancer and achieves a complete pathologic response to neoadjuvant chemotherapy? A. No radiation required B. Radiation therapy to the chest wall only C. Radiation therapy to the chest wall and supraclavicular lymph nodes D. Radiation therapy to the chest wall, supraclavicular, axillary, and internal mammary lymph nodes

answer is A. RATIONALE: According to the 2008 National Cancer Institute (Buchholz, et al.) report, postmastectomy radiation was recommended for clinical stage III breast cancer prior to neoadjuvant chemotherapy or for positive lymph nodes after neoadjuvant chemotherapy. Data from NSABP and MD Anderson (referenced in the report) indicate the risk of locoregional recurrence for clinical stage II and pathologic negative lymph nodes is less than 10%. It is unclear whether radiation offers a clinical benefit in those patients. REFERENCE: Buchholz, et al. National Cancer Institute report (review of the literature and consensus statement). Journal of Clinical Oncology (J Clin Oncol). 2008;26:791-797.

21. Which of the following types of leukemia most commonly occurs in children? A. Acute lymphoblastic B. Acute myelogenous C. Chronic myelogenous D. Chronic lymphocytic

answer is A. RATIONALE: Acute lymphoblastic leukemia is the most common type of pediatric leukemia, accounting for 75% of cases. Acute lymphoblastic leukemia has a median age of occurrence of 10 to 12 years. The other disorders have a median age of occurrence in the 7th decade of life. REFERENCE: Gunderson LL, Tepper JE (eds). Clinical Radiation Oncology. 2000; p 1089. Exam answers and rationales Radiation Oncology In-Training Exam 2011 7

312. Which of the following management options would be best for a 10-year-old girl who has undergone complete resection of a cerebellar pilocytic astrocytoma? A. Surveillance B. Temozolomide C. Carboplatin and vincristine D. 50 to 54 Gy of EBRT

answer is A. RATIONALE: After complete resection of a pilocytic astrocytoma, the recurrence rate is 10% or less; therefore, surveillance is recommended. Chemotherapy can be considered in young patients with unresectable, recurrent, or progressive disease to delay the potential morbidity of radiation therapy. In general, radiation therapy is recommended after 10 years of age in patients who have non-NF1 with gross unresectable, progressive pilocytic astrocytoma.

164. Which of the following patients with thyroid carcinoma is most likely to have the best prognosis, assuming that all other prognostic factors are equal? A. A 25-year-old patient with papillary carcinoma B. A 25-year-old patient with medullary carcinoma C. A 55-year-old patient with papillary carcinoma D. A 55-year-old patient with follicular carcinoma

answer is A. RATIONALE: Age is the most powerful prognostic factor in papillary and follicular thyroid carcinoma. In adults, younger is better. The breakpoint in the staging and classification systems is 45 years old, such that the answers in this question are not equivocal.

320. Which of the following particles is NOT an elementary particle? A. Alpha B. Beta C. Proton D. Neutron

answer is A. RATIONALE: Alpha particles consist of other particles, two neutrons and two protons. The other three are elementary particles with beta being a fast electron emitted from the nucleus of an atom.

194. Which of the following genetic changes commonly occurs during the process of carcinogenesis? A. Upregulation of BCL2 B. N-terminal mutation in TP53 C. Deletion of EGFR D. Loss of KRAS

answer is A. RATIONALE: Although the loss of p53 activity is also a common feature of the carcinogenesis process, inactivation of the protein is typically secondary to a mutation in the core domain of the TP53 gene, not at the N-terminus. Activation of EGFR and KRAS, not their loss, is implicated in carcinogenesis for some cancers.

316. Which of the following elevated serum tumor markers suggests that a patient with testicular cancer has non-seminoma versus pure seminoma? A. AFP B. CEA C. LDH D. -hCG

answer is A. RATIONALE: An increased -fetoprotein (AFP) serum level in patients with testicular cancer will point to the diagnosis of non-seminoma. Serum levels of AFP in patients with pure seminoma are normal. Carcinoembryonic antigen (CEA) is not related to testicular cancer. Lactate dehydrogenase (LDH) is not a specific tumor marker.

57. Which of the following laboratory abnormalities is most common at presentation in patients with multiple myeloma? A. Anemia B. Thrombocytopenia C. Lymphocytopenia D. Neutropenia

answer is A. RATIONALE: Anemia is the most common laboratory abnormality at presentation in patients with multiple myeloma. REFERENCE: DeVita VT, Hellman S, Rosenberg, SA (eds). Cancer: Principles & Practice of Oncology. 6th ed. Lippincott: 2001; p 2471.

261. Which of the following statements about reirradiation tolerance of the spinal cord is true? A. The time interval between the original radiation course and reirradiation is inversely proportional to the incidence of myelopathy. B. The spinal cord can undergo reirradiation to a full dose if the previous radiation course was administered at least 6 months earlier. C. Reirradiation data in animals and humans suggest partial repair of subclinical radiation damage becoming evident more than 2 years after the original radiation course. D. Animal studies have shown that the spinal cord does not recover a sufficient amount of subclinical radiation damage to allow for safe reirradiation.

answer is A. RATIONALE: Animal and human data suggest recovery of subclinical spinal cord damage starting at 6 months after irradiation and increasing over the next 2 years. It is often assumed that up to 50% of the prior dose can be "recovered" provided there is a sufficient time interval between treatments. REFERENCES: Nieder, et al. International Journal of Radiation Oncology, Biology, Physics (Int J Radiat Oncol Biol Phys). 2006;66:1446-49. Kirkpatrick, et al. International Journal of Radiation Oncology, Biology, Physics (Int J Radiat Oncol Biol Phys). 2010;76:S42-49.

305. Which of the following brain tumors occurs most commonly in children? A. Astrocytoma B. Ependymoma C. Medulloblastoma D. Germinoma

answer is A. RATIONALE: Astrocytomas make up approximately 30% of pediatric brain tumors in patients younger than 15 years of age and 20% of patients between 15 to 19 years of age per the Central Brain Tumor Registry of the United States (CBTRUS). Medulloblastoma is the most common posterior fossa tumor. Germinoma is more common in adolescent males. Exam answers and rationales 92 American College of Radiology

234. Autologous stem cell transplantation for patients with relapsed Hodgkin lymphoma has been shown to: A. increase a patient's chances for progression-free survival. B. increase the toxicity risks, outweighing the treatment benefits. C. provide no significant benefit compared to radiation alone. D. be useful only in patients who achieve a complete response to induction chemotherapy.

answer is A. RATIONALE: Autologous stem cell transplantation (ASCT) has been proven to be a very important part of a salvage regimen in patients with refractory and relapsed Hodgkin lymphoma (HL) and is now the generally accepted standard of care for patients with primary refractory or first-relapsed HL. It has resulted in over 50% of progression-free survival and superior outcome compared to non-transplantation regimens. REFERENCES: Linch DC, Winfield D, Goldstone AH, et al. Dose intensification with autologous bone-marrow transplantation in relapsed and resistant Hodgkin's disease: Results of a BNLI randomized trial. Lancet. 1993;341:1051-1054. Schmitz N, Pfistner B, Sextro M, et al. Aggressive conventional chemotherapy compared with high-dose chemotherapy with autologous hemopoietic stem-cell transplantation for relapsed chemosensitive Hodgkin's disease: a randomized trial. Lancet. 2002;359:2065-2071.

20. Radiation therapy would be most appropriate for malignant phyllodes tumors of the breast in a patient with: A. close (1 mm) surgical margins after excision of recurrent disease initially treated by a simple mastectomy. B. close (2 mm) but negative surgical margins after a partial mastectomy for a 3 cm lesion. C. negative surgical margins after a simple mastectomy for a 5 cm lesion. D. negative surgical margins after a partial mastectomy for a 6 cm lesion.

answer is A. RATIONALE: Based on anecdotal experience, radiation may be selectively indicated for locally advanced recurrent phyllodes tumors. No series have shown a clinical benefit for the use of radiation therapy in the primary treatment of phyllodes tumors. The mainstay of therapy is surgical excision with widely negative (1 to 2 cm) surgical margins. REFERENCE: Calhoun KE, Lawton TJ, Kim JN, et al. Phyllodes tumors. Harris JR, Lippman ME, Morrow M, Osborne CK (eds). Diseases of the Breast. 4th ed. Chapter 65. Philadelphia: Lippincott Williams & Wilkins Publishers. 2010; 781-792.

53. Which of the following radioactive equilibriums results when a parent radionuclide with a very long half-life decays to a daughter radionuclide with a shorter half-life? A. Secular B. Temporary C. Equivalent D. Transient

answer is A. RATIONALE: By definition, secular equilibrium is established when the half-life of a parent radionuclide is significantly longer than the daughter radionuclide.

75. The quality of a CT image can be improved by: A. increasing the mAs. B. increasing the kVp. C. increasing the reconstruction field-of-view. D. using an anti-scatter grid.

answer is A. RATIONALE: By increasing mAs, you also increase the number of x-rays used to produce the CT image. By increasing the mAs you increase the signal-to-noise ratio and the contrast resolution. Increasing the kVp usually decreases image contrast. Increasing the reconstruction field-of-view increases the physical voxel size and, therefore, introduces blurring into the image. Anti-scatter grids are not used in conventional CT imaging, but they may be used for cone-beam CT devices. REFERENCE: Bushberg JT, et al. The Essentials Physics of Medical Imaging. 2nd ed. 2002. Chapter 13.

148. Which of the following treatment results is most likely to be associated with patients who have ocular melanoma? A. Brachytherapy and enucleation result in similar overall survival rates for medium-sized tumors. B. Enucleation results in a higher overall survival rate than brachytherapy for medium-sized tumors. C. Brachytherapy results in a lower local recurrence rate than charged particle irradiation for large-sized tumors. D. EBRT followed by enucleation results in a higher overall survival rate than enucleation alone for large choroidal tumors.

answer is A. RATIONALE: COMS report 18: Medium tumors: Both brachytherapy and enucleation yield 5-year overall survival rates of 82%. Per COMS report 10: Large tumors: EBRT adds no survival benefit to enucleation. Several studies have suggested lower rates of recurrence with charged particle irradiation.

142. The phase III CONKO-001 trial evaluated the efficacy and toxicity of which adjuvant treatment after complete resection of pancreatic cancer? A. Gemcitabine versus observation B. Gemcitabine and oxaliplatin versus gemcitabine C. 5-FU-based chemoradiation with gemcitabine versus 5-FU D. Preoperative versus postoperative chemotherapy

answer is A. RATIONALE: CONKO-001 is a study that is often quoted in the medical oncology literature to justify omission of adjuvant chemoradiation therapy for pancreatic cancer. It is worth knowing that this study exists. Treatment with gemcitabine for 6 months after complete resection of pancreatic cancer significantly increased overall survival compared with observation alone (presented, not yet published) and delayed the development of recurrent disease (published in JAMA, 2007). Exam answers and rationales Radiation Oncology In-Training Exam 2011 43

49. Ataxia telangiectasia (AT) cells are characterized by: A. the inability to repair PLD. B. exquisite sensitivity to UV radiation. C. nonfunctional DNA repair systems. D. x-ray survival curves with broad shoulders.

answer is A. RATIONALE: Cells from patients who are homozygous for defective or lost ATM genes do not repair PLD, are exquisitely sensitive to ionizing radiation (not UV radiation), and are characterized by x-ray survival curves with little or no shoulder. The defect leading to this phenotype is not a DNA repair deficiency per se, but rather, a defect in the signaling pathways that coordinate damage recognition, recruitment of repair complexes, and cell cycle checkpoint control. Exam answers and rationales Radiation Oncology In-Training Exam 2011 15

41. Which of the following genetic conditions is associated with an increased risk for the development of acute lymphoblastic leukemia? A. Down syndrome B. Warner syndrome C. Christmas syndrome D. von Hippel-Lindau syndrome

answer is A. RATIONALE: Down syndrome is associated with a 15% increased risk of developing acute lymphoblastic leukemia (ALL). REFERENCE: DeVita VT, Hellman S, Rosenberg SA (eds). Cancer: Principles & Practice of Oncology. 6th ed. Lippincott: 2001; p 2407.

283. Which of the following salivary gland carcinomas has the worst prognosis? A. Ductal B. Mucoepidermoid C. Polymorphous D. Acinic cell

answer is A. RATIONALE: Ductal carcinoma of the salivary gland is rare. It is characterized by its histologic resemblance to ductal carcinoma of the breast and prostate with a highly aggressive biological behavior. Polymorphous adenocarcinoma is least aggressive.

266. Which of the following outcomes was associated with the use of PCI after chemotherapy for patients with extensive-stage SCLC, according to the EORTC 08993/22993 randomized trial? A. The rate of symptomatic brain metastasis was reduced by 25%. B. The benefit of PCI was limited to patients who had a complete response to chemotherapy. C. There was no improvement in the overall survival rate. D. The majority of patients died of CNS disease.

answer is A. RATIONALE: EORTC 08993/22993 explored the use of PCI in patients with extensive-stage small cell lung cancer (SCLC). Patients were only required to have a response to chemotherapy and did not need to have a complete response. The primary endpoint was the rate of symptomatic brain metastases, and this was significantly lower with the use of PCI (40% vs. 15%). Extracranial progression occurred in 90% of patients. Also an increase in overall survival was associated with treatment, on the order of 27% vs. 13% at 1 year. Patients were not required to have repeat brain imaging before the initiation of PCI, which is a controversial point for many with regard to interpreting the results of the trial. REFERENCE: Slotman B, et al. New England Journal of Medicine (N Engl J Med). 2007 Aug 16;357(7):664-72. Exam answers and rationales Radiation Oncology In-Training Exam 2011 81

238. According to the EORTC 22921 randomized trial, which outcome was significantly improved by chemotherapy administered concurrently with preoperative radiation therapy or as adjuvant therapy in patients with locally advanced rectal cancers? A. Local control B. Distant metastases-free survival C. Overall survival D. Late toxicity

answer is A. RATIONALE: EORTC 22921 randomized 1011 patients with resectable T3 or T4N0 rectal cancer by 2 x 2 randomization to either receive 1) preoperative radiation therapy alone; 2) preoperative chemoradiotherapy alone; 3) preoperative radiation therapy and postoperative chemotherapy; or 4) preoperative chemoradiotherapy and postoperative chemotherapy. With a median follow-up of 5.4 years, there was no significant difference in overall survival between the groups that received chemotherapy preoperatively or postoperatively (p=0.12). However, patients that received chemotherapy either preoperatively or postoperatively were found to have significantly lower rates of local recurrence compared to patients who did not receive chemotherapy (p=0.002). Specifically, the 5-year cumulative incidence rates of local recurrence were 8.7%, 9.6%, and 7.6% for patients treated with chemotherapy preoperatively, postoperatively, and both, respectively compared to 17.1% among patients treated with radiation therapy alone. The cumulative incidence of distant metastases did not differ significantly according to the preoperative or the postoperative treatment (p=0.14, p=0.62, respectively). There was no difference in late toxicity between the four treatment groups. REFERENCE: Bosset JF, Collette L, Calais G, et al. Chemotherapy with preoperative radiotherapy in rectal cancer. New England Journal of Medicine (N Engl J Med). 2006;355:1114-23. Exam answers and rationales 72 American College of Radiology

227. Which of the following clinical features is associated with pleural malignant mesothelioma? A. Approximately 80% of patients are male. B. It more commonly occurs in patients less than 50 years old. C. Chest pain occurs in less than 30% of patients. D. Weight loss is a common presenting symptom.

answer is A. RATIONALE: Eighty percent of patients with pleural malignant mesothelioma are male, and patients commonly present with a pleural effusion associated with breathlessness accompanied by chest wall pain (more than 60% of patients). The combination of an unexplained pleural effusion and pleural pain should raise the suspicion of malignant mesothelioma, even if the initial cytologic findings are negative. Weight loss and fatigue are common later in the progression of pleural mesothelioma but are less common at presentation (occurring in less than 30% of patients). Although a cytologic diagnosis can be made quickly, malignant mesothelioma is usually not diagnosed until 2 or 3 months after the onset of symptoms; delays of this length are especially frequent in centers in which the disease is uncommon. Mesothelioma is occasionally discovered incidentally on routine chest radiography. The risk of developing mesothelioma increases with age. Clubbing of the digits occurs in less than 1% of cases.

279. Which of the following management strategies would be most appropriate after excisional biopsy is performed for a patient with localized Langerhans cell histiocytosis without organ dysfunction or symptoms? A. Expectant observation B. Chemotherapy and radiation therapy C. Radiation therapy alone D. Chemotherapy alone

answer is A. RATIONALE: Excisional biopsy followed by expectant observation is the most appropriate management strategy for a patient with localized Langerhans cell histiocytosis without organ dysfunction or symptoms. Further treatment is considered for patients with multiple sites of involvement, unresectable, or recurrent disease. Radiation therapy can be considered in unifocal unresectable disease sites or in those not responding to chemotherapy. REFERENCE: Halperin, Constine, Tarbell, Kun. Langerhans cell histiocytosis. Pediatric Radiation Oncology. 4th ed. pp 487-508. Exam answers and rationales Radiation Oncology In-Training Exam 2011 85

239. Which clinical stage has been eliminated from the updated FIGO staging system for cervical cancer? A. Stage 0 B. Stage IA1 C. Stage IB2 D. Stage IIIB

answer is A. RATIONALE: FIGO staging no longer includes Stage 0 or carcinoma in situ. REFERENCE: AJCC Cancer Staging Manual. 7th ed. 2009.

272. Which of the following doses of prophylactic cranial irradiation would be most appropriate for a 4-year-old boy with high-risk acute lymphoblastic leukemia? A. 18 Gy B. 23 Gy C. 36 Gy D. 45 Gy

answer is A. RATIONALE: For prophylactic treatment, 18 Gy is sufficient. Recent protocols use even lower doses of 12 Gy.

167. Which of the following x-ray energies would be best for treating superficial skin lesions? A. 050 keV B. 500 keV C. 000.25 MeV D. 002.5 MeV

answer is A. RATIONALE: For superficial clinical targets, the photon energy with the lowest penetration would be preferred. The lowest energy among the choices given is 50 keV, which is a useful energy for treating superficial skin lesions.

175. Which of the following is an advantage of film over ion chambers for IMRT quality assurance? A. Better spatial resolution B. More accurate dosimetry C. Easier data processing D. Easier measurement

answer is A. RATIONALE: Generally speaking, ion chambers are the standard for absolute dosimetry. They are generally easier to use than film because they don't require complex film processing and can be irradiated multiple times (as compared to "one-time-only" film dosimetry). However, ion chambers do not boast the spatial resolution of film and are especially confounded at regions of high-dose gradients.

160. Which of the following proteins phosphorylate histone H2AX in response to DNA damage? A. ATM and ATR B. MLH1 and MSH2 C. BRCA1 and BRCA2 D. NBS1 and MRE11

answer is A. RATIONALE: Histone H2AX is phosphorylated at the serine 139 amino acid moiety in response to DNA double strand breaks produced by ionizing radiation exposure. ATM and ATR, members of the PI3K-like kinase family, phosphorylate H2AX to form H2AX. This, in turn, plays a critical role in the recruitment of DNA repair proteins to sites of DNA double strand breaks. MLH1, MSH2, BRCA1/2, NBS1, and MRE11 are proteins that participate in DNA damage sensing, screening, or repair, but are not protein kinases per se.

231. Which of the following outcomes was associated with the addition of temozolomide to EBRT for patients with glioblastoma, according to long-term follow-up of the EORTC/NCIC randomized (Stupp, et al.) trial? A. Improved overall survival rate regardless of MGMT status B. No survival benefit in patients older than 70 years of age C. Survival benefit only for patients with a KPS of 80 D. Survival rate of 3% at 5 years

answer is A. RATIONALE: In the updated publication, overall survival was 10% after chemoradiation at 5 years. Patients older than 70 years of age were not eligible to be enrolled in this trial, so controversy remains as to the role of temozolomide in this group of patients. REFERENCE: Stupp, et al. EORTC/NCIC randomized trial. Lancet Oncology. 2009 May;10(5):459-466, (S1470-2045).

230. Which of the following factors is most important in determining the estimated fetal dose for EBRT? A. Distance of the fetus from the closest radiation field edge B. Number and energy of the photon beams used C. Size of the planning target volume D. Size of the treatment volume

answer is A. RATIONALE: It is assumed that the fetus receives only scatter radiation and not any primary radiation. To a good approximation, scatter radiation decreases exponentially perpendicular to the beam edge. The closest beam edge, therefore, is the largest contributor to scatter radiation. The size of the PTV may affect the number of beams used to treat the target, but still the closest beam edge affects the scattered dose to the fetus the most. The size of the treatment volume is influenced by the number and energy of the photon beams used, which influences the scattered dose. However, if the beam geometry does not change, the effect of the number is still less than the distance of the fetus from the closest radiation field edge. REFERENCE: AAPM TG-36 Report.

120. When should initial imaging surveillance be performed once conformal radiation therapy has started for a patient with cystic craniopharyngioma? A. 1 to 2 weeks after the start of therapy B. 6 to 8 weeks after the start of therapy C. 3 to 6 months after the start of therapy D. 8 to 12 months after the completion of therapy

answer is A. RATIONALE: It is important to minimize the amount of normal brain that is in the treatment fields to reduce toxicity with conformal planning and reproducible in an effort to keep the treated volume as low as possible. Craniopharyngioma cysts, however, have been shown to change in size and shape during and after radiation therapy. Adaptive radiation therapy techniques have been used to ensure adequate conformal coverage of the tumor and the associated cysts. Weekly-biweekly imaging has been used to monitor the size and shape of the cyst. If there is a significant change noted, replanning with the new tumor volume can be performed. REFERENCE: Winkfield KM, et al. Surveillance of craniopharyngioma cyst growth in children treated with proton radiation therapy. International Journal of Radiation Oncology, Biology, Physics (IJROBP). 2009 Mar 1;73(3):716-21.

46. Which of the following management strategies is most appropriate for a 34-year-old woman with a 3 cm lymphepithelioma limited to the nasopharynx and no evidence of lymph node involvement? A. Radiation therapy to a total dose of 70 Gy alone B. Carboplatin and paclitaxel weekly with concurrent radiation therapy to a total dose of 70 Gy C. Cisplatin at 30 mg/m2 weekly with hyperfractionated radiation therapy to a total dose of 76.4 Gy D. Cisplatin at 100 mg/m2 every 3 weeks with concurrent radiation therapy to a total dose of 70 Gy, followed by 3 cycles of adjuvant cisplatin and 5-FU

answer is A. RATIONALE: Knowing that radiation therapy alone is a very effective treatment for early-stage nasopharyngeal cancer is vital. Chemotherapy has been proven to be effective for stage III and IV disease only. Exam answers and rationales 14 American College of Radiology

7. Which of the following findings is NOT an adverse prognostic factor for squamous cell carcinoma of the penis? A. Koilocytosis B. Vascular invasion C. Perineural invasion D. High histologic grade

answer is A. RATIONALE: Koilocytosis is associated with low-grade tumors. Vascular invasion, perineural invasion, and high histologic grade are important adverse prognostic factors for squamous cell carcinoma of the penis. REFERENCE: Cubilla. The role of pathologic prognostic factors in squamous cell carcinoma of the penis. World Journal of Urology (World J Urol). 2009;27:169-177. Exam answers and rationales Radiation Oncology In-Training Exam 2011 3

315. The largest number of leptomeningeal carcinomatosis cases per year occurs with which of the following primary malignancies? A. Breast cancer B. Thyroid cancer C. Non-small cell lung cancer D. Malignant melanoma

answer is A. RATIONALE: Leptomeningeal carcinomatosis (LC) is found in 23% of patients with melanoma, 9% to 25% of patients with small cell lung cancer, and 5% of patients with breast cancer. Breast cancer is the most common diagnosis in patients with LC, however, because of the higher overall incidence of breast cancer in comparison. Exam answers and rationales Radiation Oncology In-Training Exam 2011 95

215. Which of the following postoperative radiation therapy doses should be administered to a 7-year-old girl with a completely resected, 9 cm embryonal rhabdomyosarcoma of the vagina with negative histologic margins? A. 00 Gy B. 36 Gy C. 41.4 Gy D. 50.4 Gy

answer is A. RATIONALE: No radiation is required for a stage 1, group I embryonal rhabdomyosarcoma of the vagina. Exam answers and rationales Radiation Oncology In-Training Exam 2011 65

268. Approximately what percent oxygen concentration yields cellular radiosensitivity halfway between the fully aerobic and fully anoxic response? A. 00.5% B. 02% C. 05% D. 10%

answer is A. RATIONALE: Oxygen's "k-curve," a plot of relative cellular radiosensitivity as a function of oxygen concentration, suggests that the oxygen concentration that yields radiosensitivity halfway between the fully aerobic and fully anoxic response is approximately

36. P-glycoprotein expression in patients with osteosarcoma is associated with: A. increased mortality. B. increased sensitivity to doxorubicin. C. decreased sensitivity to radiation. D. decreased expression of EGFR.

answer is A. RATIONALE: P-glycoprotein is a product of the multidrug resistance (MDR1) gene. It can remove doxorubicin from osteosarcoma cells, thereby decreasing drug sensitivity. Its expression is associated with a several-fold increase in the risk of mortality. Its expression is independent of the epidermal growth factor receptor (EGFR) status for tumors, and it has not been correlated with radiation sensitivity. REFERENCE: AJCC Cancer Staging Manual. 7th ed. 2009. pp 336-7.

68. According to the RTOG 97-14 trial, which response was associated with the use of a single fraction of 8 Gy versus 30 Gy/10 fractions for palliation of painful breast or prostate bone metastases? A. There was no difference in the complete or partial pain response between the two treatment groups. B. The complete pain response was higher with the multiple-dose fraction. C. The partial pain response was higher with the single-dose fraction. D. Higher rates of progressive pain occurred with the single-dose fraction.

answer is A. RATIONALE: Patients were randomized to 8 Gy in a single fraction vs. 30 Gy in 10 fractions. There were no significant differences in complete (17%) vs partial (49%) response rates measured at 3 months between the two groups. Rates of progressive pain scores were 9% and 10%, respectively, for these two groups. REFERENCE: Hartsell, WF, Scott CB, Bruner, DW, et al. Randomized trial of short versus long course radiotherapy for palliation of painful bone metastases. Journal of National Cancer Institute. 2005;97:798-804.

83. A 55-year-old patient with stage T1bN2M0 adenocarcinoma of the right lung has received preoperative chemotherapy and 45 Gy of thoracic radiation therapy. The restaging CT scan of the chest after completion of chemoradiation shows progressive disease in both the primary tumor and the mediastinal lymph nodes. The best course of action would be to: A. continue with chemoradiation therapy to a dose of 66 Gy. B. proceed with definitive lobectomy and mediastinal lymph node dissection. C. add one more course of chemotherapy and 10.8 Gy of radiation therapy, followed by surgery. D. recommend no further cancer therapy.

answer is A. RATIONALE: Patients with progressive disease after preoperative chemoradiation did not undergo surgery in either the SWOG 88-05 or Intergroup 0139 (surgical arm) study due to the poor prognosis associated with these patients. Even though there is no good standard approach for such patients, it is reasonable to continue full chemoradiation to achieve a durable intrathoracic response and to minimize the risk of metastatic disease. REFERENCE: Albain KS, et al. Lancet. 2009.

42. Which treatment would be best in protecting normal cells and tissues from the damaging effects of ionizing radiation? A. Pentoxifylline, a mitigator of radiation-induced fibrosis, given for several weeks after irradiation B. Amifostine, a radioprotector of salivary glands, administered a few hours after irradiation C. Vitamin C, a nonspecific antioxidant supplement, given during radiation therapy D. Palifermin, a cytokine that protects bone marrow, given just prior to irradiation

answer is A. RATIONALE: Radiation mitigators such as pentoxifylline are typically given for extended periods after irradiation and are thought to interfere with the long, complicated molecular processes that culminate in the expression of normal tissue injury. Amifostine, a radioprotector, needs only to be present at the time of irradiation (and not hours after) to be effective, because its putative mechanism of action is free-radical based. The use of antioxidant supplements (such as vitamin C) by patients during the course of radiation and/or chemotherapy is generally discouraged because of the potential to protect tumors as well as normal tissues. Palifermin is a cytokine that stimulates the growth of keratinocytes (not bone marrow cells), and as such, has the potential to reduce oral mucositis associated with radiation therapy for head and neck cancers. Exam answers and rationales Radiation Oncology In-Training Exam 2011 13

197. Which of the following treatments is most appropriate for leptomeningeal metastases from breast cancer? A. WBRT for cranial nerve deficits B. Craniospinal irradiation C. Steroids to improve neurological symptoms D. Intrathecal chemotherapy after placement of a VP shunt

answer is A. RATIONALE: Radiation should be given to symptomatic areas whether or not structural disease is identified on imaging, as well as to sites of bulky disease. Radiation therapy is usually the most effective modality for treating focal leptomeningeal nodules. Radiation therapy to bulky sites may restore CSF flow. Use of complete neuro-axis irradiation is discouraged because it seldom controls the disease and is associated with significant toxicities, such as esophagitis and myelosuppression. Intrathecal chemotherapy cannot be given after placement of a VP shunt. REFERENCE: Fisher R, DeAngelis LM. Leptomeningeal metastasis. Harris JR, Lippman ME, Morrow M, Osborne CK (eds). Diseases of the Breast. 4th ed. Chapter

60. Radiation therapy has the greatest effect in the shortest amount of time for which of the following conditions? A. Acromegaly B. Prolactinoma C. TSH-secreting adenoma D. Cushing's disease

answer is A. RATIONALE: Radiation therapy (more commonly radiosurgery) has been effective in treating acromegaly with reduction in GH and IGF-1 by approximately 50% within the first two years and continued reduction over time. Radiation therapy is also effective in treating the other secreting adenomas; however, results may take longer.

176. What is the best management for a 60-year-old woman who has MALT lymphoma of the right orbit with negative results on systemic workup? A. Radiation therapy alone to 30 Gy B. Radiation therapy alone to 45 Gy C. Chemotherapy alone with R-CHOP D. Chemotherapy with CHOP, followed by radiation therapy to 30 Gy

answer is A. RATIONALE: Radiation therapy alone to a median dose of 30 Gy will result in a 99% to 100% local control rate. REFERENCES: Quyn-Thu Le, et al. Primary radiotherapy for localized orbital malt lymphoma. International Journal of Radiation Oncology, Biology, Physics (Int J Radiat Oncol Bio Phys). 2002;52(3):657-63. Tsang RW, et al. Journal of Clinical Oncology (J Clin Oncol). 2003;21(22):4157-64. Exam answers and rationales Radiation Oncology In-Training Exam 2011 53

13. Randomized trials for localized prostate cancer have shown that dose escalation improves which of the following endpoints? A. Biochemical failure-free survival B. Distant metastasis-free survival C. Overall survival D. Quality of life

answer is A. RATIONALE: Randomized trials have shown improvements in biochemical failure-free survival when dose escalation was used for patients with localized prostate cancer.

292. Which of the "Four R's of Radiotherapy" will increase cell survival for fractionated irradiation? A. Repair and repopulation B. Reoxygenation and repopulation C. Reassortment and reoxygenation D. Reassortment and repair

answer is A. RATIONALE: Repair eliminates radiation-induced damage while repopulation increases cell number between fractions, which increase cell survival. Reassortment facilitates the movement of surviving cells into more radiosensitive phases of the cell cycle. Reoxygenation allows oxygen — a potent radiosensitizer — better access to previously hypoxic regions of tumors. Reassortment and reoxygenation both have the net effect of reducing cell survival.

2.5 cm peripheral NSCLC? A. 50 Gy in 4 fractions using SBRT B. 66 Gy in 33 fractions using 3D-CRT C. 69.6 Gy in 58 fractions twice daily using 3D-CRT D. 70 Gy in 30 fractions using IMRT

answer is A. RATIONALE: SBRT with BED >100 Gy has been shown to provide >90% local control with minimal side effects for a peripherally located stage I NSCLC. SBRT is emerging as standard treatment for medically inoperable stage I NSCLC. Exam answers and rationales Radiation Oncology In-Training Exam 2011 31

35. The sensitivity of a screening test is used to detect the presence or absence of disease in the percentage of subjects: A. with disease who are classified as having disease. B. with disease who are classified as not having disease. C. without disease who are classified as having disease. D. without disease who are classified as not having disease.

answer is A. RATIONALE: Screening refers to the application of a test to people who as yet have no symptoms of a particular disease. It is classified as having a positive (disease likely) or negative (disease unlikely) finding. Diagnostic tests tell whether or not a subject actually has the disease. The performance of a screening test is considered by the sensitivity and specificity. The sensitivity is the percentage of subjects with disease who are classified as having disease and the specificity is the percentage of subjects without disease who are classified as not having disease. Those subjects with the disease should all be classified as having disease, and those subjects without the disease should be classified as not having disease. Therefore, a highly sensitive and specific test is preferred. Exam answers and rationales Radiation Oncology In-Training Exam 2011 11

98. Which statistical term has a high value when the variation in repeated measurements is poor? A. Standard deviation B. Number of subjects C. Mean D. Median

answer is A. RATIONALE: Standard deviation measures variability of data. For poor repeatability, there is considerable variation in repeated measurements.

206. What is the standard treatment for a supraglottic larynx carcinoma extending through the thyrohyoid membrane into the strap muscles? A. Total laryngectomy B. Radiation therapy alone C. Concurrent chemoradiation D. Induction chemotherapy, followed by radiation therapy

answer is A. RATIONALE: Standard treatment for a stage T4 lesion would require total laryngectomy.

11. What is the treatment of choice for an otherwise healthy 55-year-old man who has thymoma with pericardial involvement? A. Surgical resection B. Primary radiation therapy C. Radiation therapy, followed by surgery D. Chemoradiation, followed by surgery

answer is A. RATIONALE: Surgical resection is the treatment of choice for a thymoma. The goal of surgery is to completely excise the thymoma via total thymectomy with complete resection of contiguous and noncontiguous disease, which may require resecting adjacent structures such as the pericardium, phrenic nerve, pleura, lung, and even major vascular structures. Pericardial involvement is considered to be a resectable disease. Exam answers and rationales 4 American College of Radiology

92. What is the most appropriate treatment after gross total resection of a 3 cm, right frontal, low-grade astrocytoma in an otherwise healthy 40-year-old patient? A. Surveillance B. Temozolomide alone C. EBRT, followed by temozolomide D. EBRT alone

answer is A. RATIONALE: Surveillance would be the best option for a completely resected low-grade glioma. If recurrence is noted, radiation therapy should be considered. Chemotherapy is of unclear benefit in an adult with low-grade astrocytoma.

308. Which of the following systemic chemotherapy agents used alone in a patient with breast cancer has the highest risk for inducing congestive heart failure? A. Trastuzumab (Herceptin) B. Bevacizumab (Avastin) C. Lapatinib (Tykerb) D. Paclitaxel (Taxol)

answer is A. RATIONALE: Taxanes, in general, are not associated with congestive heart failure (CHF) or cardiomyopathy when used independently of anthracycline or trastuzumab. Taxanes are more likely to increase the risk of chest pain, myocardial infarction, or cardiac arrhythmias. Trastuzumab alone is associated with a 3% to 7% risk of CHF. The risk of CHF with adjuvant trastuzumab and paclitaxel is 13%; trastuzumab + anthracycline = 27% risk of CHF. The CHF risk with lapatinib is 1.5% to 2%, and the CHF risk with bevacizumab is 1.4% to 4%. Bevacizumab is more commonly associated with an increased risk for hypertension (12% to 34%), arterial embolic events (4.4% to 8.5%), and deep vein thrombosis (6% to 9%). REFERENCE: Aspitia AM, Perez EA. Side effects of systemic therapy: neurocognitive, cardiac, and secondary malignancies. Harris JR, Lippman ME, Morrow M, Osborne CK (eds). Diseases of the Breast. 4th ed. Chapter 55. Philadelphia: Lippincott Williams & Wilkins Publishers. 2010; 698-701. Exam answers and rationales Radiation Oncology In-Training Exam 2011 93

25. According to the EORTC randomized trial, the use of a radiation boost versus no boost for early-stage breast cancer most benefited patients who were what year of age? A. 35 B. 45 C. 55 D. 65

answer is A. RATIONALE: The EORTC randomized trial results showed an age-related difference in the magnitude of the absolute benefit of a radiation boost for patients with early-stage breast cancer. There was an overall similar relative decrease in the risk of local recurrence with a boost across age groups, but the baseline risk of recurrence was much higher in the younger women. REFERENCE: Bartelink, et al. Journal of Clinical Oncology (J Clin Oncol). 2007;25:3259-3265.

259. Which of the following induction chemotherapy regimens would be most appropriate before administration of radiation therapy for a patient with stage T2N2 carcinoma of the pyriform sinus? A. Docetaxel, cisplatin, and 5-FU B. Docetaxel and cisplatin only C. Cisplatin and 5-FU only D. Cisplatin and cetuximab

answer is A. RATIONALE: The GORTEC trial of patients with larynx/hypopharynx cancer shows that induction TPF (docetaxel, cisplatin, and 5-FU) induction chemotherapy is superior to PF (cisplatin and 5-FU) induction chemotherapy for organ preservation. REFERENCE: Calais G, et al. GORTEC 2000-01 trial. ASCO. 2006.

2. According to the Intergroup 0123 trial, patients who received 64.8 Gy compared to 50.4 Gy for esophageal cancer had: A. similar overall survival rates. B. significantly higher overall survival rates. C. significantly lower overall survival rates. D. significantly improved local control.

answer is A. RATIONALE: The Intergroup 0123 trial showed no significant difference in survival between the two arms (64.8 Gy compared to 50.4 Gy) for patients with esophageal cancer. Concurrent 5-FU and cisplatin was administered in both arms. REFERENCE: Minsky, et al. Journal of Clinical Oncology (J Clin Oncol). 2002.

47. According to the MOSAIC trial, which outcome was demonstrated in patients with resected colon cancer? A. Overall survival was improved with the addition of oxaliplatin to 5-FU and leucovorin (FOLFOX) versus 5-FU and leucovorin alone. B. Disease-free survival was improved with FOLFOX in patients with stage II and stage III disease. C. Metastatic disease was decreased with the addition of bevacizumab to 5-FU, leucovorin, and oxaliplatin therapy. D. Toxicity was decreased with FOLFOX versus 5-FU and leucovorin alone.

answer is A. RATIONALE: The MOSAIC trial randomized more than 2000 patients with stage II and stage III colon cancer after resection to 5-FU and leucovorin alone versus 5-FU and leucovorin with the addition of oxaliplatin (FOLFOX). Overall survival (OS) and disease-free survival (DFS) were significantly improved in the FOLFOX group. However, DFS was significantly improved only in patients with stage III disease and not in patients with stage II disease. Bevacizumab (Avastin) was not used in this trial. Toxicity was significantly increased in the FOLFOX arm.

192. Which of the following treatments is most appropriate for a patient with stage T2N0 glottic larynx carcinoma with impaired vocal cord mobility? A. Hyperfractionated radiation therapy to escalate radiation dose B. Conventionally fractionated radiation therapy of 2 Gy per day C. Cisplatin with delayed concomitant boost radiation therapy D. Induction chemotherapy, followed by radiation therapy for responders

answer is A. RATIONALE: The RTOG 95-12 trial demonstrated that hyperfractionated radiation improved local control compared to conventional treatment (per ASTRO 2006). Exam answers and rationales Radiation Oncology In-Training Exam 2011 57

153. The bremsstrahlung process produces: A. a continuous spectrum of x-rays. B. a continuous spectrum of electrons. C. monoenergetic electrons. D. monoenergetic x-rays.

answer is A. RATIONALE: The continuous spectrum of photons, up to the maximum kinetic energy of the incident electron beam, is a characteristic feature of bremsstrahlung ("braking" radiation produced by decelerated electrons).

205. A 30-year-old man with stage I seminoma is scheduled to undergo adjuvant radiation therapy with a standard paraaortic field at 2 Gy per fraction. What is the approximate daily dose per fraction to the remaining testis without and with a gonadal shield? Without Shield With Shield A. 2 cGy 0.7 cGy B. 4 cGy 0.7 cGy C. 4 cGy 1.5 cGy D. 8 cGy 1.5 cGy

answer is A. RATIONALE: The dose to the testes bears significance due to the relatively low doses that can cause transient azoospermia (~50 cGy), total azoospermia (~100 cGy), and sterilization (~200 cGy). In a study that measured testicular dose using thermoluminescent dosimeters, the daily dose to the testes for a man treated at 1.8 Gy per fraction to a paraaortic field is 1.86 cGy per fraction without gonadal shielding, and 0.65 cGy per fraction with gonadal shielding. A gonadal shield can therefore reduce the dose to the testes twofold to threefold. Meanwhile, doses are roughly doubled when the ipsilateral hemipelvis is included (option C). Keeping the dose to the testes <20 cGy can help avoid impairment of spermatogenesis; this scatter dose is typically only achievable when using only a paraaortic field and a scrotal shield. REFERENCE: Bieri, et al. Radiation Oncology. 1999;50:349-353.

82. As a poly-energetic spectrum of photons passes through homogeneous material, what is the quality of the first half-value layer (HVL) in relation to subsequent HVLs? A. The first HVL is the thinnest. B. The first HVL is the thickest. C. All HVLs have the same thickness. D. The first HVL is exponentially thicker than the subsequent HVLs.

answer is A. RATIONALE: The first HVL is the thinnest. As the poly-energetic beam passes through additional material, the lower-energy photons are more preferentially absorbed, and the beam is "hardened," making the subsequent HVLs thicker than the first. Exam answers and rationales Radiation Oncology In-Training Exam 2011 25

301. What is the incidence of lymphedema in the upper extremities for the following breast cancer treatments? A. 10% after sentinel lymph node biopsy B. 15% after axillary irradiation alone C. 20% after level I/II lymph node dissection alone D. 50% after level I/II lymph node dissection and axillary irradiation

answer is A. RATIONALE: The incidence of arm edema after irradiation alone to the axilla is less than 5%. The incidence of arm edema is 10% to 15% following axillary lymph node dissection (AXLND) alone. The addition of axillary irradiation increases the risk of arm edema, and the ranges vary with comorbidity of obesity and length of follow-up. Reported rates range from 10% to 77%. REFERENCE: Morrow M, Harris JR. Treatment of the axilla and other regional nodes. Harris JR, Lippman ME, Morrow M, Osborne CK (eds). Diseases of the Breast. 4th ed. Chapter 46. Philadelphia: Lippincott Williams & Wilkins Publishers. 2010; p 597. Exam answers and rationales Radiation Oncology In-Training Exam 2011 91

243. What is the main reason that proton therapy may be preferred over photon therapy? A. The integral dose is much smaller. B. The beam penumbra is much sharper. C. The dose distributions are much more conformal. D. The RBE value is significantly higher.

answer is A. RATIONALE: The integral dose in proton treatments is considerably lower than that in photon therapy. Option C is incorrect if only the high-dose volume (i.e., 90%) is considered. Although the RBE for protons is higher than that for photons, it is not significantly higher. The RBE for protons only is around 1.1.

66. Which stage (AJCC) is most appropriate for adenocarcinoma of the uterus that invades 3 mm of 9 mm of the myometrium, has a positive peritoneal cytology, and does not involve the pelvic and paraaortic lymph nodes? A. IA B. II C. IIIA D. IIIC2

answer is A. RATIONALE: The most recent AJCC Cancer Staging Manual no longer considers positive peritoneal cytology in the staging of endometrial cancer. In the previous edition, this would be considered to be stage IIIA disease. Stage IA tumor invades less than one half of the myometrium and does not extend to the cervical stroma, serosa, adnexa, or lymph nodes. REFERENCE: AJCC Cancer Staging Manual. 7th ed. 2009. Exam answers and rationales 20 American College of Radiology

220. Which of the following radiation therapy regimens is best for a patient with stage T1 glottic larynx carcinoma? A. Keeping total treatment time to <6 weeks B. Hyperfractionation to allow dose escalation C. 1.8 Gy per fraction to minimize toxicity D. 63 Gy in 28 fractions of IMRT

answer is A. RATIONALE: The optimal treatment regimen is 63 Gy in 28 fractions, which yields the best balance of toxicity and efficacy. There is no advantage of hyperfractionated radiation therapy for stage T1 glottic lesions. At the present time, IMRT is not appropriate for a stage T1 glottic tumor. REFERENCE: Le QT, et al. Influence of fraction size, total dose, and overall time on local control of stage T1-T2 glottic carcinoma. International Journal of Radiation Oncology, Biology, Physics (Int J of Radiat Oncol Biol Phys). 1997;37:115-126.

201. Which of the following radiation dose regimens is most appropriate for group III orbital embryonal rhabdomyosarcoma? A. 45 Gy in 25 daily fractions B. 50.4 Gy in 28 daily fractions C. 54 Gy in 30 daily fractions D. 59.4 Gy in 54 twice-daily fractions

answer is A. RATIONALE: The recommended radiation dose used for group III orbital rhabdomyosarcoma in IRS V is 45 Gy in 25 fractions with good local control and survival. A dose of 50.4 Gy is given to non-orbital gross disease, 59.4 Gy given with a hyperfractionated regimen was not found to be better than 50.4 Gy given in a daily regimen in a prospective randomized IRS study. REFERENCE: Friedman, Tarbell, Constine. Rhabdomyosarcoma. Halperin, Constine, Tarbell, Kun (eds). Pediatric Radiation Oncology. 4th ed. pp 319-346. Exam answers and rationales 60 American College of Radiology

273. How would the addition of a radiosensitizer affect the radiation dose response curve for tumor control probability (TCP) for a human tumor xenograft grown in an immunosuppressed mouse? A. Shift curve to the left, toward lower total doses B. Shift curve to the right, toward higher total doses C. Shift curve upwards, toward higher tumor control probabilities D. Shift curve downwards, toward lower tumor control probabilities

answer is A. RATIONALE: The sigmoid-shaped tumor control probability curve would be shifted to the left, that is, toward a lower total dose that achieves the same level of tumor control. REFERENCE: Joiner and van der Kogel. Basic Clinical Radiobiology. 4th ed. Chapters 7 and 17. Exam answers and rationales Radiation Oncology In-Training Exam 2011 83

112. What is the remission odds ratio of men to women if 20 of 70 women achieve remission compared to 30 of 65 men? A. 2.14 B. 1.62 C. 0.62 D. 0.47

answer is A. RATIONALE: This question can be calculated as follows: Since 30 of 65 men achieved remission, 35 men did not achieve remission. Thus, the odds of achieving remission among men would be 30/35. Similar logic indicates that the odds of achieving remission among women would be 20/50. Therefore, the odds ratio comparing men with women is as follows: 30/35 divided by 20/50 = 2.14. Exam answers and rationales 34 American College of Radiology

23. Tumors arising from the penile urethra most frequently metastasize to which of the following lymph nodes? A. Inguinal B. External Iliac C. Internal Iliac D. Obturator

answer is A. RATIONALE: Tumors arising from the penile urethra most frequently metastasize to the inguinal lymph nodes, while tumors arising from the bulbomembranous and prostatic urethra most frequently metastasize to the pelvic lymph nodes. REFERENCE: Perez and Brady. Principles and Practice of Radiation Oncology. 5th ed. p 1520.

146. A patient with a defect in nonhomologous end-joining repair of DNA double strand breaks is also likely to demonstrate: A. immune deficiency. B. increased sensitivity to cisplatin. C. decreased sensitivity to ionizing radiation. D. excess chromatid-type aberrations.

answer is A. RATIONALE: V(D)J recombination is the process whereby segments of the immunoglobulin genes and the T-cell receptor are rearranged during development of the vertebrate immune system. V(D)J recombination is initiated by the introduction of a single strand DNA break at specific sites in the DNA by the RAG1/RAG2 protein complex. This single strand break is converted to a DNA double strand break, which is repaired during the process of V(D)J recombination by the nonhomologous end-joining (NHEJ) machinery. Therefore, defective NHEJ leads to immune deficiency.

119. Which of the following factors is most important when matching multiple photon fields? A. Beam divergence B. Beam penumbra at depth C. Aperture matching at the surface D. Internal scatter inside the patient

answer is A. RATIONALE: While options B-D represent issues for treating patients with multiple fields, geometric beam divergence is the most important issue when treating patients with multiple matching photon fields (i.e., craniospinal, multi-port breast, and 3D head and neck radiation therapy). Exam answers and rationales 36 American College of Radiology

116. Which of the following adjuvant radiation therapy regimens is most appropriate for an otherwise healthy 65-year-old man who has stage pT3N2M0 NSCLC with negative surgical margins? A. 50 Gy in 25 fractions B. 60 Gy in 30 fractions C. 70 Gy in 35 fractions D. 74 Gy in 37 fractions

answer is A. RATIONALE: With negative surgical margins and positive mediastinal lymph nodes, the recommended postoperative radiation dose is 50 Gy in 25 fractions. Doses of 60 Gy and above are recommended for positive surgical margins.

134. The risk for developing optic neuropathy after administration of 54 Gy of radiation in 28 fractions to a meningioma adjacent to the optic chiasm is: A. 1% at 5 years. B. 02% at 1 year. C. 15% at 1 year. D. 15% at 5 years.

answer is A. REFERENCE: Mayo C, Martel MK, Marks LB, Flickinger J, Nam J, Kirkpatrick J. QUANTEC: Radiation dose-volume effects of optic nerves and chiasm. International Journal of Radiation Oncology, Biology, Physics (Int J Radiat Oncol Biol Phys). 2010 Mar 1;76(3 Suppl):S28-35. Review.

228. What is the staging and group classification for a 3 cm alveolar rhabdomyosarcoma of the thigh with positive microscopic margins and inguinal lymph node involvement? A. Stage 3, group III B. Stage 3, group II C. Stage 2, group III D. Stage 2, group II

answer is B. RATIONALE: A 3 cm lesion in the extremity with positive lymph nodes would be stage 3. Positive microscopic margins are classified as group II.

133. According to ICRU Report 62, a planning target volume (PTV) margin is achieved by creating a: A. composite PTV margin that does not extend into any organ at risk. B. composite PTV margin as a compromise of disease control and minimization of risk complication. C. PTV margin that is derived by using a sigma squared formalism of each individual uncertainty. D. PTV margin that is a linear addition of the individual uncertainties.

answer is B. RATIONALE: A composite PTV margin should be created as a compromise of disease control and minimization of risk complication. This may or may not include the portion of the PTV that extends into an organ at risk (OAR). A linear addition of all uncertainties would yield an excessively large PTV. A quantitative approach would require a detailed knowledge of the magnitude of each uncertainty, and this is often not feasible.

174. A linear-quadratic dose response for dicentric formation will be produced by: A. low LET radiation delivered at a low dose rate. B. low LET radiation delivered at a high dose rate. C. high LET radiation delivered at a low dose rate. D. high LET radiation delivered at a high dose rate.

answer is B. RATIONALE: A dicentric chromosome aberration can be produced when two chromosome breaks are formed in two different chromosomes, and these breaks are in close enough proximity to one another with respect to space and time that an improper exchange occurs. For high LET radiations, both breaks are likely to be produced along the same charged particle track, resulting in a linear dose response, regardless of dose rate. For low LET radiations, while some exchanges can be produced by single particle tracks, an increasing number are produced by two independent tracks as the dose increases, yielding a linear-quadratic dose response. At a low dose rate, most dicentrics result from single track events. Two track exchanges are reduced because the tracks are spaced out temporally, allowing time for one of the lesions potentially involved in the dicentric to be repaired before the second lesion can be produced. This results in a linear dose response.

73. Hypofractionated radiation therapy is used to treat prostate cancer because these tumors: A. express a high EGFR. B. have a low alpha/beta ratio. C. are radioresistant. D. are hypoxic.

answer is B. RATIONALE: A hypofractionated radiation therapy approach is aimed to overcome the low alpha/beta ratio of prostate tumors and to deliver a higher biologically effective dose (BED). Exam answers and rationales 22 American College of Radiology

278. What percent of patients with SCLC present with limited-stage disease? A. 10% to 20% B. 30% to 40% C. 50% to 70% D. >90%

answer is B. RATIONALE: A minority of patients with small cell lung cancer (SCLC) present with limited-stage disease, and this proportion has not changed significantly over the last 30 years (32.5% in 1973, 39.6% in 2002). REFERENCE: Govindan R, et al. Journal of Clinical Oncology. 2006 October 1;24(28):4539-4544.

135. Approximately 75% of prostate cancers originate from which zone? A. Central B. Peripheral C. Transitional D. Anteromedial

answer is B. RATIONALE: A straight-forward question on the distribution of prostate cancers highlights that 75% of the cancers originate from the peripheral zone. Exam answers and rationales Radiation Oncology In-Training Exam 2011 41

137. Which of the following histological types of primary cutaneous non-Hodgkin lymphoma has the worst outcome? A. Follicular B. Peripheral T-cell C. Anaplastic large cell D. Mycosis fungoides

answer is B. RATIONALE: According to large European registry data using WHOEORTC classification, patients with primary cutaneous peripheral T-cell lymphoma have the worst disease outcome with a <20% disease-free survival rate at 5 years. REFERENCE: Willemze R, Jaffe ES, Burg G, Cerroni L, et al. WHO-EORTC classification for cutaneous lymphomas. Blood. 2005 May 15;105(10):3768-85.

103. Which of the following exams would be best for diagnosis of spinal cord compression in a patient with multiple myeloma? A. CT B. MRI C. PET D. Bone scan

answer is B. RATIONALE: An MRI is the best study for diagnosis of spinal cord compression in patients with multiple myeloma. REFERENCE: Dimopoulos MA, Moulopoulos LA, Maniatis A, et al. Solitary plasmacytoma of bone and asymptomatic multiple myeloma. Blood. 2000;96(6):2037-44.

114. What effector caspase is activated by both the extrinsic death receptor and the intrinsic mitochondrial pathways that trigger cells to undergo apoptosis? A. Caspase 2 B. Caspase 3 C. Caspase 8 D. Caspase 9

answer is B. RATIONALE: Apoptosis is triggered by two major signaling pathways: the extrinsic death-receptor pathway and the intrinsic mitochondrial pathway. Both signaling pathways converge on caspase 3, an effector caspase that causes cells to undergo apoptosis. REFERENCE: Hall and Giaccia. Radiobiology for the Radiologist. 6th ed. Chapter 17. Figures

58. Which of the following statements about thermotolerant cells is true? A. Cells heated at 39°C to 42°C remain thermotolerant for several days after hyperthermia treatment is ceased. B. It may take as long as a week after hyperthermia treatment for thermotolerant cells to revert to their normal heat sensitivity. C. Because of thermotolerance, a patient should receive four hyperthermia treatments per field per week. D. The development of thermotolerance can cause a decrease in the slope of the heat survival curve by as much as a factor of two.

answer is B. RATIONALE: At temperatures of 39°C to 42°C, thermotolerance can occur during heating, but it does not persist for days thereafter. Thermotolerance is a substantial effect, where the slope of the heat survival curve can decrease by a factor of 4- to 10-fold. Because of this, use of hyperthermia in the clinic is generally limited to once, or at the most twice, per week. Exam answers and rationales 18 American College of Radiology

233. Compared to radiation therapy alone, concurrent chemoradiation for a patient with stage T3N0 supraglottic larynx cancer has been shown to improve: A. overall survival by 10%. B. larynx preservation by 15% to 20%. C. distant metastasis-free survival by 20%. D. swallowing function by 30%.

answer is B. RATIONALE: Based on Intergroup trial 91-11, there was no difference in overall survival. The laryngeal preservation rate was improved from 56% to 78%. Distant metastasis-free survival at 5 years was improved from 22% to 12%. There was no significant difference in swallowing function.

144. A 3 cm carcinoma in the right upper lobe of the lung with a separate nodule in the same lobe and aortopulmonary (AP) window adenopathy would be classified as stage: A. T3N2. B. T3N3. C. T4N2. D. T4N3.

answer is B. RATIONALE: Based on the new, 2009 IASLC staging system, the T stage for a separate tumor nodule in the same lobe as the primary tumor has been changed from stage T4 to stage T3. For primary right lung cancer, the aortopulmonary (AP) window lymph node is considered to have contralateral mediastinal lymph node involvement, and therefore, should be stage N3 disease.

251. According to the German Rectal Cancer Study, what was the rate of pathologic complete response in patients receiving preoperative chemoradiotherapy? A. 00% B. 08% C. 24% D. 32%

answer is B. RATIONALE: Between 1995 and 2002, the German Rectal Cancer Study Group enrolled 823 patients in a phase III trial comparing conventionally fractionated preoperative radiation therapy given with concurrent 5-FU chemotherapy (n=421) with the same treatment given postoperatively (n=402) in patients with clinical stage T3/T4 or node-positive rectal cancer between 1995 and 2002. Preoperative radiation therapy consisting of a total of

209. Neglecting patient scatter, what is the dose estimate at the field edge if a rectangular spinal field is prescribed to deliver 2 Gy at the 100% isodose line at a depth of 3 cm? A. 0.75 Gy B. 1.0 Gy C. 1.5 Gy D. 2.0 Gy

answer is B. RATIONALE: By definition, the field size is defined as the lateral distance between the 50% isodose lines. Thus, at the field edge, the dose is 2 Gy x 0.5 = 1 Gy.

202. Which of the following characteristics best describes apoptotic cell death? A. Cleavage pattern of DNA that creates a smear on an electrophoretic gel B. Condensation and marginalization of nuclear chromatin C. Increased uptake of cytochrome C by the mitochondria D. Increased cytoplasmic volume

answer is B. RATIONALE: Cells undergoing apoptosis show condensation and marginalization of nuclear chromatin increased the release of cytochrome C from the mitochondria and decreased cytoplasmic volume. Double strand DNA breaks from cells undergoing apoptosis occur in linker regions between nucleosomes and result in distinct "ladders" on DNA gels, whereas cells dying from necrosis appear as "smears" on DNA gels.

159. Which of the following chromosomal translocations is most likely to be found in patients with Burkitt's lymphoma? A. t(2;5) B. t(8;14) C. t(11;14) D. t(14;18)

answer is B. RATIONALE: Chromosomal translocation t(8;14) is observed in Burkitt's lymphoma. Chromosomal translocation t(2;5) is seen with anaplastic large cell lymphoma, t(11;14) is seen in multiple myeloma, and t(14;18) is noted in follicular B cell lymphoma. REFERENCE: Tarbell, Weinstein. Non-Hodgkin lymphoma. Halperin, Constine, Tarbell, Kun (eds). Pediatric Radiation Oncology. 4th ed. pp 261-270. Exam answers and rationales 48 American College of Radiology

149. Which of the following histologic subtypes most commonly occurs in adults who have kidney cancer? A. Wilms tumor B. Clear cell carcinoma C. Papillary renal carcinoma D. Oncocytoma

answer is B. RATIONALE: Clear cell carcinoma is the most common subtype for adult renal cell carcinoma (RCC). Oncocytoma is considered to be nonmalignant. Exam answers and rationales Radiation Oncology In-Training Exam 2011 45

106. What cytogenetic finding is associated with oligodendroglioma? A. Loss of heterozygosity of 11q B. Co-deletion of 1p and 19q C. Deletion of 16p D. Deletion of 22p

answer is B. RATIONALE: Co-deletion of 1p and 19q has been found to be specific to oligodendroglioma and confers a better prognosis and increased sensitivity to both chemotherapy and radiation therapy. Exam answers and rationales 32 American College of Radiology

297. According to the Turrisi trial for SCLC, which of the following outcomes was associated with twice-daily administration of radiation and chemotherapy? A. The 5-year overall survival rate was 16%. B. The rate of grade 3 esophagitis was 27%. C. The rate of grade 3 pneumonitis was 40%. D. The objective response rate was 40%.

answer is B. RATIONALE: Compared to 45 Gy of radiation delivered in once-daily fractions, 45 Gy of radiation delivered in 1.5-Gy twice-daily fractions resulted in improved overall survival. Five-year overall survival was 26%, compared to 16% for the once-daily treatment arm. Although survival improved with twice-daily radiation therapy, it came at the cost of increased esophagitis (grade 3 esophagitis rate was 27% vs. 11% for once-daily radiation). Elective irradiation of the supraclavicular regions was specifically forbidden. Objective response rates approached 90% overall. REFERENCE: Turrisi AT, et al. New England Journal of Medicine (N Engl J Med). 1999 Jan 28;340(4):265-71. Exam answers and rationales 90 American College of Radiology

314. Which of the following radionuclides is NOT used currently for prostate brachytherapy? A. Ir-192 B. Cs-137 C. I-125 D. Pd-103

answer is B. RATIONALE: Cs-137 is not currently used for prostate brachytherapy. Ir-192 is used for HDR brachytherapy, and both I-125 and Pd-103 are used for LDR permanent brachytherapy.

118. What is the equivalent dose for an organ exposed to 10 Gy from a 1 MeV neutron beam with a radiation weighting factor of 20? A. 2,000 Sv B. 0,200 Sv C. 00,20 Sv D. 000,2 Sv

answer is B. RATIONALE: Equivalent dose (in Sv) = Absorbed dose (in Gy) x Radiation weighting factor......or 10 Gy x 20 = 200 Sv.

29. According to the 2009 ASTRO consensus statement, which of the following pathologic features is contraindicated for the use of accelerated partial breast irradiation outside of a prospective clinical trial for a patient with breast cancer? A. Focal lymphovascular invasion B. Positive axillary lymph nodes C. 1 cm extensive DCIS D. LCIS

answer is B. RATIONALE: Extensive ductal carcinoma in situ (DCIS) that is >3 cm in size, extensive lymphovascular invasion, and positive lymph nodes make patients "unsuitable" for accelerated partial breast irradiation (APBI) according to this consensus statement. Lobular carcinoma in situ (LCIS) did not exclude patients from being suitable for APBI. REFERENCES: Smith, et al. ASTRO consensus statement. International Journal of Radiation Oncology, Biology, Physics (Int J Radiat Oncol Biol Phys). 2009;74:987-1001. Journal of the American College of Surgeons (J Am Coll Surg). 2009;209:269-277.

70. Familial adenomatous polyposis is associated with: A. defects in mismatch repair. B. mutations in the APC gene. C. a late increased risk for colon cancer. D. the majority of colorectal cancers.

answer is B. RATIONALE: Familial adenomatous polyposis (FAP) is characterized by germline mutations in the APC gene. Defects in mismatch repair are characteristic of hereditary nonpolyposis colorectal cancer (HNPCC), not FAP. There is a 90% increased risk for colon cancer in the fifth decade of life. FAP only accounts for approximately 1% of colon cancers.

10. Which of the following occurs after an alpha decay? A. Atomic number, Z, increases by 2. B. Atomic number, Z, decreases by 2. C. Atomic mass, A, increases by 2. D. Atomic mass, A, decreases by 2.

answer is B. RATIONALE: Following an alpha decay, the atomic number, Z, decreases by 2, and the atomic mass, A, decreases by 4.

40. A patient has a grossly resected melanoma of the temporal scalp involving intraparotid lymph nodes. Adjuvant radiation therapy to the lymphatics has been shown to: A. increase overall survival. B. increase regional control. C. decrease distant metastasis. D. decrease local failure.

answer is B. RATIONALE: For patients with positive lymph nodes, radiation therapy improves regional control. REFERENCE: Burmeister B, et al. Adjuvant radiation therapy improves regional (lymph node field) control in patients with melanoma after lymphadenectomy: results of an intergroup randomized trial (RTOG 02.01/ANZMTG 01.02). ASTRO. 2009.

170. Which of the following is most characteristic of gallbladder cancer? A. Isolated locoregional failures are more common than in extrahepatic cholangiocarcinoma. B. Stage T2 disease involves invasion of the perimuscular connective tissue. C. Peritoneal seeding is increased in cancers on the hepatic parenchymal surface than on the serosal surface. D. It is associated with hepatitis B and C virus infection.

answer is B. RATIONALE: Gallbladder cancer is more likely to recur with distant failures and less likely to recur with isolated locoregional failures compared to extrahepatic cholangiocarcinoma. A study by Jarnagin, et al., found that of those patients who developed disease recurrence, isolated locoregional disease as the first site of failure occurred in 15% of patients with gallbladder cancer compared with 59% of patients with cholangiocarcinoma. According to the 2009 AJCC 7th edition staging system, stage T2 disease is defined by invasion into the perimuscular connective tissue without extension into the serosa or liver. REFERENCES: Jarnagin, et al. Cancer. 98(8):1689-700. AJCC Cancer Staging Manual. 7th ed. 2009. Exam answers and rationales Radiation Oncology In-Training Exam 2011 51

123. What is the most appropriate treatment for patients with Helicobacter pylori negative, stage IAE gastric MALT non-Hodgkin lymphoma? A. Total gastrectomy B. Definitive radiation therapy C. Multiagent chemotherapy D. Watchful waiting

answer is B. RATIONALE: Gastric MALT lymphomas are commonly associated with Helicobacter pylori infection, and if patients are positive for Helicobacter pylori infection, anti-Helicobacter pylori drugs are the treatment of choice. Untreated disease will eventually progress, and there is a chance of its transformation to a more aggressive type of disease. In patients who have persistent disease after Helicobacter pylori treatment or who are negative for Helicobacter pylori, radiation therapy is a very effective method for achieving a high rate of disease control. REFERENCE: Tsang RW, Gospodarowicz MK, Pintilie M, et al. Localized mucosa-associated lymphoid tissue lymphoma treated with radiation therapy has excellent clinical outcome. Journal of Clinical Oncology (J Clin Oncol). 2003;21:4157-4164.

184. Which of the following adjuvant radiation therapies would be best to administer postoperatively to patients with supratentorial primitive neuroectodermal tumor (S-PNET)? A. A total dose of 54 Gy to the postoperative tumor bed alone B. A total dose of 36 Gy to the neuro-axis and a boost of 18 Gy to the postoperative bed C. A total dose of 36 Gy to the whole cranium and a boost of 18 Gy to the postoperative bed D. A total dose of 18 Gy to the neuro-axis and a boost of 36 Gy to the postoperative bed

answer is B. RATIONALE: German brain tumor trials HIT 88/89 and 91 have shown that a significant decrease in survival was associated with modifications to radiation therapy, such as reducing the total dose to the craniospinal axis or tumor region, omitting craniospinal radiation, or restricting the fields to the tumor region only. REFERENCE: Timmermann B, et al. Journal of Clinical Oncology (J Clin Oncol). 2002;20:842-849. Exam answers and rationales Radiation Oncology In-Training Exam 2011 55

12. According to Patchell's study of patients with a solitary brain metastasis, what percentage of them were found to have an unexpected histology after resection or biopsy, followed by WBRT? A. 06% B. 11% C. 16% D. 21%

answer is B. RATIONALE: In Patchell's study, 6 of 54 eligible patients thought to have a solitary brain metastasis were found to have a second primary tumor or a nonmalignant pathology after resection or biopsy.

269. Which of the following treatments would be best for a patient with increasing T1 enhancement and T2 abnormality on a routine MRI 1 month after completion of concurrent temozolomide and 60 Gy of EBRT for glioblastoma? A. Bevacizumab and steroids B. Temozolomide and follow-up MRI C. PET scan to assess tumor metabolism D. Biopsy for further evaluation

answer is B. RATIONALE: Increased enhancement after chemoradiation is likely "pseudo-progression." The new RANO criteria address this phenomenon. Biopsy, steroids, or bevacizumab are not required. Most physicians would recommend commencing with adjuvant temozolomide. REFERENCE: Journal of Clinical Oncology (J Clin Oncol). 2010 Apr 10;28(11):1963-72. Exam answers and rationales 82 American College of Radiology

237. Compared to other locally advanced breast cancers, inflammatory cancer is more likely to demonstrate: A. low Ki-67 level. B. high histologic grade. C. HER2/neu overexpression. D. ER-positive status.

answer is B. RATIONALE: Inflammatory breast cancers are more likely to have a high S-phase (Ki-67 level), lack ER expression, and have high levels of p53 and epidermal growth factor. Most investigators have found that HER2/neu overexpression is no more common in patients with inflammatory disease than in those with locally advanced breast cancer. REFERENCE: Buchholz TA and Haffty BG. Breast cancer: locally advanced and recurrent disease, postmastectomy radiation, and systemic therapies. Halperin EC, Perez CA, Brady LW (eds). Principles and Practice of Radiation Oncology. 5th ed. Chapter 54. Philadelphia: Lippincott Williams & Wilkins Publishers. 2008; p 1296.

181. Lead shielding around a pregnant patient receiving upper thoracic radiation therapy will reduce all of the following contributions to fetal dose EXCEPT: A. head leakage. B. internal scatter. C. scatter from blocks. D. scatter from wedges.

answer is B. RATIONALE: Internal scatter is impossible to shield, as it originates from dose deposition inside the patient. The remaining items are all external to the patient; therefore, they may be attenuated by dense material.

213. What is the TNM (AJCC) stage of cystadenocarcinoma of both ovaries with a ruptured right ovarian capsule and malignant cells in the peritoneal washings? A. T1b (IB) B. T1c (IC) C. T2b (IIB) D. T2c (IIC)

answer is B. RATIONALE: Involvement of both ovaries is stage T1b, but the ruptured capsule and positive peritoneal washings makes the stage T1c (IC). Extension to other pelvic tissues (stage T2) is not suggested in this scenario, even though stage T2c (IIC) also requires positive cytology in the washings. REFERENCE: AJCC Cancer Staging Manual. 7th ed. 2009.

306. Approximately how many DNA double strand breaks are produced by 1 Gy of x-rays? A. 01 to 20 B. 21 to 40 C. 41 to 60 D. 61 to 80

answer is B. RATIONALE: It has been estimated that 1 Gy of x-rays produces between 25 and 40 DNA double strand breaks per exposed cell. REFERENCE: Hall and Giaccia. Radiobiology for the Radiologist. 6th ed. Chapter 2.

94. A 58-year-old man with a 100-pack-year smoking history presents with a 4 cm, right jugulodigastric lymph node. Biopsy of the lymph node reveals squamous cell carcinoma. What is the most likely primary tumor site? A. Nasal vestibule B. Base of tongue C. Pyriform sinus D. Glottic larynx

answer is B. RATIONALE: It is important to understand that the most common primary cancer site for a patient with a squamous cell carcinoma of unknown primary is the oropharynx.

54. Which of the following genetic alterations has been shown to be predictive of a negative response to the EGFR-inhibitor cetuximab in patients with metastatic colon cancer? A. PTEN deletion B. KRAS mutation C. MDM2 amplification D. TP53 loss of heterozygosity

answer is B. RATIONALE: KRAS mutations have been shown to be predictive of a lack of response to both cetuximab and panitumumab. Given that the benefit of these agents is restricted to KRAS wild-type patients, ASCO recommendations now reflect that patients should have KRAS testing prior to initiation of anti-EGFR therapies, and if they are KRAS-mutant, alternative therapies should be considered.

91. Which figure in the graph below demonstrates a typical contrast detail curve from a medical imaging system? A. Figure A B. Figure B C. Figure C D. Figure D

answer is B. RATIONALE: More contrast is needed for smaller objects, but can resolve smaller contrast for larger objects; hence, the curve has to start in the upper left corner and end in the lower right corner (Figure B or Figure D). Figure D is not a typical depiction of a contrast detail curve from a medical imaging system. Exam answers and rationales 28 American College of Radiology

260. A 23-year-old woman with nodular-sclerosing Hodgkin lymphoma has pathologically enlarged bilateral axillary, right infraclavicular, and left supraclavicular lymph nodes. She reports having no fever, night sweats, or unexplained weight loss. Which of the following Ann Arbor stages would be most appropriate for this patient? A. IA B. IIA C. IIIA D. IVA

answer is B. RATIONALE: Multiple nodal sites are considered to be stage II as long as they are on the same side of the diaphragm. Exam answers and rationales Radiation Oncology In-Training Exam 2011 79

258. What is the 5-year overall survival rate after bladder preservation therapy is performed on a patient with bladder cancer? A. 30% B. 50% C. 70% D. 90%

answer is B. RATIONALE: Multiple studies show that the 5-year overall survival rate following bladder preservation therapy for bladder cancer is approximately 50%. REFERENCES: Shipley, et al. RTOG 89-03. Journal of Clinical Oncology (JCO). 1998;16:3576- 3583. MGH series 1986-1993. Kachnic. Bladder preservation by combined modality therapy for invasive bladder cancer, Journal of Clinical Oncology (JCO). 1997;15:1022-1029.

244. A 25-year-old patient has received radiosurgery for a 1.5 cm vestibular schwannoma and is being followed for a 0.3 cm contralateral intracanalicular lesion. The patient's current symptoms include leg weakness and an enhancing intra-axial lesion at the T12 vertebral level. Which of the following diagnoses is most appropriate for the intra-axial lesion? A. Neurofibroma B. Ependymoma C. Low-grade astrocytoma D. Metastatic acoustic neuroma

answer is B. RATIONALE: Neurofibromatosis type 2 is characterized by bilateral vestibular schwannomas and associated with multiple meningiomas and ependymomas. The enhancing nature of the tumor at the T12 vertebral level suggests that it is not likely to be a low-grade astrocytoma. The intra-axial nature suggests it is not likely a neurofibroma. Exam answers and rationales 74 American College of Radiology

211. The use of WBRT for brain metastases secondary to breast cancer is most likely to result in: A. a median survival rate of 8 to 10 months. B. an overall complete or partial response rate of 90% to 95%. C. improvement of neurologic symptoms in 45% to 55% of affected patients. D. improvement of cranial nerve deficits in 90% of affected patients.

answer is B. RATIONALE: Nieder, et al., studied volumetric response rates of brain metastases to WBRT. The overall complete or partial response rate was 93%, and patients with breast cancer tended to have favorable response rates in relation to other histologies. Median expected survival after treatment for brain metastases is 4 to 6.5 months vs. 1 to 2 months with supportive care only. Radiation provides durable improvement and/or stability in ~70% to 90% Exam answers and rationales Radiation Oncology In-Training Exam 2011 63 of patients. For patients with cranial nerve deficits, approximately 40% may have improvement after WBXRT. MRI imaging should be done to confirm meningeal or skull base lesions so that these areas are included in the treatment field. REFERENCE: Lin NU, Ramakrishna NR. Brain metastases. Harris JR, Lippman ME, Morrow M, Osborne CK (eds). Diseases of the Breast. 4th ed. Chapter 81. Philadelphia: Lippincott Williams & Wilkins Publishers. 2010; p 982.

199. According to the 2010 NCCN guidelines, the most appropriate management for a 70-year-old woman with AJCC stage IC, grade 3, optimally surgically debulked epithelial ovarian cancer is: A. observation. B. IV paclitaxel and carboplatin given in 3 to 6 cycles. C. 198Au intraperitoneal radiocolloid therapy. D. whole-abdomen radiation therapy.

answer is B. RATIONALE: Observation can be a treatment option for patients with stage IA-IB, grade 1 or grade 2 tumors. An Italian study showed that delayed cisplatin use at the time of relapse had the same 5-year overall survival rate as immediate treatment. Patients with stage IC or grade 3 usually receive 3 to 6 cycles of taxane/carboplatin. Whole-abdomen radiation therapy is no longer included in the 2010 NCCN guidelines. 198Au intraperitoneal radiocolloid therapy is no longer available in North America as it possesses risks to other people in contact with the patient.

198. According to the SHARP trial, which outcome was significantly improved with sorafenib versus a placebo in patients with advanced hepatocellular carcinoma? A. Time to symptomatic progression B. Time to radiographic progression C. Grade 3 or greater GI toxicity D. Response rate

answer is B. RATIONALE: Overall survival and time to radiographic progression were both improved. However, time to symptomatic progression was not improved. Single-agent sorafenib was associated with a low (2.3%) response rate. Side effects include diarrhea and hand-foot syndrome. REFERENCE: New England Journal of Medicine (N Engl J Med). 2008;359:378-90. Exam answers and rationales Radiation Oncology In-Training Exam 2011 59

87. According to the PORTEC-2 (Nout, et al.) trial, what is the 5-year risk of pelvic recurrence in a patient who received vaginal brachytherapy for intermediate-risk to high-risk endometrial carcinoma? A. 4.8% B. 3.3% C. 2.4% D. 0.6%

answer is B. RATIONALE: PORTEC-2 randomized patients with intermediate- to high-risk endometrial carcinoma (age >60 years and 1988 FIGO stages IC, grade 1 or 2; stage IIA any age, except with grade 3 and >50% myometrial invasion) to receive pelvic irradiation or vaginal brachytherapy. The pelvic recurrence risk at 5 years was 0.6% after pelvic irradiation and 3.3% after vaginal brachytherapy. The vaginal recurrence risk was 1.9% and 1.5%, respectively. REFERENCE: Nout RA, et al. Lancet. 2010;375:816-23.

109. Which of the following treatments would be most appropriate for a 45-year-old woman with stage IIBX, nodular-sclerosing Hodgkin lymphoma? A. Two cycles of ABVD chemotherapy alone B. Four to six cycles of ABVD chemotherapy and involved-field radiation therapy C. High-dose chemotherapy with autologous stem cell transplantation D. Extended-field radiation therapy alone

answer is B. RATIONALE: Patients with early-stage, unfavorable Hodgkin lymphoma can achieve a high cure rate with either multiple cycles of ABVD-like chemotherapy or a combination of reduced cycles of ABVD (4-6) with involved-field radiation therapy. Extended-field radiation therapy is effective but is fairly toxic, particularly for long-term complication risk. High-dose chemotherapy with autologous stem cell transplantation is a very intense treatment that can cause significant toxicities and is only used in a setting of refractory or relapsed disease. REFERENCES: Meyer RM, Gospodarowicz MK, Connors JM, et al. Randomized comparison of ABVD chemotherapy with a strategy that includes radiation therapy in patients with limited-stage Hodgkin's lymphoma: National Cancer Institute of Canada Clinical Trials Group and the Eastern Cooperative Oncology Group. Journal of Clinical Oncology (J Clin Oncol). 2005;23(21):4634-4642. Straus DJ, Portlock CS, Qin J, et al. Results of a prospective randomized clinical trial of doxorubicin, bleomycin, vinblastine, and dacarbazine (ABVD) followed by radiation therapy (RT) versus ABVD alone for stages I, II, and IIIA nonbulky Hodgkin disease. Blood. 2004;104(12):3483-3489. Exam answers and rationales Radiation Oncology In-Training Exam 2011 33

14. A female urethral cancer that invades the periurethral muscle and involves a single 1.5 cm inguinal lymph node is classified as AJCC stage: A. T1N1. B. T2N1. C. T2N2. D. T3N1.

answer is B. RATIONALE: Periurethral muscle invasion is classified as stage T2, and a single node in the groin less than 2 cm is classified as stage N1. REFERENCE: AJCC Cancer Staging Manual. 7th ed. 2009.

245. According to the RTOG 89-03 study, approximately what proportion of patients treated with a bladder-preserving regimen of concurrent chemotherapy and radiation therapy will be alive with a functioning bladder in 5 years? A. 20% B. 35% C. 50% D. 65%

answer is B. RATIONALE: RTOG 89-03 was a phase III study designed to test the efficacy of 2 cycles of MCV (methotrexate, cisplatin, Vinblastine) chemotherapy after maximal transurethral resection of the bladder tumor and prior to a bladder-preserving regimen of cisplatin and radiation therapy. The study was closed after only 123 of a planned 174 patients were randomized, due to the high rate of severe neutropenia. There was no statistically significant difference in disease outcome between the two arms. For all patients, the 5-year rate of survival with a functioning bladder was 38%. The 5-year rate of overall survival was 49%. Pooling together results of other RTOG bladder trials, ~75% of patients who survive after bladder-preserving therapy for urothelial cancer are able to maintain functioning bladders. REFERENCES: Shipley, et al. Journal of Clinical Oncology (J Clin Oncol). 1998;16:3576-3583. Shipley, et al. Cancer. 2003;97:2115-2119.

158. A 63-year-old man has a 6.2 cm squamous cell carcinoma metastatic to the ipsilateral hilum without evidence of distant metastases. This patient's disease is classified as stage: A. IIA. B. IIB. C. IIIA. D. IIIB.

answer is B. RATIONALE: Refer to the new AJCC staging system introduced on January 1, 2010.

263. Which regional lymph node is most commonly involved in patients who had a chest wall recurrence of breast cancer after a mastectomy? A. Infraclavicular B. Supraclavicular C. Internal mammary D. Axillary

answer is B. RATIONALE: Regional recurrence in the axillary, supraclavicular, or internal mammary lymph nodes occurs in ~30% to 40% of patients who had a local recurrence after mastectomy. The supraclavicular lymph nodes are the most common site of regional recurrence (11% to 35% of patients). Five-year survival after supraclavicular fossa (SCF) lymph node recurrence is 16% to 28%. Axillary lymph node recurrence 5-year survival rates are 37% to 50%. REFERENCES: Stearns V, Davidson NE. Adjuvant systemic therapy: chemoendocrine. Harris JR, Lippman ME, Morrow M, Osborne CK (eds). Diseases of the Breast. 4th ed. Chapter 50. Philadelphia: Lippincott Williams & Wilkins Publishers. 2010; pp 646-647. Exam answers and rationales 80 American College of Radiology

321. Which of the following findings independently indicates the need for radiation therapy to a posterior axillary field after axillary surgery for breast cancer? A. The largest axillary lymph node is 2 cm. B. One of four dissected axillary lymph nodes is positive. C. Lymphovascular space invasion is present. D. Extracapsular extension is present.

answer is B. RATIONALE: Removal of less than six nodes during a level I/II dissection is a clear risk factor for increased recurrence in the axilla. Extracapsular extension (ECE) does not independently increase the risk of axillary recurrence. Lymphovascular invasion (LVI) and axillary nodal size also do not increase the risk of recurrence after mastectomy in the low-to-mid axilla. REFERENCES: Fisher, et al. Surgery, Gynecology, and Obstetrics. 1981;152:765-772. Gruber, et al. Annals of Oncology. 2008;19:1393-1401. Strom, et al. International Journal of Radiation Oncology, Biology, Physics (Int J Radiat Oncol Biol Phys). 2005;63:1508-1513. Mignano, et al. Cancer. 1999;86:1258-62. Hetelekidis, et al. International Journal of Radiation Oncology, Biology, Physics (Int J Radiat Oncol Biol Phys). 2000;46:31-34. Pierce, et al. International Journal of Radiation Oncology, Biology, Physics (Int J Radiat Oncol Biol Phys). 1995;33:253-259. Leonard, et al. Journal of Clinical Oncology (J Clin Oncol). 1995;13:47-53. Strom, et al. International Journal of Radiation Oncology, Biology, Physics (Int J Radiat Oncol Biol Phys). 2005;63:

51. Which method best describes a retrospective study designed to correlate vitamin B6 dietary intake with the development of lung cancer based on the health histories of patients with and without the disease? A. Cohort study B. Case-control study C. Hazard ratio analysis D. Relative risk ratio

answer is B. RATIONALE: Retrospective case-control studies are studies in which a group with a disease (case arm) is matched with a group without the disease (control arm) to examine if potential risk factors can be associated with subsequent disease development in the case arm. In contrast, retrospective cohort studies investigate two similar groups of patients who differ by a certain characteristic and compare the subsequent disease development between the two groups. Case-control studies begin with patients who have a disease to look for risk factors for disease development, whereas cohort studies begin with patients with suspected risk factors and assess if the factors are linked with disease development. A hazard ratio is performed to ascertain the effect of a certain variable on the risk (or "hazard") of an event occurring. It is commonly employed in drug studies to determine to what degree a drug can impact a disease outcome. (i.e., to what extent tamoxifen impacts disease-free survival compared to a placebo). A relative risk ratio describes the probability of an event occurring in an exposed group to an unexposed group (i.e., the probability of a person with adequate vitamin B6 levels developing lung cancer compared to those with inadequate dietary consumption of vitamin B6). While both a hazard ratio and a relative risk ratio can be performed as part of a cohort or case-control study, both of these represent statistical methods of data analysis rather than a type of study design. REFERENCES: McNutt L, et al. Estimating the relative risk in cohort studies and clinical trials of common outcomes. American Journal of Epidemiology (Am J Epidemiol). 2003;157:940-943. Spruance S, et al. Hazard ratio in clinical trials: antimicrobial agents and chemotherapy. 2004 Aug;48(8):2787-2792. Sistrom, et al. Proportions, odds, and risk. Radiology. 2004;230:12-19. National Cancer Institute, Dictionary of Cancer Terms, http://www.cancer.gov/dictionary. Exam answers and rationales 16 American College of Radiology

89. What percent of solitary plasmacytomas are medullary? A. 90% B. 70% C. 50% D. 10%

answer is B. RATIONALE: Roughly 70% of plasmacytomas are found to be medullary (bone) only disease. REFERENCE: Corwin J, Lindberg RD. Solitary plasmacytoma of bone vs. extramedullary plasmacytoma and their relationship to multiple myeloma. Cancer. 1979;43:1007-1013. Exam answers and rationales Radiation Oncology In-Training Exam 2011 27

48. A patient with a 15 cm Wilms tumor in the right kidney that extends into the renal vein, negative lymph nodes, and no tumor spill at the time of surgery has which of the following cancer stages? A. I B. II C. III D. IV

answer is B. RATIONALE: Stage I: A tumor confined to the kidney with an intact capsule and no prior biopsy. Stage II: Any tumor that is completely resected with negative margins but with extension beyond the kidney by regional extension of the tumor or of the blood vessels within the nephrectomy specimen outside the renal parenchyma. Lymph nodes are negative and there is not intra-operative spill. Stage III: Residual non-hematogenous tumor following surgery confined to the abdomen with any one of the following: Abdominal or pelvic lymph node involvement. Tumor penetration through the peritoneal surface. Peritoneal tumor implants. Incomplete resection because of tumor infiltration into vital structures. Tumor spillage before or during surgery. Tumor that is treated with pre-operative chemotherapy (with or without biopsy of any sort). Tumor removed in more than one piece. If the tumor thrombus in the renal vein is removed separately from the nephrectomy specimen. Extension of the primary tumor within vena cava into thoracic vena cava and heart (this is not stage IV). Stage IV: Distant metastasis outside of the abdomen or pelvis. Stage V: Synchronous bilateral tumors.

95. According to the ECOG 1484 (Horning, et al.) randomized trial, which of the following involved-field radiation doses was administered to patients who had non-Hodgkin lymphoma and a complete response to initial chemotherapy? A. 20 Gy B. 30 Gy C. 40 Gy D. 50 Gy

answer is B. RATIONALE: The ECOG 1484 trial randomized patients who had non- Hodgkin lymphoma and a complete response after 8 cycles of CHOP (cyclophosphamide, doxorubicin, vincristine, and prednisone) chemotherapy to either 30-Gy, involved-field radiation therapy or no further therapy. REFERENCE: Horning, et al. Journal of Clinical Oncology. 2004. Exam answers and rationales Radiation Oncology In-Training Exam 2011 29

110. Which of the following tumor kinetic parameters is correctly ordered from shortest to longest duration? (NOTE: Tc = cell cycle time; Td = tumor volume doubling time; TPOT = tumor potential doubling time; Ts = length of S phase) A. Ts < Tc < Td < TPOT B. Ts < Tc < TPOT < Td C. Ts < TPOT < Tc < Td D. TPOT < Ts < Tc < Td

answer is B. RATIONALE: The cell cycle kinetics of a tumor is an important consideration in how it grows and also how it responds to radiation therapy. The length of a particular cell cycle phase, such as the S phase (Ts), is necessarily shorter than the entire cell cycle time (Tc), which also includes the durations of G1, G2, and M phase. A tumor's potential doubling time (TPOT) takes into account both the cell cycle time of the individual tumor cells and the fact that typically not all cells in a particular tumor are actively proliferating. The fraction of proliferating cells in a tumor is termed the tumor's growth fraction (GF), for which values frequently fall between 30% and 50%. Finally, a tumor's overall volume doubling time (Td) takes into account both the TPOT and the fact that most tumors have high rates of cell loss (), the cell loss factor, with estimates as high as 90% for many carcinomas. REFERENCE: Hall and Giaccia. Radiobiology for the Radiologist. 6th ed. Chapter 21.

254. What was the cumulative 10-year incidence of secondary neoplasms in patients, according to the Children's Cancer Group (CCG) acute lymphoblastic leukemia trials? A. 1% B. 1% to 3% C. 4% to 9% D. 10%

answer is B. RATIONALE: The cohort of over 8,000 patients from the Children's Cancer Group (CCG) studies showed an incidence of 1.3%. A large German study showed the incidence to be <3%.

32. What is the most appropriate EBRT regimen for a 15-year-old patient with an unresectable Ewing's sarcoma of the pelvis after 12 weeks of systemic chemotherapy? A. 36 Gy to the initial tumor volume with a 9 Gy boost to the post-chemotherapy volume B. 45 Gy to the initial tumor volume with a 10.8 Gy boost to the post-chemotherapy volume C. 60 Gy to the initial tumor volume with a 6 Gy boost to the post-chemotherapy volume D. 60 Gy to the initial tumor volume with no boost

answer is B. RATIONALE: The current recommended dose is 55.8 Gy given in 31 fractions. The initial tumor volume encompasses the pre-chemotherapy volume with consideration of anatomic boundaries and boost of 10.8 Gy is given to the post-chemotherapy volume with consideration of normal tissue toxicities. Exam answers and rationales 10 American College of Radiology

299. What would be the equivalent dose in 2 Gy fractions for a patient originally scheduled to receive 10 daily fractions of 3 Gy to the spinal cord for palliation of pain associated with bony metastases? (Assume / = 2 Gy for the spinal cord.) A. 30.0 Gy B. 37.5 Gy C. 40.0 Gy D. 47.5 Gy

answer is B. RATIONALE: The equivalent dose in 2 Gy fractions = EQD2. EQD2 = D • [ d + / 2 Gy +], where D = total dose for original fractionation scheme, d = dose per fraction for original fractionation scheme, and the ratio for tissue at risk (spinal cord) = 2 Gy.

18. Which of the following factors most significantly increases the risk for the development of esophageal adenocarcinoma? A. Female gender B. Barrett's esophagus C. Alcohol consumption D. Cigarette smoking

answer is B. RATIONALE: The incidence of gastroesophageal junction adenocarcinoma is increasing in the United States, especially among white males and females. Barrett's esophagus is one of the most important risk factors for developing esophageal adenocarcinoma. Both smoking and alcohol are risk factors for developing esophageal squamous cell carcinoma. Exam answers and rationales 6 American College of Radiology

240. After initial diagnosis, patients with malignant mesothelioma have a median survival time of: A. 06 months. B. 12 months. C. 18 months. D. 24 months.

answer is B. RATIONALE: The median survival of patients with malignant mesothelioma from the time of diagnosis is 12 months. The prognosis is worse in male patients and in patients with extensive disease, poor performance status, elevated white blood cell counts, anemia, thrombocytosis, sarcomatoid histologic findings, or high standardized uptake value ratios on PET scans. A worse prognosis is also associated with expression of certain biochemical markers (cyclooxygenase-2 and VEGF), hypermethylation of the P16INK4a gene, increased vascularity, and evidence of SV40 virus in the tumor.

37. According to Early Breast Cancer Trialists' Collaborative Group (2005) findings, the addition of radiation therapy to breast-conserving surgery reduces the 15-year breast cancer mortality rate by: A. 02%. B. 05%. C. 10%. D. 25%.

answer is B. RATIONALE: The meta-analysis of trials of radiation therapy after breast-conserving surgery by the Early Breast Cancer Trialists' Collaborative Group showed a reduction in breast cancer mortality from 35.9% to 30.5% with the addition of breast irradiation. REFERENCE: Early Breast Cancer Trialists' Collaborative Group (EBCTCG). Lancet. 2005;366:2087-2106.

218. Which of the following types of ependymoma has the best prognosis? A. Thoracic spinal cord anaplastic B. Filum terminale myxopapillary C. Supratentorial clear cell D. Fourth ventricular

answer is B. RATIONALE: The myxopapillary subtype of ependymoma is a WHO grade I neoplasm. The sacral location is generally more likely to be surgically resectable.

105. Increasing an x-ray tube current will: A. produce more penetrating x-rays. B. increase the amount of x-rays produced. C. increase the maximum energy of the x-rays produced. D. decrease the amount of heat generated in the x-ray tube.

answer is B. RATIONALE: The number of photons produced increases with the tube current, as does the heat generated in the target; whereas increasing the tube voltage increases the average and maximum energy of x-ray production and the penetration of the x-rays.

290. A 62-year-old woman is diagnosed with SCLC. Diagnostic CT and PET scans show no evidence of metastatic disease. What should be done next? A. Obtain a bone marrow biopsy. B. Obtain an MRI of the brain. C. Sample the mediastinal lymph nodes. D. Administer concurrent chemoradiation.

answer is B. RATIONALE: The overall incidence of asymptomatic brain metastases at presentation for all patients with small cell lung cancer (SCLC) ranges from 10% to 20%. Brain imaging is a critical component of staging for all patients with SCLC, and must be obtained before proceeding with definitive chemotherapy and chest irradiation. REFERENCE: Hochstenbag, et al. Journal of Neuro-Oncology. 2000 July;48(3). Exam answers and rationales 88 American College of Radiology

189. A Lucite screen is placed in front of patients during total body irradiation procedures to: A. improve the homogeneity of the treatment beam. B. increase the superficial dose delivered to the patient. C. decrease the amount of low-energy electrons reaching the patient. D. compensate for irregularities in the patient's topology.

answer is B. RATIONALE: The purpose of the Lucite screen/spoiler is to produce low-energy scatter, which increases the superficial dose delivered to the patient.

81. According to the RTOG 97-14 trial, which retreatment finding was associated with the use of a single fraction of 8 Gy versus 30 Gy/10 fractions for palliation of painful breast and prostate bone metastases? A. Married women were less likely than single women to receive retreatment in both radiation treatment groups. B. Single men were less likely than married men to receive retreatment in the single-dose treatment group. C. The retreatment rate was the same for both radiation treatment groups. D. The retreatment rate was lower in the single-dose treatment group.

answer is B. RATIONALE: The rate of retreatment was higher in the 8 Gy radiation treatment group than in the 30 Gy radiation treatment group (18% vs. 9%). This disparity in the rate of treatment occurred despite nearly identical rates of stable (26% vs 24%) or progressive pain scores (9% vs. 10%) and similar rates of narcotic use between both groups. The data further suggested that subgroups may benefit from the longer course of radiation therapy. Married men and single or married women were more likely to receive retreatment after receiving 8 Gy vs. 30 Gy. There was no difference in the retreatment rate of single men. The authors suggest that social support factors may significantly impact the ability of some patients to access repeat therapies for painful bony metastases and that such individuals may benefit from the longer course of radiation therapy. REFERENCES: Hartsell, WF, Scott CB, Bruner, DW, et al. Randomized trial of short versus long course radiotherapy for palliation of painful bone metastases. Journal of National Cancer Institute. 2005;97:798-804. Konski A, Desilvio M, Harsell W, et al. Continuing evidence for poorer treatment outcomes for single male patients: retreatment data from RTOG 97-14. International Journal of Radiation Oncology Biology and Physics (IJROBP). 2006;66:229-233.

252. The RTOG 92-10 trial showed which of the following outcomes in patients with cervical cancer and positive paraaortic lymph nodes after administration of 1.2 Gy twice-daily radiation to the pelvis and paraaortic lymph nodes with concurrent chemotherapy? A. The estimated overall survival rate was 50% at 4 years. B. There was unacceptably high grade 4, late toxicity at 3 years. C. The probability of disease failure at any site was 60% at 1 year. D. The probability of locoregional failure was 50% at 1 year.

answer is B. RATIONALE: The results of the trial demonstrated unacceptably high grade 4, late toxicity at 3 years (17%, 5/29 patients). REFERENCE: International Journal of Radiation Oncology, Biology, Physics (Int J Radiat Oncol Biol Phys). 2001;51(4):982-987.

126. A receiver operating characteristic curve (ROC) evaluates the performance of a diagnostic test by graphing: A. the sensitivity against the specificity. B. the true-positive rate against the false-positive rate. C. the true-negative rate against the false-negative rate. D. 1 minus the sensitivity against the specificity.

answer is B. RATIONALE: This is the definition of an ROC curve. Note that an equivalent definition is that the ROC curve graphs the sensitivity against 1 minus the specificity.

191. The most common histology associated with ureteral malignancies in the United States is: A. adenocarcinoma. B. transitional cell carcinoma. C. squamous cell carcinoma. D. small cell carcinoma.

answer is B. RATIONALE: Transitional cell carcinoma is the most common cell type in ureteral malignancies.

56. Superficial voltage radiation therapy is preferable to electron radiation therapy for basal cell carcinoma of the lower eyelid due to its: A. longer SSD. B. tighter margins. C. lower oxygen enhancement ratio. D. higher relative biological effectiveness.

answer is B. RATIONALE: Unlike electron radiation therapy, superficial voltage radiation therapy does not have a bowing of the prescription isodose curves, so tighter margins may be used.

45. What is the calculated risk of pelvic lymph node involvement in a patient who has prostate cancer with a Gleason score of 4+4 and PSA level of 18 ng/mL? A. 21% B. 32% C. 47% D. 75%

answer is B. RATIONALE: Using the Roach formula, the risk of lymph node metastases is as follows: (%) = 2/3 PSA + 10 x (Gleason - 6).

307. Which of the following parameters will be constant for all barriers in a linear accelerator vault when secondary barrier calculations are performed for total body irradiation? A. Maximum permissible dose (P) B. Use factor (U) C. Occupancy (T) D. Distance (d)

answer is B. RATIONALE: When secondary barrier calculations are performed, the use factor is equal to one for all barriers. The use factor is defined as the fraction of time the x-ray falls on a given barrier. Scatter and leakage radiation have an equal probability of striking any barrier in a treatment vault, hence U = 1 for all barriers. According to NCRP 151, the shielding design goal, or maximum permissible dose, will depend on whether the area behind the barrier is controlled (P = 0.1 mSv/wk) or uncontrolled (P = 0.02 mSv/wk).

264. What is the stage for a patient who has rectal adenocarcinoma with perirectal fat invasion, two positive perirectal lymph nodes, and no distant metastases? A. T2N1M0 B. T3N1M0 C. T3N2M0 D. T4N2M0

answer is B. REFERENCE: AJCC Cancer Staging Manual. 7th ed. 2009.

222. According to Emami, et al., what total x-ray dose delivered with conventional fractionation to the entire brachial plexus will yield a 5% risk of complications within 5 years of treatment? A. 50 Gy B. 60 Gy C. 70 Gy D. 80 Gy

answer is B. REFERENCE: Emami, et al. International Journal of Radiation Oncology, Biology, Physics (Int J of Radiat Oncol Biol Phys). 1991;21:109-122.

288. In the Intergroup randomized (Macdonald, et al.) trial, patients with gastric cancer received postoperative radiation therapy administered concurrently with: A. 5-FU and cisplatin. B. 5-FU and leucovorin. C. 5-FU protracted venous infusion. D. ECF chemotherapy.

answer is B. REFERENCE: Macdonald, et al. Landmark trial on postoperative chemoradiation for gastric cancer. New England Journal of Medicine (N Engl J Med). 2001.

295. In the Intergroup randomized (Macdonald, et al.) trial, what postoperative radiation dose was administered concurrently with chemotherapy for patients with gastric cancer? A. 30.6 Gy B. 45.0 Gy C. 50.4 Gy D. 54.0 Gy

answer is B. REFERENCE: Macdonald, et al. Landmark trial on postoperative chemoradiation for gastric cancer. New England Journal of Medicine (N Engl J Med). 2001.

162. What is the most appropriate radiation dose for patients with Graves' ophthalmopathy? A. 10 Gy in 5 fractions B. 20 Gy in 10 fractions C. 30 Gy in 15 fractions D. 40 Gy in 20 fractions

answer is B. REFERENCE: Marquez SD. Long-term results of irradiation for patients with progressive Graves' ophthalmopathy. International Journal of Radiation Oncology, Biology, Physics (Int J Radiat Oncol Biol Phys). 2001 Nov 1;51(3):766-74.

275. Involvement of the infraclavicular lymph nodes in patients with breast cancer is classified as clinical stage: A. N2a. B. N2b. C. N3a. D. N3c.

answer is C. RATIONALE: N2a = Involved axillary lymph nodes fixed to one another or to adjacent structures N2b = Involved internal mammary lymph nodes without concurrent axillary disease N3a = Involvement of the infraclavicular region N3c = Involvement of the ipsilateral supraclavicular region REFERENCE: Singletary SE, Greene FL: Revision of breast cancer staging. The 6th edition of the TNM Classification. Seminars in Surgical Oncology. 2003;21:53-59.

8. Which study design is used to compare the results of patients taking a new treatment versus those taking the standard treatment? A. Phase I B. Phase II C. Phase III D. Phase IV

answer is C. RATIONALE: A Phase I trial is used to determine the best way to give a new treatment and the best dose. A Phase II study tests whether a new treatment has an anticancer effect and if it works against a certain cancer. A Phase IV study is used when a treatment has been approved and is marketed, but it evaluates side effects not apparent in a Phase III trial.

30. Carcinoma of the urethra that involves the anterior vagina is classified as AJCC stage: A. T1. B. T2. C. T3. D. T4.

answer is C. RATIONALE: A T3 lesion involves the anterior vagina or the bladder neck in a female patient. REFERENCE: AJCC Cancer Staging Manual. 7th ed. 2009.

265. Based on the updated TNM staging system, what clinical tumor stage should be assigned to a 5 cm squamous cell carcinoma of the cervix that extends to the proximal one third of the vagina only? A. T1b2 B. T2a1 C. T2a2 D. T3a

answer is C. RATIONALE: A clinically visible cervical cancer that is more than 4 cm in greatest dimension with extension to the proximal two thirds of the vagina without parametrial involvement is classified as stage T2a2 (by TNM category), based on the AJCC Cancer Staging Manual. 7th ed. 2009.

223. A pair of tangent beams is used for the treatment of breast cancer in a woman with a pacemaker. Since the pacemaker is not in the direct treatment beam, what is the recommended dose limit to the pacemaker? A. No dose limit required B. A fractional dose limit of 2 Gy C. A total accumulated dose limit of 2 Gy D. A total accumulated dose limit of 20 Gy

answer is C. RATIONALE: According to AAPM TG-34, the total dose to the pacemaker should be limited to 2 Gy (i.e., should not exceed 2 Gy). Exam answers and rationales Radiation Oncology In-Training Exam 2011 67

104. The excess absolute risk per Gy of childhood cancers following in utero exposure to diagnostic x-rays is approximately: A. 00.1%. B. 00.6%. C. 06%. D. 10%.

answer is C. RATIONALE: According to Doll and Wakeford, the best estimate for the excess absolute risk coefficient is approximately 6.0% per Gy. REFERENCES: Doll and Wakeford. British Journal of Radiology (Br. J. Radiol). 1997;70:130-139. Hall and Giaccia. Radiobiology for the Radiologist. 6th ed. Chapter 12.

267. Which of the following types of abdominal tumor most commonly occurs during adolescence? A. Neuroblastoma B. Rhabdomyosarcoma C. Non-Hodgkin lymphoma D. Wilms tumor

answer is C. RATIONALE: According to SEER 2002 data, non-Hodgkin lymphoma is very uncommon in children under 3 years of age. The incidence increases in adolescence. The incidence of neuroblastoma, rhabdomyosarcoma, and Wilms tumor is highest in patients under 10 years of age and is very uncommon in the adolescent age group.

17. For a typical shipment of radioactive I-125 seeds for a permanent prostate implant, which of the following parameters is required to be on the hazard label affixed to a shipping container of radioactive material? A. Assay date B. Patient identifier C. Hazardous material class D. Red biohazard background

answer is C. RATIONALE: According to Title 49 of the Code of Federal Regulations Part 172.403, the activity in Becquerel, hazardous material class, and content (type of radioactive material) are required to be on the hazard label affixed to a shipping container of radioactive material.

248. Which statement about early and late normal tissue complications after radiation therapy is true? A. The kidney expresses radiation damage within weeks of treatment. B. The spinal cord expresses radiation damage within 3 months of treatment. C. Radiation pneumonitis occurs 1 to 6 months after irradiation. D. Bone marrow expresses radiation damage 6 or more months after treatment.

answer is C. RATIONALE: Acutely responding normal tissues express radiation damage during or shortly after treatment, whereas slowly or non-proliferating tissues express radiation damage several months or years after treatment. The latency period for manifestation of the damage is not a function of the underlying radiosensitivity of critical cells in the tissue, but rather of the tissue's proliferation kinetics. For example, the bone marrow is a very radiosensitive tissue and typically expresses its injury within weeks of the start of treatment. The kidney is also a relatively radiosensitive organ, but expresses its damage much later, generally 6-12 months after irradiation. Depending on the volume irradiated, the lung can be considered either radiosensitive (large volumes) or relatively resistant (small volumes) with respect to radiation pneumonitis, which tends to occur 1-6 months after irradiation. The spinal cord, on the other hand, is a more radioresistant organ and expresses its damage even later, typically a year or more after the completion of radiation therapy.

107. A 64-year-old man has a positive surgical margin after prostatectomy for a stage T3A, Gleason score 4+3 prostate adenocarcinoma. Which of the following endpoints will be improved by adjuvant radiation therapy? A. PSA doubling time B. Erectile function C. Overall survival D. Rectal stricture

answer is C. RATIONALE: Adjuvant radiation therapy has been shown in the SWOG randomized trial to improve biochemical failure-free survival, distant metastasis-free survival, and overall survival.

284. Which of the following factors is associated with a poor outcome in patients with acute lymphoblastic leukemia? A. Age, LDH, peripheral WBC counts at diagnosis, and CNS status B. Age, LDH, performance status, and rapidity of response to initial chemotherapy C. Age, WBC count, histologic subtype, cytogenetic test results, and response to chemotherapy D. Performance status, peripheral WBC count, fever, and rapidity of response to initial chemotherapy

answer is C. RATIONALE: Age, WBC count, acute lymphoblastic leukemia (ALL) subtype, cytogenetic test results, and response to chemotherapy are known to have the most significant impact on the outcome of patients diagnosed with ALL. REFERENCE: DeVita, et al. Principles and Practice of Oncology.

101. Which type of cancer is most likely to be associated with a 65-year-old woman who presents with postmenopausal bleeding? A. Vaginal B. Cervical C. Endometrial D. Vulvar

answer is C. RATIONALE: Although all of these conditions can lead to vaginal bleeding in a postmenopausal woman, endometrial cancer is the most common female genital tract malignancy in the United States. There were an estimated 40,000 new cases in 2008 compared to 11,000, 3,500, and 2,000 cases for cervical, vulvar, and vaginal cancers, respectively. REFERENCE: Jemal A, et al. CA Cancer J Clin. 2009;59:225-49.

122. An otherwise healthy 60-year-old patient has a 5 cm squamous cell carcinoma of the base of tongue with bilateral jugulodigastric lymph node involvement. Which of the following systemic therapy regimens combined with radiation therapy is most appropriate for this patient? A. Cisplatin at 30 mg/m2 weekly B. Cetuximab at 250 mg/m2 weekly C. Cisplatin at 100 mg/m2 every 3 weeks D. Carboplatin at 100 mg/m2 and paclitaxel at 20 mg/m2 every 3 weeks

answer is C. RATIONALE: Although carboplatin/Taxol is a commonly used chemocombination, it has not been proven in a prospective trial to be better than radiation therapy alone or standard cisplatin therapy. Cetuximab can be used in lieu of cisplatin for elderly patients or patients with a poor performance status; a loading dose is needed before radiation. Exam answers and rationales Radiation Oncology In-Training Exam 2011 37

236. In electron beam therapy, an increase in the incident beam obliquity (relative to an angle perpendicular to the skin surface) will: A. increase the virtual source position. B. shift d80 away from the skin surface. C. shift dmax toward the skin surface. D. decrease the dose at dmax.

answer is C. RATIONALE: An increase in the angle of incidence for an electron beam will result in the point closer to the surface receiving greater side scatter from adjacent electrons that transverse a greater amount of material, while the points at deeper depths receive less scatter (i.e., d10 will shift away from the surface). REFERENCE: Khan FM, Deibel FC, Soleimani-Meigooni A. Medical Physics (Med Phys). 1985;12(6):749-753.

86. A 4 cm anal canal tumor with no lymphadenopathy should be classified as stage: A. T4N0. B. T3N0. C. T2N0. D. T1N0.

answer is C. RATIONALE: Anal canal tumors that are >2 cm but 5 cm are classified as stage T2.

168. What is the 95% confidence interval if the true-positive rate of a diagnostic test is 0.8 and the standard error is 0.04? A. 0.68 to 0.92 B. 0.70 to 0.80 C. 0.72 to 0.88 D. 0.74 to 0.86

answer is C. RATIONALE: Approximate 95% confidence bounds are computed as the value of interest +2 times the standard error.

319. Which of the following tumors is characterized by a deletion of the INI-1 (SMARCB1) tumor suppressor gene on chromosome 22? A. Choroid plexus carcinoma B. Dysembryoplastic neuroectodermal tumor C. Atypical teratoid rhabdoid tumor D. Pleomorphic xanthoastrocytoma

answer is C. RATIONALE: Atypical teratoid rhabdoid tumor (ATRT) is an uncommon aggressive malignancy with a poor prognosis that is characterized by the absence of INI-1 (SMARCB1) tumor suppressor gene. It most commonly occurs in the posterior fossa, but it can occur anywhere in the neural axis. Exam answers and rationales 96 American College of Radiology

311. According to Aupérin's meta-analysis, which outcome was associated with the use of PCI for patients who had limited-stage SCLC and achieved a complete response? A. The 3-year overall survival rate was improved by 10%. B. A clear dose response in overall survival was demonstrated. C. The risk of CNS metastasis was reduced by 25% at 3 years. D. The risk of neuropsychological sequelae was increased.

answer is C. RATIONALE: Aupérin's meta-analysis looked at individual data from 987 patients from 7 randomized trials comparing PCI to no PCI. The absolute improvement in 3-year overall survival was approximately 5% (from 15 to 21%). While larger total doses of radiation appeared to improve local control, there was no improvement in overall survival. The brain (CNS) failure rate was reduced from 58.6% to 33.3%, a 25.3% risk reduction. The effect of PCI on neuropsychological sequelae could not be addressed by this meta-analysis. REFERENCE: Aupérin A. New England Journal of Medicine (N Engl J Med). 1999 Aug 12;341(7):476-8. Exam answers and rationales 94 American College of Radiology

304. Which of the following chemotherapy regimens is most commonly delivered along with chest irradiation for patients with limited-stage SCLC? A. Carboplatin and etoposide weekly B. Carboplatin and paclitaxel weekly C. Cisplatin and etoposide every 3 weeks D. Cisplatin and vinblastine every 3 weeks

answer is C. RATIONALE: Based on the results of INT-0096, cisplatin and etoposide given every 3 weeks for 4 total cycles (cycles 1 and 2 delivered at the start, and immediately following the completion of radiation) has become the most commonly used regimen. As such, it is listed as a category 1 recommendation by the NCCN guidelines for small cell lung cancer (SCLC). REFERENCE: Turrisi AT, et al. New England Journal of Medicine (N Engl J Med). 1999 Jan 28;340(4):265-71.

3. Camptothecins belong to which chemical class? A. Antibiotics B. Antimetabolites C. Topoisomerase I inhibitors D. Hypoxic cell cytotoxins

answer is C. RATIONALE: Camptothecin and its derivatives target DNA topoisomerase I. In the presence of camptothecin, a camptothecin/topoisomerase I/DNA complex is stabilized and interacts with the advancing replication fork. This, in turn, causes the conversion of single strand breaks in DNA into irreversible, irreparable double strand breaks, leading to cell death.

64. Clear cell sarcoma of the kidney is most likely to metastasize to the: A. lung. B. brain. C. bone. D. other kidney.

answer is C. RATIONALE: Clear cell sarcoma of the kidney has a high frequency of metastasis to the bone; therefore, a bone scan is recommended for staging. Wilms tumor has a higher chance of metastasis to the lung; rhabdoid tumor is associated with brain tumors.

182. Which statistical method is most appropriate to calculate time to recurrence of cancer in two groups of patients after adjusting for age and sex? A. Logrank test B. Generalized Wilcoxon test C. Cox proportional hazards regression D. Logistic regression

answer is C. RATIONALE: Cox regression analysis is designed to deal with settings in which the outcome measure is dichotomous and time dependent.

85. According to NCCN guidelines, which of the following findings justifies whole-breast irradiation after breast-conserving surgery in a 72-year-old woman with 2 cm breast cancer, negative surgical margins, no axillary lymph node sampling, and an ER/PR-negative status? A. Age of 72 years B. Tumor size of 2 cm C. ER/PR-negative status D. No axillary lymph node sampling

answer is C. RATIONALE: Current National Comprehensive Cancer Network (NCCN) guidelines for breast cancer indicate that breast irradiation may be omitted in patients who are 70 years of age or older with an estrogen receptor positive (ER-positive) status, clinically negative lymph nodes, stage T1 tumors and who are receiving adjuvant endocrine therapy. This is heavily based upon the results of a randomized trial by the CALGB that reported a small difference in local recurrence with and without radiation (1% versus 7%) in these patients with clinical stage I breast cancer, and axillary dissection was not required for eligibility or performed in most patients. Exam answers and rationales 26 American College of Radiology

302. D2 lymphadenectomy for gastric cancer does NOT include dissection of the: A. celiac axis lymph nodes. B. perigastric lymph nodes. C. paraaortic lymph nodes. D. lymph nodes along the splenic vessels.

answer is C. RATIONALE: D2 lymphadenectomy for gastric cancer includes dissection of perigastric lymph nodes and nodes along the celiac axis, left gastric, hepatic, and splenic vessels.

291. Which of the following conditions is the most common CNS manifestation in children with Langerhans cell histiocytosis? A. Hydrocephalus B. Parinaud syndrome C. Diabetes insipidus D. Opsoclonus myoclonus

answer is C. RATIONALE: Diabetes insipidus occurs in 10% to 15% of children with Langerhans cell histiocytosis. REFERENCE: Ghirardello S, Garrè ML, Rossi A, Maghnie M. The diagnosis of children with central diabetes insipidus. Journal of Pediatric Endocrinology & Metabolism (J Pediatr Endocrinol Metab). 2007 March;20(3):359-75.

203. Ultrasound-guided, high-intensity focused ultrasound (HIFU) is being used in current U.S. clinical trials to treat patients with: A. skin cancer. B. lung cancer. C. prostate cancer. D. brain tumors.

answer is C. RATIONALE: Due to large differences in acoustic impedance, lesions adjacent to or within air or bone cannot be effectively imaged with ultrasound. Thus, the prostate is the only site listed that is suitable for ultrasound-guided HIFU treatments. REFERENCE: Dogra V, Zhang M, and Shweta Bhatt. High-intensity focused ultrasound (HIFU) therapy applications. Ultrasound Clinics. 2009;4(3):307-321.

163. Everolimus (Afinitor), a targeted therapy recently approved by the FDA for the treatment of metastatic renal cell carcinoma, inhibits: A. VEGF. B. EGFR. C. mTOR. D. HER2/neu.

answer is C. RATIONALE: Everolimus is an mTOR (mammalian target of rapamycin) inhibitor which has been shown to be effective for metastatic renal cell carcinoma. Exam answers and rationales Radiation Oncology In-Training Exam 2011 49

77. Expectant management (active surveillance) is a valid alternative for patients who have prostate cancer with: A. a PSA density of 0.5 ng/mL/g. B. a Gleason score of 7. C. a PSA level of 7 ng/mL. D. 6 positive biopsy cores.

answer is C. RATIONALE: Expectant management is a valid treatment option for patients with very low-risk prostate cancer (stage T1a, Gleason score of <7, PSA of <10 ng/mL, <3 positive biopsy cores, <50% cancer per core, and PSA density of <0.15 ng/mL/g). Exam answers and rationales Radiation Oncology In-Training Exam 2011 23

212. Which of the following factors is included in the Cancer of the Liver Italian Program (CLIP) score for predicting survival in patients with hepatocellular carcinoma without extrahepatic metastases? A. CEA level B. Weight loss C. Portal vein thrombosis D. Tumor grade

answer is C. RATIONALE: Factors not used in determining the CLIP score include CEA level, weight loss, or tumor grade. The CLIP score is important for radiation oncologists in comparing results of liver-directed therapy. The following factors (illustrated in the table below) are used to determine the CLIP score. Child-Pugh stage A 0 B 1 C 2 Tumor morphology Uninodular and extension ≤50% 0 Multinodular and extension ≤50% 1 Massive or extension >50% 2 Alpha-fetoprotein <400 0 ≥400 1 Portal vein thrombosis No 0 Yes 1 Exam answers and rationales 64 American College of Radiology

96. Which of the following transcription factors is stabilized under hypoxic conditions and helps coordinate the hypoxic stress response in cells? A. c-Myc B. NF-B C. HIF-1 D. AP-1

answer is C. RATIONALE: HIF-1 (hypoxia induced factor-1) is stabilized only under hypoxic conditions and transactivates many downstream target genes to help regulate cellular adaptation to hypoxic conditions. REFERENCE: Semenza GL. Defining the role of hypoxia-inducible factor 1 in cancer biology and therapeutics. Oncogene. 2010;29:625-634.

61. Which of the following treatment approaches would be most appropriate for patients who have prostate cancer with rectal distention on a treatment-planning CT scan? A. No prolonged overall treatment time B. Hormonal therapy to decrease prostate size C. Image-guided radiation therapy D. Hypofractionation radiation therapy

answer is C. RATIONALE: Image-guided radiation therapy (IGRT) offers solutions to optimize the treatment geographically and hopefully improve local control for patients with prostate cancer.

34. Which of the following processes produces clinical electrons in a medical linear accelerator? A. Bremsstrahlung B. Compton scattering C. Thermionic emission D. Auger effect

answer is C. RATIONALE: In a medical linear accelerator, electrons are produced by an electron gun via the process of thermionic emission. During this process, the filament in the electron gun is heated until electrons are "boiled off."

190. What was the 1-year local control rate after administration of individualized stereotactic body radiation therapy for liver metastases with no size limits, according to a multi-institutional phase I study? A. 29% B. 52% C. 71% D. 87%

answer is C. RATIONALE: In this multi-institutional study of individualized stereotactic body radiation therapy (SBRT) that used dosing based on liver metastases, the 1-year local control rate was estimated to be 71%. It's important to note that there was no size restriction in this study. REFERENCE: Lee MT, et al. Journal of Clinical Oncology (J Clin Oncol). 2009;27:1585-1591.

27. What histopathologic criterion is required for the diagnosis of anaplastic (malignant) meningioma, according to the WHO 2000 classification? A. Invasion into the brain parenchyma B. MIB-1 labeling index of 20% by immunohistochemistry C. More than 20 mitotic figures per 10 high-power field D. More than 30 mitotic figures per 10 high-power field

answer is C. RATIONALE: Invasion into the brain parenchyma is no longer pathognomonic of an anaplastic (malignant) classification. The MIB labeling index is not used for the WHO classification. Current histopathologic criteria require more than 20 mitotic figures per 10 high-power field (HPF) for the diagnosis of anaplastic (malignant) meningioma.

16. A 68-year-old smoker has a 2 cm squamous cell carcinoma of the tonsil with no clinical evidence of lymph node involvement. Which of the following treatments would be most appropriate? A. Concurrent chemotherapy and radiation therapy B. Induction chemotherapy, followed by concurrent chemoradiation therapy C. Radiation therapy to the oropharynx and ipsilateral cervical lymph nodes D. A radical tonsillectomy and bilateral lymph node dissection

answer is C. RATIONALE: It is important to understand that chemotherapy has not improved outcomes for patients with early-stage oropharyngeal cancer. In fact, RTOG 0022 has a 94% locoregional control rate with radiation therapy alone for this cohort of patients. REFERENCE: International Journal of Radiation Oncology, Biology, Physics (Int J Radiat Oncol Biol Phys). 2010;76(5):1333-8.

161. A noninvasive head frame is used most commonly for accurate positioning of a patient receiving: A. palliative WBRT. B. single-fraction SRS. C. hypofractionated SBRT. D. IMRT for large intracranial tumors.

answer is C. RATIONALE: Large intracranial tumors and WBRT do not require the accuracy of a relocatable head frame; a face mask is sufficient. Single-fraction SRS usually uses an invasive head frame.

132. Which of the following mechanisms removes bulky adducts from DNA? A. Base excision repair B. Homologous recombination repair C. Nucleotide excision repair D. Mismatch repair

answer is C. RATIONALE: Nucleotide excision repair handles the removal of bulky adducts from DNA. Base excision repair repairs damaged bases only (not bulky adducts), homologous recombination is a DNA double strand break repair mechanism, and mismatch repair corrects DNA replication errors such as base-base mismatches.

6. Which of the following genes is commonly mutated in HNPCC? A. RB B. PTEN C. hMSH2 D. c-MYC

answer is C. RATIONALE: Of the four option choices, only hMSH2 is a mismatch repair gene that, when mutated or lost, causes the "mutator phenotype" characteristic of HNPCC.

129. What is a valid criticism of the radiation technique used to treat the inguinal lymph nodes in the GOG 88 (Stehman) study of patients who had vulvar cancer with clinically negative lymph nodes? A. The pelvis was treated with a four-field technique. B. The inguinal lymph nodes were treated with 12 MV photons. C. Patients with surgically positive lymph nodes received adjuvant radiation therapy to both the pelvic and inguinal lymph nodes. D. A midline block was used for treatment of the pelvic field.

answer is C. RATIONALE: One criticism of the study was that it prescribed 9- to 12-MeV electrons with 4- to 6-MV photons to a depth of 3 cm for radiation treatment of the inguinal lymph nodes in the non-CT era. With modern CT planning, the mean depth of inguinal lymph nodes is 6 cm. This difference meant about 30% less radiation dose was delivered to the inguinal lymph nodes. Another criticism was that patients with positive lymph nodes after surgery received adjuvant radiation therapy to both the pelvic and inguinal lymph nodes, while patients in the radiation arm received radiation to the inguinal lymph nodes only. Midline block causing an increased local failure at the primary site was a criticism of the Dusenberry study. REFERENCES: Stehman FB, et al. Groin dissection versus groin radiation in carcinoma of the vulva: a Gynecologic Oncology Group study. International Journal of Radiation Oncology, Biology, Physics (Int J Radiat Oncol Biol Phys). 1992;24(2):389-96; Koh WJ, et al. Femoral vessel depth and the implications for groin node radiation. International Journal of Radiation Oncology, Biology, Physics (Int J Radiat Oncol Biol Phys). 1993 Nov 15;27(4):969-74. Dusenbery KE, et al. Radical vulvectomy with postoperative irradiation for vulvar cancer: therapeutic implications of a central block. International Journal of Radiation Oncology, Biology, Physics (Int J Radiat Oncol Biol Phys). 1994 Jul 30;29(5):989-98.

185. What is the TNM stage of a right fallopian tube carcinoma that extends through the serosa and has surface implants on the ovary and the liver capsule? A. T1c B. T2 C. T3 D. M1

answer is C. RATIONALE: One fallopian tube with extension through the serosa (stage IC, T1c) would be upstaged to stage IIA if the ovary is involved. However, staging in this scenario is driven by the liver capsule implant. Peritoneal implants on the liver capsule are considered to be implants outside the pelvis (stage III, T3) while a liver parenchymal metastasis is stage M1 (IV)—this is also true of ovarian cancer.

165. Which of the following trials initially demonstrated an overall survival advantage for patients who received involved-field radiation therapy in addition to chemotherapy versus chemotherapy alone for non-Hodgkin lymphoma? A. ECOG B. RTOG C. SWOG D. CALBG

answer is C. RATIONALE: Only the initial report from the SWOG study showed an overall survival advantage in patients who received involved-field radiation therapy (RT) in addition to chemotherapy. The ECOG study demonstrated only a disease-free survival advantage. The GELA study included patients older that 60 years with good IPI and showed no advantage to RT. The RTOG has not conducted a non-Hodgkin lymphoma trial with chemotherapy and radiation therapy. There is no such cooperative group as the "CALBG." The organization is the CALGB. REFERENCES: Miller, et al. SWOG study. New England Journal of Medicine (NEJM). 1998. Horning, et al. ECOG study. Journal of Clinical Oncology (JCO). 2004. Bonnett, et al. GELA study. Journal of Clinical Oncology (JCO). 2007.

204. Which of the following statements about supratentorial primitive neuroectodermal tumor (S-PNET) is true? A. INI-1 deletion is commonly associated. B. S-PNET and medulloblastoma have identical genetic abnormalities. C. The survival rate is worse in patients with S-PNET than in those with infratentorial PNET. D. It is most commonly diagnosed in patients during the third decade of life.

answer is C. RATIONALE: Outcomes of supratentorial PNET (S-PNET) are worse than that of medulloblastoma (infratentorial PNET), with a progression-free survival rate of 49% vs. 78%, respectively. INI-1 deletion is found in ATRT. S-PNET is commonly diagnosed in younger patients. They have been found to have different cytogenetic abnormalities than medulloblastoma. REFERENCE: Timmermann B, et al. Journal of Clinical Oncology (J Clin Oncol). 2002;20:842-849. Exam answers and rationales Radiation Oncology In-Training Exam 2011 61

226. The leading cause of gynecological cancer deaths in the United States is: A. endometrial cancer. B. cervical cancer. C. ovarian cancer. D. vulvar cancer.

answer is C. RATIONALE: Ovarian cancer is the leading cause of death from gynecological malignancies in the United States. It is the 5th overall cause of cancer death in women, while cervical cancer is the 8th overall cause of cancer death in the United States. REFERENCE: American Cancer Society (ACS) Cancer Statistics. http://www.cancer.org/downloads/STT/Cancer_Facts_and_Figures_2010.pdf. Exam answers and rationales 68 American College of Radiology

313. Oxygen is a potent radiosensitizer because it: A. increases the yield of toxic aqueous hydrated electrons. B. generates hydrogen peroxide, which is an even more potent sensitizer. C. interacts with unpaired electrons in irradiated molecules to "fix" the damage. D. depletes naturally occurring antioxidants in cells.

answer is C. RATIONALE: Oxygen is the most potent chemical modifier of radiosensitivity because it readily combines with unpaired electrons in the outer shell of radiation-induced free radicals to generate peroxides. If this type of damage occurs in cellular DNA, the injury becomes fixed and irreversible, and in many cases, is more difficult to repair. Aqueous hydrated electrons and hydrogen peroxide are less toxic species than oxygen.

241. Which of the following structures is a parameningeal site associated with rhabdomyosarcoma? A. Orbit B. Parotid gland C. Nasopharynx D. External auditory canal

answer is C. RATIONALE: Parameningeal RMS sites include the infratemporal fossa, middle ear, mastoid region, nasal cavity, nasopharynx, paranasal sinus, and pterygopalatine fossa (IMMNNPPP). The parotid and external auditory canal would be considered head and neck, non-parameningeal sites. The orbit is not a parameningeal site and has its own treatment algorithm. REFERENCE: Friedman, Tarbell, Constine. Rhabdomyosarcoma. Halperin, Constine, Tarbell, Kun (eds). Pediatric Radiation Oncology. 4th ed. pp 319-346. Exam answers and rationales Radiation Oncology In-Training Exam 2011 73

172. According to the recent Quantitative Analysis of Normal Tissue Effects in the Clinic (QUANTEC), which of the following combinations of V20 and mean lung dose (MLD) of conventional fractionated radiation therapy is required to keep the risk of radiation pneumonitis below 20%? A. V20 <20%, MLD <40 Gy B. V20 <30%, MLD <30 Gy C. V20 <35%, MLD <20 Gy D. V20 <50%, MLD <15 Gy

answer is C. RATIONALE: QUANTEC recommends limiting V20 <30 to 35% and total mean lung dose <20 to 23 Gy to keep pneumonitis to <20% using conventional fractionation. It has been found by several institutions that V5, V10, and V20 are associated with radiation pneumonitis. Available clinical data showed that IMRT may reduce radiation pneumonitis due to more conformal treatment. SBRT could be associated with bronchial injury and stenosis if the lesion is located in the bronchial tree and a very high dose (60 Gy in 3 fractions) is delivered.

90. Which of the following statements about the heritable effects of radiation exposure is true? A. Heritable effects appear in the exposed individual. B. Heritable effects may be divided into two classes: prompt and delayed. C. There is a lower risk of heritable effects than somatic effects. D. Cancer is the most common heritable effect.

answer is C. RATIONALE: Risks for radiation-induced genetic effects in humans are considerably less than the risks for somatic effects. Somatic effects of radiation are limited to the exposed individual, as distinct from genetic effects, which affect subsequent generations of otherwise unexposed individuals. Somatic, but not genetic, effects can be divided into two classes: prompt and delayed (or early and late). Cancer is the most notable long-term somatic effect.

229. Which of the following organisms is correctly ordered from most radiosensitive to most radioresistant? A. HeLa cells < HIV < S. cerevisiae < E. coli B. S. cerevisiae < HeLa cells < HIV < E. coli C. HeLa cells < E. coli < S. cerevisiae < HIV D. E. coli < HIV < HeLa cells < S. cerevisiae

answer is C. RATIONALE: Since mammalian cells (HeLa) have the largest DNA content compared to bacteria (E. coli), yeast (S. cerevisiae), and viruses (HIV), they have the largest target for radiation damage; therefore, they are the most radiosensitive. Other factors that affect the inherent radiosensitivity of cells from different organisms include the ability to sense and repair DNA damage, the prevalent mode(s) of cell death, the cell cycle duration and phase, the degree of differentiation, etc. Exam answers and rationales Radiation Oncology In-Training Exam 2011 69

140. Specificity can be defined as the probability of: A. correctly classifying a person with a disease. B. falsely classifying a person with a disease. C. correctly classifying a person without a disease. D. falsely classifying a person without a disease.

answer is C. RATIONALE: Specificity is the probability that a test will be negative for patients who do not have a disease or that persons without a disease will be correctly identified.

100. Which of the following management options is most appropriate for a patient who has clinically residual anal cancer 4 weeks after completion of chemoradiation? A. Referral for salvage chemotherapy B. Referral for immediate salvage abdominoperineal resection C. Continued close clinical observation until disease progression D. Radiographic imaging to evaluate local disease progression

answer is C. RATIONALE: Squamous cell carcinoma (SCC) of the anal canal can regress slowly. Therefore, residual disease 4 weeks after therapy is not uncommon. Typically, biopsy is reserved for clinical progression or lack of complete regression at a later point in time (some advocate 12 weeks). If there is disease progression, a salvage abdominoperineal resection (APR) is appropriate. Chemotherapy would be reserved for metastatic disease recurrence. Patients are typically followed by a clinical exam and then a biopsy to document disease progression or persistence. Imaging may be obtained prior to surgery to ensure that there is no metastatic disease, but it would not typically be part of the surveillance protocol.

217. Stereotactic body radiation therapy (SBRT) is typically delivered: A. in a single treatment fraction. B. using conventional fractionation. C. to treat well-defined lesions that are 7 cm. D. as a prophylactic therapy.

answer is C. RATIONALE: Stereotactic body radiation therapy (SBRT) is typically used to treat lesions no larger than 7 cm in dimension. SBRT should not be used as a prophylactic therapy, and it is delivered with hypofractionated doses. Unlike SRS (which by definition is a single-fraction procedure), SBRT is fractionated. By definition, SRS is a single-fraction procedure. REFERENCES: Timmerman RD, et al. Seminars in Radiation Oncology. 2005;15(3):202-207. Khan FM. The Physics of Radiation Therapy. 4th ed. Lippincott, Williams & Wilkins. 2010.

317. What is the 5-year pelvic control rate for FIGO stage III vaginal cancer after definitive radiation treatment, as reported by Frank, et al. (MD Anderson)? A. 90% B. 80% C. 70% D. 50%

answer is C. RATIONALE: The 5-year pelvic control rates were 86% for stage I, 84% for stage II, and 71% for stages III-IV vaginal cancer. REFERENCE: Frank SJ, et al. International Journal of Radiation Oncology, Biology, Physics (Int J Radiat Oncol Biol Phys). 2005;62(1):138-147.

195. A patient's separation decreases by 6 cm. How much will the central axis dose change for an AP/PA treatment with 6 MV photons if the monitor units remain unchanged? A. 12% decrease B. 06% decrease C. 09% increase D. 18% increase

answer is C. RATIONALE: The 6 MV tissue-to-phantom (or tissue-to-air) ratio decreases at 3% per cm of added tissue. In this problem, the total separation is decreased by 6 cm, translating to 3 cm lost per beam to central axis. Thus, the central axis dose will be increased by (3 cm)*(3%/cm) = 9%. Exam answers and rationales 58 American College of Radiology

151. According to the ECOG 1484 (Horning, et al.) randomized trial, which of the following involved-field radiation doses was administered to patients who had non-Hodgkin lymphoma and a partial response to initial chemotherapy? A. 20 Gy B. 30 Gy C. 40 Gy D. 50 Gy

answer is C. RATIONALE: The ECOG 1484 trial used 40 Gy of involved-field radiation therapy for patients who had non-Hodgkin lymphoma and a partial response (PR) after 8 cycles of CHOP chemotherapy. REFERENCE: Horning, et al. Journal of Clinical Oncolog. 2004.

154. What is the appropriate statistical method to measure the degree of reliability between different and independent raters? A. Chi-square test B. McNemar's test C. kappa coefficient D. one-way analysis of variance

answer is C. RATIONALE: The Kappa coefficient is a widely used statistic for measuring the degree of reliability between different and independent raters. It compares the agreement against that which might be expected by chance. The Kappa coefficient ranges from 1 (perfect agreement) to 1 (complete disagreement). Here is one possible interpretation of Kappa. Poor agreement = Less than 0.20. Fair agreement = 0.20 to 0.40. Moderate agreement = 0.40 to 0.60. Good agreement = 0.60 to 0.80. Very good agreement = 0.80 to 1.00.

143. What is the risk of occult inguinofemoral lymph node metastasis in patients with a 1.5 cm squamous cell carcinoma of the vulva? A. <5% B. 08% C. 20% D. 34%

answer is C. RATIONALE: The Mayo clinic reviewed over 200 patients and determined that a stage T1 tumor had a 15% risk and a stage T2 tumor had a 30% risk of occult inguinofemoral lymph node metastasis. Based on the tumor size at the primary site, the risk was 7% (<1 cm), 22% (1.1 to 2 cm), 27% (2.1 to 3 cm), and 34% (3.1 to 5 cm). REFERENCE: Gonzalez Bosquet J, et al. Risk of occult inguinofemoral lymph node metastasis from squamous carcinoma of the vulva. International Journal of Radiation Oncology, Biology, Physics (Int J Radiat Oncol Bio Phys). 2003 Oct 1;57(2):419-24.

3.5 cm bladder tumor, with a bimanual examination revealing a residual 2.5 cm mobile mass? A. T1 B. T2 C. T3 D. T4

answer is C. RATIONALE: The clinical staging of bladder cancer is based on bimanual examination following cystoscopic evaluation and complete TURBT. The absence of a palpable abnormality is classified as stage T2 and represents muscle-invasive disease. A mobile mass following complete TURBT suggests residual tumor due to extravesicular extension and is classified as stage T3. A fixed mass is consistent with invasion into adjacent structures and is classified as stage T4.

186. Based on the current Response Evaluation Criteria in Solid Tumors (RECIST), a partial response (PR) is defined as a decrease in the: A. tumor volume by 30%. B. tumor volume by 50%. C. longest target lesion dimension by 30%. D. longest target lesion dimension by 50%.

answer is C. RATIONALE: The current (unidimensional) criteria for tumor response rely on the longest dimension, not on a product of two dimensions (as was the case with the previous bidimensional RECIST). Therefore, the percent decrease is 30% (not 50%), which is frequently a source of confusion. Tumor response was never based on tumor volume, which is not obtainable on standard imaging.

78. According to prospective randomized studies for head and neck cancer, which of the following is an absolute indication for postoperative chemotherapy and radiation? A. 4 positive lymph nodes B. 3 cm positive lymph nodes C. Presence of a positive margin D. Presence of perineural invasion

answer is C. RATIONALE: The indications for the addition of postoperative chemoradiation therapy are positive margins and extranodal extension. The other options are not based on the combined EORTC & RTOG analyses.

155. Based on the 12-year results from the NSABP B17 trial, the IBTR rate of DCIS after excision alone was: A. 08%. B. 16%. C. 32%. D. 40%.

answer is C. RATIONALE: The ipsilateral breast tumor recurrence (IBTR) rate of DCIS after excision alone was 32%. All surgical margins were negative (no tumor-filled ducts at ink), 80% of cases were detected by mammography, and treatment was 50 Gy to the breast, without a boost. REFERENCE: Fisher B, Land S, Mamounas E, et al. Prevention of invasive breast cancer in women with ductal carcinoma in situ: an update of the national surgical adjuvant breast and bowel project experience. Seminars in Oncology (Semin Oncol). 2001;28:400-418.

169. Based on the 12-year results from the NSABP B17 trial, the IBTR rate of DCIS after excision and radiation therapy was: A. 04%. B. 08%. C. 16%. D. 24%.

answer is C. RATIONALE: The ipsilateral breast tumor recurrence (IBTR) rate of DCIS after excision and irradiation was 16%. All surgical margins were negative (no tumor-filled ducts at ink), 80% of cases were mammographically detected, and treatment consisted of 50 Gy to the breast, without a boost. REFERENCE: Fisher B, Land S, Mamounas E, et al. Prevention of invasive breast cancer in women with ductal carcinoma in situ: an update of the National Surgical Adjuvant Breast and Bowel Project experience. Seminars in Oncology (Semin Oncol). 2001;28:400-418.

125. According to AAPM TG-51, what is the most compelling reason that water phantoms are used for absorbed dose calibrations? A. The attenuation factors of water most closely mimic materials used to construct ion chambers. B. The stopping powers of charged particles are more accurate in water than any other medium. C. The calibration factor is a direct absorbed dose to water factor. D. Water is the clinically relevant endpoint.

answer is C. RATIONALE: The linac output is measured with ion chambers calibrated by standards laboratories. Ion chambers can be made of various materials, which can be (but need not be) matched to the phantom material. The AAPM TG-51 outlines the protocol for clinical reference dosimetry in terms of absorbed dose to water calibration factors. The absorbed dose to water calibration factor (Nd,w) introduced in AAPM TG-51 is traceable to National Institute of Standards and Technology (NIST) standards. Exam answers and rationales 38 American College of Radiology

173. Precursor T cell lymphoblastic lymphoma most commonly involves the: A. skin. B. bone. C. mediastinum. D. spleen.

answer is C. RATIONALE: The mediastinum is the most common site of involvement for precursor T cell lymphoblastic lymphoma. REFERENCE: Tarbell and Weinstein. Non- Hodgkin lymphoma. Halperin, Constine, Tarbell, Kun (eds). Pediatric Radiation Oncology. 4th ed. pp 261-270. Exam answers and rationales 52 American College of Radiology

117. What is the most common site of presentation in a patient with a solitary extramedullary plasmacytoma? A. Pelvis B. Extremities C. Head and neck D. Abdomen and thorax

answer is C. RATIONALE: The most common anatomical region for a solitary extramedullary plasmacytoma is the head and neck area. REFERENCE: Corwin J, Lindberg RD. Solitary plasmacytoma of bone vs. extramedullary plasmacytoma and their relationship to multiple myeloma. Cancer. 1979;43:1007-1013.

43. The rapid dose falloff of a Cs-137 source in a 5 cm radius of tissue is primarily due to: A. anisotropy. B. tissue attenuation. C. the inverse square law. D. gamma-ray spectrum hardening.

answer is C. RATIONALE: The most significant factor of Cs-137 dosimetry implants is the dose falloff due to a distance change.

249. Which of the following parameters remains unchanged with decreasing electron field size? A. Surface dose B. Depth of maximum dose C. Practical range of the electron D. Distance between depth of 90% and 10% dose

answer is C. RATIONALE: The only parameter listed that remains unchanged with decreasing electron field size is the practical range, Rp. Surface dose will increase; the depth of maximum dose (dmax) will shift toward the surface; and the steepness of the depth dose curve beyond dmax will increase, resulting in a larger d90 to d10 distance.

293. The AAPM TG-142 updated report on QA procedures for medical accelerators does NOT include QA procedures for: A. multileaf collimators. B. electronic portal imaging devices. C. non-radiographic localization and positioning systems. D. respiratory gating systems.

answer is C. RATIONALE: The only topic listed above that is not covered in AAPM TG-142 is QA recommendations for non-radiographic localization and positioning systems (i.e., Calypso). In 2010, the AAPM approved a new task group, TG-147, to address this issue.

207. What is the 10-year overall survival rate for a patient with solitary plasmacytoma of the bone? A. 10% B. 25% C. 50% D. 80%

answer is C. RATIONALE: The prognosis is important for providing informed discussions with a patient who has a solitary plasmacytoma of the bone. There can be longterm survivors, but they are at risk for the development of myeloma. Exam answers and rationales 62 American College of Radiology

55. The square root of the variance equals the: A. risk ratio. B. error coefficient. C. standard deviation. D. degrees of freedom.

answer is C. RATIONALE: The square root of the variance equals the standard deviation and is a measure of the spread of a set of data. Exam answers and rationales Radiation Oncology In-Training Exam 2011 17

93. The use of an endorectal balloon during radiation therapy for prostate cancer has been shown to: A. interfere with image guidance. B. increase daily prostate motion. C. decrease the severity of rectal proctopathy. D. improve freedom from biochemical failure.

answer is C. RATIONALE: The use of an endorectal balloon has been shown to be helpful during radiation therapy for prostate cancer to assist with image guidance, to decrease daily prostate motion, and to decrease the severity of rectal proctopathy, but not to improve freedom from biochemical failure. REFERENCE: Smeenk RJ, Teh BS, Butler EB, van Lin EN, Kaanders, JH. Is there a role for endorectal balloons in prostate radiotherapy? A systematic review. Radiotherapy and Oncology. 2010;95:277-282.

271. Comprehensive irradiation for a stage T0N2b squamous cell carcinoma in the level II lymph nodes of the neck will decrease the emergence of the primary site from: A. 70% to 30%. B. 50% to 25%. C. 30% to 15%. D. 10% to 2%.

answer is C. RATIONALE: This item is based on multiple retrospective reviews. REFERENCES: Colletier, PJ, et al. Postoperative radiation therapy for squamous cell carcinoma metastatic to cervical lymph nodes from an unknown primary site: outcomes and patterns of failure. Issing, Wj. Carcinoma of unknown primary, a survey in 167 patients. Head and Neck. 1998;20:674-681. Laryngorhinootologie. 2003;82:659-665.

179. A 50-year-old man presents with stage IIB, non-bulky, diffuse, large B-cell non-Hodgkin lymphoma involving the neck and superior mediastinum. Which of the following statements about the management of this patient is true? A. Regardless of treatment, the 5-year overall survival rate is less than 50%. B. Treatment should include high-dose chemotherapy, followed by autologous stem cell transplantation. C. Treatment should include either 8 cycles of R-CHOP or 3 cycles of R-CHOP, followed by involved-field radiation therapy. D. R-CHOP for 3 cycles is the best treatment.

answer is C. RATIONALE: This patient presents with aggressive non-Hodgkin lymphoma (NHL); however, he still has a fair survival probability with either 8 cycles of R-CHOP or a combination of 3 cycles of R-CHOP and involved-field radiation therapy (IFRT) as confirmed by randomized data. Rituximab as anti-CD20 targeted agent was found to improve patients' outcome. REFERENCES: Horning SJ, Weller E, Kim K, et al. Chemotherapy with or without radiotherapy in limited-stage diffuse aggressive non-Hodgkin's lymphoma: Eastern Cooperative Oncology Group study 1484. Journal of Clinical Oncology (J Clin Oncol). 2004;22: 3032-3038. Miller TP, Spier CM, Rimsza L. Diffuse aggressive histologies of non-Hodgkin lymphoma: treatment and biology of limited disease. Seminars in Hematology (Semin Hematol). 2006;43:207-212.

232. According to a pooled (Warde, et al.) analysis of patients with stage I seminoma treated with inguinal orchiectomy and no adjuvant therapy, what is the 5-year relapse risk for a man with a 5 cm tumor that invades the rete testis? A. 16% B. 25% C. 32% D. 40%

answer is C. RATIONALE: This pooled analysis of 638 patients surveilled with stage I seminoma identified two variables associated with relapse on multivariable analysis: rete testes invasion and tumor size >4 cm. The 5-year risk of relapse was 12%, 16%, and 32% for patients with 0, 1, or 2 adverse factors, respectively. REFERENCE: Warde, et al. Journal of Clinical Oncology (J Clin Onc). 2002;20:4448-4452. Exam answers and rationales 70 American College of Radiology

50. Medical linear accelerators produce high-energy x-rays when: A. photons are accelerated by a resonant microwave waveguide only. B. electrons are accelerated by a resonant microwave waveguide and radiate x-rays during the acceleration. C. electrons are accelerated by a resonant microwave waveguide and are then rapidly decelerated by striking a target, producing bremsstrahlung radiation. D. electrons are accelerated across a voltage potential and are then rapidly decelerated by striking a target, producing bremsstrahlung radiation.

answer is C. RATIONALE: This question requires basic knowledge of a linear accelerator. Option A is incorrect because photons cannot be accelerated. Option B does not describe correctly how electrons are converted to usable photons. Option D more accurately describes an x-ray tube. REFERENCE: Khan FM. The Physics of Radiation Therapy. 4th ed. Chapter 4. pp 38-43.

256. What is the typical energy range of protons used for radiation therapy? A. 080 to 120 kVp B. 004 to 18 MeV C. 070 to 250 MeV D. 400 to 800 MeV

answer is C. RATIONALE: To achieve a depth of penetration of about 40 cm (an approximate upper limit for patient thickness), protons on the order of 250 MeV are required.

257. Which of the following factors is associated with a favorable prognosis for patients with CNS germ cell tumors? A. Age of less than 3 years B. Elevated serum AFP level C. Pure germinoma histology D. Origin in the pineal gland

answer is C. RATIONALE: Unlike mixed germ cell tumors, pure CNS germinomas have an excellent prognosis after chemoradiation regimens or extended-field radiation therapy alone. Being less than 3 years of age is associated with worse event-free survival because of the reluctance to use radiation therapy. An elevated serum AFP level is diagnostic of a nongerminomatous component. The site of the disease (pineal vs. hypothalamic) is not of prognostic significance. Exam answers and rationales 78 American College of Radiology

67. Which statistical technique is used when recent trial outcomes of partial breast irradiation are compared to previously published outcomes of whole-breast irradiation? A. Cohort analysis B. Univariate analysis C. Historical control D. Case control

answer is C. RATIONALE: Using historically published data as a comparison to the results of more recently published results is an example of a historical control. The control group is represented by patients who were treated in the past in comparison to patients who were treated more recently. Cohort analysis seeks to follow prospectively a group of patients who share common characteristics over a length of time. Univariate analysis seeks to determine the effect of one variable on an outcome (ex - age at diagnosis and local control). Case control studies are retrospective analyses in which patients with a certain diagnosis (controls) are compared against similar patients without the diagnosis to assess for differences between the two groups.

310. Which structure is most likely to be involved in a patient with FIGO stage II vaginal cancer? A. Upper vagina with extension to the cervix B. Middle vagina with extension to the pelvic sidewall C. Lower vagina with extension to the paravaginal tissues D. Lower vagina with extension to the labia minora

answer is C. RATIONALE: Vaginal tumors that involve the cervix or vulva are staged as cancer arising from those structures, not as vaginal cancer. If the vaginal cancer extends to the pelvic sidewall, it is considered to be stage III disease. Stage II vaginal cancers are confined to the vagina and paravaginal tissues. REFERENCE: AJCC Cancer Staging Manual. 7th ed. 2009.

277. According to the GOG-92 trial, which outcome was associated with postoperative pelvic irradiation in patients who had stage IB cervical carcinoma with poor prognostic features? A. There was a statistically significant improvement in overall survival. B. The reduction in the risk of recurrence did not reach statistical significance. C. There were fewer recurrences in patients with adenocarcinomas or adenosquamous carcinomas than with other histologies. D. Seventy percent of all disease-related deaths occurred in the first year after enrollment.

answer is C. RATIONALE: With postoperative radiation, 8.8% of patients with adenocarcinoma or adenosquamous tumors experienced disease recurrence versus 44% of patients with other histologies. The addition of radiation showed a statistically significant reduction in risk of recurrence (46%), but it did not reach significance in overall survival. Seventy percent of disease-related deaths occurred in the first 3 years after enrollment. REFERENCE: International Journal of Radiation Oncology, Biology, Physics (Int J Radiat Oncol Biol Phys). 2006;65(1):169-176.

28. What is the tumor stage of a squamous cell carcinoma that arises from the penile urethra and invades the corpus cavernosum? A. T1 B. T2 C. T3 D. T4

answer is C. REFERENCE: AJCC Cancer Staging Handbook. 7th ed. 2009. Exam answers and rationales Radiation Oncology In-Training Exam 2011 9

72. Which of the following is associated with primary radiation therapy for skin cancers of the nose? A. Nasal ala involvement has the worst outcome. B. Regardless of the tumor size, both basal cell and squamous cell carcinomas have the same control rate. C. The failure rate is much higher when the nasal columella and vestibule are involved. D. Orthovoltage irradiation has a higher risk of necrosis than brachytherapy.

answer is C. REFERENCE: Refer to the largest reported series: Radiation therapy of carcinomas of the skin of the nose and nasal vestibule: A report of 1676 cases by the Groupe European de Curietherapie. Radiotherapy and Oncology. 1989;3:165-73.

171. Which of the following gross surgical resection margins is considered to be the minimum required for a wide local excision of an early squamous cell carcinoma of the vulva? A. 02 mm B. 05 mm C. 07 mm D. 10 mm

answer is D. RATIONALE: A 10-mm surgical margin (8 mm if formalin fixed) is associated with a high local control rate and does not need adjuvant radiation therapy, as compared to a 48% local failure rate if the surgical margin is <8 mm (inadequate). REFERENCE: Heaps JM, et al. Surgical-pathologic variables predictive of local recurrence in squamous cell carcinoma of the vulva. Gynecologic Oncology.(Gynecol Oncol). 1990 Sep;38(3):309-14.

253. Which of the following regimens is considered to be first-line chemotherapy for malignant mesothelioma? A. Cisplatin alone B. Pemetrexed alone C. Cisplatin and etoposide D. Cisplatin and pemetrexed

answer is D. RATIONALE: A combined first-line regimen using cisplatin and pemetrexed is considered the gold standard for malignant mesothelioma. In a multicenter phase III study involving 448 patients, those treated with pemetrexed plus cisplatin had a longer overall median survival (12.1 months) than those treated with cisplatin alone (9.3 months) and had an objective response rate (shrinkage of the tumor by at least 50%) of 41%. REFERENCE: Journal of Clinical Oncology (J Clin Oncol). 2003;21:2636-44. Exam answers and rationales Radiation Oncology In-Training Exam 2011 77

65. At approximately what wavelength does electromagnetic radiation become ionizing? A. 100 μm B. 010 μm C. 001 μm D. 000.1 μm

answer is D. RATIONALE: A photon with a wavelength of approximately 0.1 μm (100 nm or 1 x 10-7 m) is capable of depositing about 125 keV of energy into the medium through which it passes. This amount of energy is minimally sufficient to disrupt atomic bonds and ionize atoms.

69. Which of the following diagnostic exams would be best for initial evaluation of a patient with suspected multiple myeloma? A. MRI B. CT scan C. Bone scan D. Skeletal survey

answer is D. RATIONALE: A skeletal survey is the best study to quantify lesions in a patient with multiple myeloma. REFERENCE: DeVita VT, Hellman S, Rosenberg, SA (eds). Cancer: Principles & Practice of Oncology. 6th ed. Lippincott. 2001; p 2473. Exam answers and rationales Radiation Oncology In-Training Exam 2011 21

262. According to AAPM TG-51, electron beam quality is characterized by: A. PDD at 10 cm. B. depth of maximum dose (dmax) absorbed in water. C. depth at which the ionization falls to 50% (I50) of its maximum value. D. depth at which the absorbed dose falls to 50% (R50) of its maximum value.

answer is D. RATIONALE: According to AAPM TG-51, the beam quality for electron beams is specified by R50. The dmax is not used because as the electron energy increases, the dose drop off following its peak becomes more gradual and may peak at similar depths for multiple electron energies. The PDD(10)x is used to characterize photon beams. Lastly, unlike photons, differences in I50 and R50 will be observed in ion chambers due to significant changes in the stopping power ratio of electrons in water with depth.

19. What is the most appropriate postoperative management for a patient who has a 1 cm, grade 1, superficial, endometrioid adenocarcinoma confined to the uterine fundus discovered incidentally during a hysterectomy for uterine fibroids? A. Pelvic EBRT B. Pelvic and paraaortic lymph node dissection C. Vaginal brachytherapy D. Observation

answer is D. RATIONALE: According to GOG 33, the risk of pelvic and paraaortic lymph node metastases in a patient with grade 1 endometrioid adenocarcinoma confined to the endometrium was 0% and 0%, respectively. This patient has low-risk endometrial cancer and would not have met the criteria for adjuvant therapy trials. Unlike patients with benign disease, this patient should undergo close follow-up with physical exams and vaginal cytology every 3-6 months for 2 years initially, and then annually after 2 years of follow-up. REFERENCE: Creasman WT, et al. NCCN uterine neoplasms guidelines. Cancer. 2010;60:2035-41.

147. A conformity index is defined by the ICRU as the ratio of the: A. gross tumor volume and planning target volume. B. clinical target volume and planning target volume. C. treated volume and gross tumor volume. D. treated volume and planning target volume.

answer is D. RATIONALE: According to ICRU Report 62, the conformity index is defined as the ratio of the treated volume and the planning target volume.

289. Based on the updated TNM staging system, what tumor stage should be assigned to a macroscopically visible squamous cell carcinoma of the cervix only with superficial invasion? A. T0 B. T1a1 C. T1a2 D. T1b

answer is D. RATIONALE: According to the AJCC Cancer Staging Manual, 7th ed., any macroscopically visible lesion of the cervix (even with superficial invasion) is assigned stage T1b.

39. What is the most significant source of background radiation received by an individual (excluding medical x-ray exams)? A. Cosmic radiation B. Internal radiation C. Nuclear power D. Radon gas

answer is D. RATIONALE: According to the UNSCEAR 2006 Report, "Effects of Ionizing Radiation," the largest contributing factor to background radiation is radon gas. Exam answers and rationales 12 American College of Radiology

59. According to the Collaborative ocular melanoma study (COMS) protocol guidelines, the dose prescription point for the radiation treatment of an ocular melanoma is: A. fixed at a depth of 5 mm. B. at the tumor apex. C. at the tumor apex if the tumor height is 5 mm, and at a depth of 5 mm if the tumor height is 5 mm. D. at the tumor apex if the tumor height is 5 mm, and at a depth of 5 mm if the tumor height is 5 mm.

answer is D. RATIONALE: According to the collaborative ocular melanoma study (COMS) protocol, the prescription point is at the tumor apex for tumors that are 5 mm or greater in height, and at 5 mm for tumors that are less than 5 mm in height. The American Brachytherapy Society (ABS), on the other hand, recommends a prescription point at the tumor apex. REFERENCE: http://www.jhu.edu/wctb/coms/index.htm (General information: Radiation Therapy). Nag S, et al. International Journal of Radiation Oncology, Biology, Physics (Int J Radiat Oncol Biol Phys). 2003;56(2):544-555.

214. A 65-year-old man underwent lobectomy and mediastinal nodal dissection for stage pT3N2 NSCLC with negative surgical margins. Which of the following treatment options is most appropriate for this patient? A. Close surveillance B. Chemotherapy alone C. Radiation therapy alone D. Both chemotherapy and radiation

answer is D. RATIONALE: Adjuvant chemotherapy improves survival in patients with stage II to stage III NSCLC. Postoperative radiation therapy improves local control and disease-free survival in patients with stage N2 disease. Concurrent chemoradiotherapy has not been shown to further improve survival or local control in the adjuvant setting for patients with stage N2 disease and negative surgical margins.

188. Radiation-induced permanent growth arrest is a form of: A. apoptosis. B. autophagy. C. mitotic catastrophe. D. senescence.

answer is D. RATIONALE: Among the functional consequences of ionizing radiation exposure, cell death is the most important process for radiation oncology. Cells can die in several different ways after irradiation, including mitotic catastrophe, apoptosis, senescence, necrosis, and autophagy. Senescence is the phenomenon by which normal diploid cells lose their ability to divide over time, which, for human cells, normally occurs after about 50 cell divisions in vitro. Typically, senescent cells remain intact and metabolically viable (at least in the short term), despite having lost their clonogenicity. Radiation can likewise cause cells to senesce; however, in this case, the process is pathological in nature rather than natural (i.e., in response to DNA damage), so it tends to go by the name "permanent growth arrest" rather than simply "senescence." Exam answers and rationales 56 American College of Radiology

224. Radiation therapy to the internal mammary lymph nodes is indicated in which of the following breast cancer scenarios? A. PSIIA (T2N0) after partial mastectomy and sentinel lymph node biopsy B. CSIIB (T2N1) after neoadjuvant chemotherapy with complete pathologic response C. PSIIB (T2N1a) after partial mastectomy and axillary lymph node dissection, with positive results in 1 of 13 lymph nodes D. CSIIIA (T1N2b), enlarged internal mammary lymph nodes on CT scan, and clinically negative axilla

answer is D. RATIONALE: An absolute indication for treatment of the internal mammary nodes is clinical involvement of the internal mammary nodes, whether by physical examination or by imaging. The addition of internal mammary nodes in other treatment scenarios is controversial at this time and is otherwise left to the discretion of the treating radiation oncologist. REFERENCE: NCCN Practice Guidelines in Oncology. vol 2. 2010. http://www.nccn.org.

136. A 60-year-old patient with a long history of tobacco use presents with an enlarging mass in the neck. Which of the following diagnostic tests is most appropriate? A. PET scan B. Incisional biopsy C. Excisional biopsy D. Fine-needle aspiration

answer is D. RATIONALE: An incisional/excisional biopsy for a patient with a presumed squamous cell carcinoma is inappropriate because it may cause violation of the neck, which may require postoperative radiation. Fine-needle aspiration (FNA) is the first treatment choice unless a lymphoma is suspected.

9. An 8-year-old girl presents with petechiae, fatigue, leukocytosis, palsy of cranial nerve VI, and acute lymphoblastic leukemia with negative results on CSF cytology. The patient's CNS classification status most likely is: A. CNS 0. B. CNS 1. C. CNS 2. D. CNS 3.

answer is D. RATIONALE: Any cranial nerve palsy at diagnosis of acute lymphoblastic leukemia is considered to be CNS leukemia, regardless of CSF cytology by definition. Therefore, CNS 3 is the proper classification.

128. Which of the following statements about the treatment of anal cancers is true? A. Combining chemotherapy with radiation therapy has a survival advantage over radiation alone. B. Mitomycin C can be omitted from chemoradiation regimens with no change in disease-free survival. C. Randomized trial data has shown that cisplatin is superior to mitomycin C in terms of colostomy-free survival when combined with 5-FU and radiation therapy. D. Colostomy-free survival is improved by the addition of 5-FU and mitomycin C compared to radiation therapy alone.

answer is D. RATIONALE: At least 2 randomized trials (EORTC and UKCCCR) have shown that the addition of 5-FU and mitomycin C to radiation therapy is superior in local control (LC) and disease-free survival (DFS) in patients with anal cancer. The UKCCCR trial showed a significant increase in colostomy-free survival. There was no advantage in overall survival (OS) demonstrated in either trial. The role of mitomycin C was assessed in a randomized trial by FLAM and colleagues, comparing radiation therapy (RT) plus 5-FU to RT plus 5-FU and mitomycin C. Mitomycin C improved colostomy-free survival and DFS (with no significant difference in OS). RTOG 98-11 assessed the role of cisplatin in the treatment of anal canal cancer by substituting it for mitomycin C, given concurrently with 5-FU and radiation therapy. This was compared to the standard Nigro regimen of 5-FU and mitomycin C. The conclusions are somewhat limited by the fact that the cisplatin arms had 2 cycles of induction cisplatin and 5-FU, whereas the standard arm had no induction therapy. There was an increase in colostomy rates and local failure in the cisplatin arm. Exam answers and rationales Radiation Oncology In-Training Exam 2011 39

166. Which of the following genes is induced in a p53-dependent manner in response to DNA damage? A. ATR B. ATM C. p63/TP63 D. p21/WAF1

answer is D. RATIONALE: Ataxia telangiectasia mutated (ATM) is a serine/threonine protein kinase that is activated by self-phosphorylation when cells are exposed to radiation. ATM then phosphorylates a number of downstream proteins involved in cell cycle checkpoint control, apoptotic responses, and DNA repair, including p53, Chk2, BRCA1, H2AX, SMC1, FANCD2, Rad17, and Artemis. P53 is activated and induces expression of p21, thereby leading to G1 cell cycle arrest. Exam answers and rationales 50 American College of Radiology

24. Which of the following sites of skin cancer has the highest risk for locoregional recurrence after surgery? A. Cheek B. Auricle C. Forehead D. Medial canthus

answer is D. RATIONALE: Basal and squamous cell carcinomas of the medial canthus may display deep invasion, involving the medial canthal tendon, the lacrimal apparatus, the ethmoid sinuses, and the periorbita, which can lead to invasion to the skull base. Exam answers and rationales 8 American College of Radiology

200. A 7.5 x 3 cm adenocarcinoma of the left upper lobe of the lung without pleural or bronchial tree involvement would be classified as stage: A. T1. B. T2a. C. T2b. D. T3.

answer is D. RATIONALE: Based on the new, 2009 IASLC staging system, the T stage is divided by the tumor size: T1a = 2 cm; T1b = 2 cm but 3 cm; T2a = 3 cm but 5 cm; T2b = 5 cm but 7 cm; T3 = 7 cm. This information is new compared with the prior Tstage system.

280. Particle selection for high LET radiation therapy should be determined by the particle's ability to: A. cause tumor cell senescence. B. inhibit DNA repair pathways in tumor cells. C. maximize the number of ionizations per cell. D. achieve a high RBE and a low OER.

answer is D. RATIONALE: Biologically speaking, the goal of high LET radiation therapy is to maximize the RBE and minimize the OER. Exposure to high LET radiation does not inhibit DNA repair per se, although the nature of the DNA damage produced is more severe, and therefore, less repairable overall; this accounts in part for why high LET radiations have high RBEs. In terms of ionizations per cell, the goal is not necessarily to maximize the number of them, but rather to ensure that the ionizations that do occur are optimally spaced to coincide with the diameter of the DNA helix, which increases the likelihood of producing lethal DNA double strand breaks from the passage of a single particle track. Finally, although radiationinduced damage can lead to cellular senescence, this phenomenon remains poorly understood and is not considered a principal justification for the use of high LET radiation therapy.

124. Which of the following statements about cell loss from tumors is true? A. Distant metastases develop from many of the "lost" cells. B. The cell loss factor is determined using the relative movement flow cytometric method. C. Tumors with high cell loss tend to regress slowly following radiation therapy. D. Cell loss factors for some human carcinomas can be as high as 90%.

answer is D. RATIONALE: Cell loss factors for human carcinomas tend to be between 70% and 90%; whereas for sarcomas, the values typically range between 30% and 50%. Although some cells lost from the primary tumor mass go on to form distant metastases, the likelihood that this will happen is very small. It is estimated that no more than one tumor cell in 100,000, if that, has both the "competency" (phenotypically speaking) to reform a tumor at a distant site and the ability to survive the perilous migration to that site. A tumor's potential doubling time (TPOT), not its cell loss factor, is determined using the relative movement flow cytometric method. Tumors with high cell loss are more likely to regress rapidly (not to regress slowly) following radiation therapy. REFERENCE: Tannock, Hill, Bristow, Harrington. The Basic Science of Oncology. 4th ed. Chapter 11.

15. Less damage occurs per unit dose for x-irradiated DNA in a lyophilized state than in an aqueous solution because: A. dry DNA tends to become more hypoxic than DNA dissolved in solution. B. chromatin conformation is more open in a dry versus aqueous form, affecting the DNA's radiosensitivity. C. DNA repair half-times are much longer for DNA in an aqueous solution than in the dry form. D. free radicals liberated by the radiolysis of water account for much of the total radiation-induced DNA damage.

answer is D. RATIONALE: Cellular DNA can be damaged by ionizing radiation either directly, in the case where one or more of the atoms comprising DNA is ionized, or indirectly, in the case where other molecules (mostly water) in the cell near the DNA are ionized, and these diffuse to the DNA and damage it. Current estimates are that, for low LET radiation like x-rays, about 30% of the DNA damage is caused by direct ionization and about 70% is caused indirectly, secondary to the radiolysis of water. However, the proportion of direct damage increases as the LET of the radiation increases. This accounts for why there is quantitatively less damage per unit dose for dry DNA than for DNA in an aqueous solution. REFERENCE: Hall and Giaccia. Radiobiology for the Radiologist. 6th ed. Chapter 1. Figure 1.8. Exam answers and rationales Radiation Oncology In-Training Exam 2011 5

113. Extracapsular extension found during an axillary lymph node dissection for breast cancer staging is most likely to increase the risk for: A. local recurrence. B. axillary recurrence. C. supraclavicular recurrence. D. distant metastatic recurrence.

answer is D. RATIONALE: Extracapsular extension (ECE) does not independently increase the risk for axillary or regional recurrence but is closely associated on multivariate analysis with the number of positive lymph nodes and, therefore, distant recurrence. REFERENCES: Gruber, et al. Annals of Oncology. 2008;19:1393-1401. Mignano, et al. Cancer. 1999;86:1258-62. Hetelekidis, et al. International Journal of Radiation Oncology, Biology, Physics (Int J Radiat Oncol Biol Phys). 2000;46:31-34. Pierce, et al. International Journal of Radiation Oncology, Biology, Physics (Int J Radiat Oncol Biol Phys). 1995;33:253-

111. For a 20 cm SSD superficial x-ray unit, a 0.5 cm change in patient position will result in approximately what percent change in depth dose at 0.5 cm? A. 2% B. 3% C. 4% D. 5%

answer is D. RATIONALE: For a 20 cm SSD machine, the loss of photon fluence per

76. Which of the following treatments would be most appropriate for a 3 cm pituitary mass with optic chiasm compression? A. SRS, 15 Gy in a single fraction B. WBRT, 45 to 50 Gy in 25 fractions C. Biopsy, followed by radiation therapy D. Resection with reassessment for further therapy

answer is D. RATIONALE: For a large pituitary mass with chiasmal compression, an effort should be made to surgically decompress the optic chiasm and maximally debulk the tumor. Postoperative radiation therapy can be considered once the extent of the resection has been determined and postoperative changes have been stabilized.

250. What is the most important clinical feature that distinguishes inflammatory breast cancer from locally advanced breast cancer with secondary dermal lymphatic congestion? A. Breast warmth B. Brawny breast induration C. Asymmetric breast enlargement D. Rapid onset of symptoms

answer is D. RATIONALE: Hallmarks of inflammatory breast cancer are rapid onset of disease and clinical findings of skin erythema, edema (peau d'orange), brawny breast induration, warmth, and asymmetric enlargement. Neglected primary tumors can lead to these same findings, particularly in the setting of bulky axillary lymphadenopathy. Inflammatory cancer has a rapid onset of disease, while locally advanced cancers tend to have long intervals between the first symptom/finding and the patient's presentation for medical treatment. REFERENCES: Buchholz TA, Haffty BG. Breast cancer: locally advanced and recurrent disease, postmastectomy radiation, and systemic therapies. Halperin EC, Perez CA, Brady LW (eds). Principles and Practice of Radiation Oncology. 5th ed. Chapter 54. Philadelphia: Lippincott Williams & Wilkins Publishers. 2008; p 1293. Exam answers and rationales 76 American College of Radiology

235. Which of the following toxicity scoring systems for assessing radiation-induced side effects has been adopted by the Cancer Therapy Evaluation Program (CTEP) of the National Cancer Institute? A. LENT/SOMA toxicity scale B. Southwest Oncology Group (SWOG) toxicity scale C. Radiation Therapy Oncology Group (RTOG) toxicity scale D. Common Terminology Criteria for Adverse Events (CTCAE)

answer is D. RATIONALE: In 1995, the EORTC and RTOG proposed a system for scoring late effects in normal tissues using subjective, objective, management and analytic scales (LENT/SOMA). In 2003, the National Cancer Institute (NCI) published the CTCAE, which incorporated many of the elements of the LENT/SOMA system, but with early and late effects contained in one system. Exam answers and rationales Radiation Oncology In-Training Exam 2011 71

180. Epigenetic silencing of which of the following genes increases the effectiveness of temozolomide in patients with glioblastoma? A. PTEN B. EGFR C. CDKN2A D. MGMT

answer is D. RATIONALE: In randomized trials testing temozolomide plus radiation versus radiation only, those patients with hypermethylation and epigenetic silencing of the MGMT (O6-methylguanine-DNA-methyltransferase) gene derived the greatest benefit from temozolomide. Exam answers and rationales 54 American College of Radiology

150. According to the RTOG 0129 trial, which of the following patients with squamous cell carcinoma of the tonsil has the worst overall survival rate? Smoking History HPV Status Disease Stage A. 10-pack-year Positive T3N1 B. 10-pack-year Negative T3N1 C. 30-pack-year Positive T1N2b D. 30-pack-year Negative T1N1

answer is D. RATIONALE: It is important for candidates to understand recursive-partitioning analysis to identify prognostic factors for oropharyngeal cancer. Option A represents a low-risk group (5-year survival rate = 93%); options B and C represent intermediate-risk groups (5-year survival rate = 70.8%); option D represents a high-risk group (5-year survival rate = 46.2%). REFERENCE: New England Journal of Medicine. 2010;363:24-35.

274. An electron beam treatment is planned for a 100 cm SSD, resulting in a calculation of 200 MU. Approximately how many MUs would be required if the clinical setup is limited to 112 cm SSD due to an immobilization device? A. 240 B. 245 C. 250 D. 255

answer is D. RATIONALE: Just doing an inverse square correction would yield 250 MU. However, at such an extended SSD, the electrons will fall off "faster" and require additional MUs. A simple 1/r2 calculation would yield too few MUs.

216. Which of the following radiation damage assays is paired with its correct definition? A. Lethality assay: Single cells are irradiated, followed by incubation for colony formation and generation of a survival curve. B. In vitro assay: After the irradiation of a loop of bowel in a laboratory rodent, the animal is sacrificed, and the clonogenic survival of intestinal crypt cells is quantified. C. In situ assay: A laboratory rodent receives whole-body irradiation, and the percent of animals surviving/dead after a given amount of time is determined. D. Transplantation assay: A tumor is irradiated in a donor rodent, removed, disaggregated, the cells suspended in solution, and then injected into an otherwise tumor-free recipient rodent.

answer is D. RATIONALE: Lethality assays are nonclonogenic and typically involve whole-body irradiation and the determination of the LD50. In vitro assays are performed using cells in culture with reproductive integrity (i.e., clonogenicity) as the endpoint. In situ assays (classically, using intestinal crypt cells) and transplantation assays represent in vivo approaches that may or may not use the reproductive integrity of single cells as an endpoint.

210. Which statistical measure requires survival models to be calculated? A. Specificity B. Sensitivity C. Odds ratio D. Hazard ratio

answer is D. RATIONALE: Logistic regression models yield estimates of odds ratios, sensitivity, and specificity. Survival models are required if hazard ratios are to be produced.

121. Which of the following findings most strongly supports the use of long-term androgen deprivation therapy combined with EBRT for a patient who has prostate cancer with a pretreatment PSA level of 2.5 ng/mL and a Gleason score of 7? A. A family history of prostate cancer B. Three of twelve positive biopsy cores C. A PSA velocity of 1 ng/mL per year D. Flattening of the left lateral sulcus on DRE

answer is D. RATIONALE: Long-term androgen deprivation therapy is a commonly accepted practice standard for men at high-risk for prostate cancer. Prostate cancer is classified as high-risk by the National Comprehensive Cancer Network Guidelines when any one of the following criteria is met: pretreatment PSA of >20 ng/mL, Gleason score of 8-10, or stage T3/4 disease. In this patient with a pretreatment PSA of 2.5 ng/mL and Gleason score of 7, evidence of T3 or T4 disease would constitute high-risk disease. Flattening of the left lateral sulcus on digital rectal exam (DRE) is consistent with clinical evidence of extracapsular extension and would meet the criteria for high-risk disease. A family history of prostate cancer is not a prognostic factor following radiation therapy for prostate cancer. A percentage of >50% positive biopsy cores is commonly associated with bulky disease when combination therapy may be beneficial. A PSA velocity of >2 ng/mL has been associated with an incidence of death related to prostate cancer and combination therapy may be beneficial.

225. Which of the following management options would NOT be appropriate for a patient who has negative lymph nodes and surgical margins after resection for extrahepatic cholangiocarcinoma? A. Observation B. 5-FU-based chemoradiation C. Gemcitabine-based chemotherapy D. SBRT to the surgical bed

answer is D. RATIONALE: Management of resected cholangiocarcinomas remains controversial due to the lack of randomized data. However, according to NCCN guidelines, observation, chemoradiation, and chemotherapy are reasonable treatment options.

44. Which of the following syndromes is associated with an increased risk for the development of meningioma? A. Gorlin syndrome B. von Hippel-Lindau C. Tuberous sclerosis D. Neurofibromatosis type 2

answer is D. RATIONALE: Neurofibromatosis type 2 is associated with an increased risk for the development of meningioma, although the most common tumors are vestibular schwannomas. Other risk factors are radiation exposure, female gender, African/African American race, and increased age.

247. What is the standard treatment for a 42-year-old man who has stage IIA nodular lymphocyte-predominant Hodgkin lymphoma in the left supraclavicular and left axillary lymph nodes, with the largest node measuring 6.5 cm? A. ABVD chemotherapy x 6 alone B. ABVD chemotherapy x 4 and 30 Gy of involved-field radiation therapy C. Subtotal nodal radiation therapy and involved-field boost to a total dose of 40 Gy D. Involved-field radiation therapy to a total dose of 36 Gy alone

answer is D. RATIONALE: Nodular lymphocyte-predominant Hodgkin lymphoma is distinct from the four types of classical Hodgkin lymphoma (nodular sclerosis, mixed cellularity, lymphocyte depleted, and lymphocyte rich). This distinction is important since nodular lymphocyte-predominant Hodgkin lymphoma has an excellent prognosis when treated with involved-field radiation therapy (RT) alone. Exam answers and rationales Radiation Oncology In-Training Exam 2011 75

74. Which radiation-induced cancer had the highest incidence during the first 50 years of follow-up of the Japanese atomic bomb survivors? A. Lung B. Breast C. Thyroid D. Leukemia

answer is D. RATIONALE: Non-CLL leukemia is the type of cancer most often associated with whole-body radiation exposure on the order of 1 Sv or higher, and it is characterized by a mean latent period for induction of approximately 10 years, significantly shorter than that for most radiation-induced solid tumors. Its incidence in Japanese A-bomb survivors has been studied extensively. In this group, leukemia showed the highest incidence of all cancers during the first 50 years of follow-up, with excess relative risks estimated at 3- to 10-fold per Sievert depending on the exact subtype. REFERENCE: Mettler and Upton. Medical Effects of Ionizing Radiation. 3rd ed. Chapter 5.

242. Which of the following statements about the shape of cell survival curves as a function of radiation dose rate is true? A. As the dose rate for alpha particle irradiation decreases, cell survival increases. B. A cell survival curve generated for x-rays using an HDR irradiator will be shallower than one generated using an LDR irradiator. C. Microenvironmental conditions after irradiation have no bearing on the shape of a cell survival curve. D. Splitting of a single x-ray dose into two dose fractions delivered over time increases cell survival.

answer is D. RATIONALE: Option A is incorrect because there is no dose rate effect for high LET radiations like alpha particles. Option B is incorrect because survival curves are shallower at low dose rates (LDR) than high dose rates (HDR) due to repair of sublethal damage during irradiation. Option C is incorrect because the shape of survival curves can be altered after irradiation if cells are manipulated to allow potentially lethal damage recovery to occur.

309. According to the MAGIC trial, patients administered perioperative chemotherapy for gastric cancer: A. received epirubicin, capecitabine, and 5-FU. B. had equivalent overall survival to those patients having surgery alone. C. had a diagnosis of squamous cell carcinoma. D. included those with gastroesophageal junction tumors.

answer is D. RATIONALE: Patients in the perioperative chemotherapy arm received epirubicin, cisplatin, and 5-FU (ECF) chemotherapy. Patients in the perioperative chemotherapy arm had significantly higher overall survival than those having surgery alone. The trial included patients with adenocarcinomas only (not squamous cell carcinomas) and those with gastroesophageal junction tumors. REFERENCE: Cunningham, et al. Landmark trial on perioperative chemotherapy for gastric cancer. New England Journal of Medicine (New Engl J Med). 2006.

88. Which of the following sites of involvement by maxillary sinus carcinoma is associated with the worst prognosis? A. Hard palate B. Pterygoid fossa C. Cribriform plate D. Orbital apex

answer is D. RATIONALE: Please refer to the current AJCC staging. The location and the extent of the mucosal lesion within the maxillary sinus have prognostic significance. Lesions involving the posterosuperior portion have a poor prognosis. It is important to know the stage of maxillary sinus cancer to determine the difference between resectable and unresectable disease. Orbital apex involvement represents a T4b disease and is considered to be an unresectable, very advanced local disease.

115. Which procedure is most appropriate for initial evaluation of a 70-year-old woman with vaginal bleeding and a normal pelvic exam? A. Pap smear B. CT scan of the abdomen and pelvis C. Dilation and curettage D. Endometrial biopsy

answer is D. RATIONALE: Postmenopausal bleeding is highly suspicious for a gynecological malignancy, namely endometrial cancer, until proven otherwise. Most patients with endometrial cancer, 70% to 80%, present with early-stage disease. CT scan of the abdomen and pelvis is generally reserved for patients with documented high-grade endometrial cancers or suspicion of extrauterine disease. Endometrial biopsies (EMB) are safe and can be performed in most patients as an outpatient procedure without the need for anesthesia. An EMB has high sensitivity and specificity with a false-negative rate of 10%. Dilation and curettage (D+C) is an invasive procedure that generally requires anesthesia. This should be performed in patients who cannot undergo EMB or if the EMB is negative. Less than 5% of patients will have malignant cells on a pap smear. REFERENCE: Cardenes H, et al. Endometrium. Perez and Brady. Principles and Practice of Radiation Oncology. 5th ed. Lippincott Williams & Wilkins; 2008. Exam answers and rationales Radiation Oncology In-Training Exam 2011 35

193. Which of the following statements most accurately describes grades 1 and 2 follicular non-Hodgkin lymphoma? A. Low-dose radiation is ineffective for palliation. B. Rituximab monotherapy is not a valid approach. C. Follicular lymphoma is most commonly diagnosed as localized disease. D. Watchful waiting is a valid approach for asymptomatic disease.

answer is D. RATIONALE: Radiation therapy is very effective in achieving long-term disease control in targeted areas; however, patients with asymptomatic disease can be safely observed without affecting their ultimate outcome. Initial response from rituximab treatment alone is excellent, but not durable. Thus, this drug is usually added to chemotherapy agents or used during a disease maintenance phase. Follicular lymphoma is mostly commonly diagnosed at an advanced disease stage. REFERENCES: Jóhannsson J, Specht L, Mejer J, Jensen BA. Phase II study of palliative low-dose local radiotherapy in disseminated indolent non-Hodgkin's lymphoma and chronic lymphocytic leukemia. International Journal of Radiation Oncology, Biology, Physics (Int J Radiat Oncol Biol Phys). 2002 Dec 1;54(5):1466-70. Ardeshna KM, Smith P, Norton A, et al. British national lymphoma investigation. Long-term effect of a watch and wait policy versus immediate systemic treatment for asymptomatic advanced-stage non- Hodgkin lymphoma: a randomized controlled trial. Lancet. 2003;362:516-522. Advani R, Rosenberg SA, Horning SJ. Stage I and II follicular non-Hodgkin's lymphoma: long-term followup of no initial therapy. Journal of Clinical Oncology (J Clin Oncol). 2004;22:1454-1459.

183. What proportion of local failures after breast-conserving surgery and radiation therapy for DCIS are invasive carcinomas? A. 20% B. 30% C. 40% D. 50%

answer is D. RATIONALE: Radiation therapy reduces ipsilateral breast tumor recurrence by 50% to 60%, and approximately half of all recurrences are from invasive disease. REFERENCE: Morrow M, Harris JR. Ductal carcinoma in situ and microinvasive carcinoma. Harris JR, Lippman ME, Morrow M, Osborne CK (eds). Diseases of the Breast. 4th ed. Chapter

22. What is the relationship between path length and range for heavy charged particles versus electrons? A. The path length is approximately equal to the range for all charged particles, including electrons and heavy charged particles. B. The path length is less than the range for all charged particles, including electrons and heavy charged particles. C. The path length is less than the range for electrons, but equal to the range for heavy charged particles. D. The path length is greater than the range for electrons, but equal to the range for heavy charged particles.

answer is D. RATIONALE: Range is the depth penetrated, and the path length is the actual distance travelled. Electrons follow tortuous paths as they travel through matter as a result of many scatter events. Therefore, the path length of electrons exceeds their range. The path length and range of heavy charged particles are practically equal and result in a linear ionization track.

152. Which of the following statements about the use of adjuvant radiation therapy for resected colon cancer is true? A. There has been no prospective trial examining the role of radiation therapy for these patients. B. The INT 0130 trial showed that chemoradiation improved survival in these patients. C. The INT 0130 trial showed that chemoradiation improved local control in these patients. D. Retrospective data suggest that chemoradiation may be beneficial in patients with a high risk of local recurrence.

answer is D. RATIONALE: Retrospective data has suggested that the addition of chemoradiotherapy in patients with a high risk of local recurrence (i.e., patients with stage T3-T4, N+ disease) may decrease recurrence rates. The INT 0130 is a prospective randomized trial that randomized 222 patients with resected colon cancer (T3N1-2 tumors or tumors with adhesion or invasion into adjacent structures) to 5-FU/Levamisole with or without radiation therapy. The trial closed prematurely due to poor accrual. There were no differences in any end points; however, the small number of patients analyzed may limit the conclusions of this study. Because of this, there are those who would advocate for postoperative chemoradiation in high-risk patients in "fixed" areas of the colon - i.e., positive margins, perforations, fistulas. Exam answers and rationales 46 American College of Radiology

38. Which of the following tumors is most likely to be found in the anterior mediastinum? A. Small cell carcinoma B. Adenoid cystic carcinoma C. Paraganglioma D. Germ cell

answer is D. RATIONALE: Small cell carcinoma more commonly involves the middle and posterior mediastinum; adenoid cystic carcinoma involves the trachea. Neurogenic tumors are located in the posterior mediastinum. Thyroid tumors, thymoma, and lymphoma are the most common tumors in the anterior mediastinum. Although mediastinal germ cell tumors are rare, they are mostly located in the anterior mediastinum.

285. A patient is diagnosed with a low-grade papillary cancer of the bladder that does not invade the subepithelial connective tissue. After TURBT alone, the 5-year recurrence rate is approximately: A. 05%. B. 10%. C. 25%. D. 50%.

answer is D. RATIONALE: Superficial urothelial cancers of the bladder are known to have a high risk of recurrence after complete transurethral resection of bladder tumor (TURBT). Although the risk of developing muscle-invasive disease is rather low after TURBT of a stage cTa, low-grade tumor, the risk of any recurrence is approximately 50%. Immediate intravesical therapy can decrease the risk of recurrence to 37%. For low-grade, stage Ta lesions, patients may be offered intravesicular mitomycin C. For high-grade, stage cTa or T1 lesions, intravesicular BCG has been shown to be superior to TURBT alone or TURBT and chemotherapy. REFERENCE: Sylvester, et al. Journal of Urology (J Urol). 2004;171:2186-2190.

63. Which of the following mediastinal neoplasms is most likely to present with myasthenia gravis? A. Parathyroid adenoma B. Small cell carcinoma C. Teratoma D. Thymoma

answer is D. RATIONALE: Symptomatic paraneoplastic syndromes are generally uncommon. However, the Lambert-Eaton myasthenic syndrome affects about 3% of patients with small cell lung cancer; myasthenia gravis affects about 15% to 30% of patients with thymoma. REFERENCE: Levy Y, et al. Malignant thymoma associated with autoimmune diseases: a retrospective study and review of the literature. Seminars in Arthritis and Rheumatism (Semin Arthritis Rheum). 1998;28:73-79.

4. Which of the following AJCC staging criteria for soft tissue sarcomas is correct? A. The three-grade system is reclassified into a four-grade system. B. Dermatofibrosarcoma protuberans is excluded from the list of histologic subtypes. C. Desmoid tumors are included in the list of histologic subtypes. D. Stage N1 disease is reclassified as stage III.

answer is D. RATIONALE: The 7th edition of the AJCC Cancer Staging Manual has instituted the following changes for soft tissue sarcomas: The grading system has been reformatted from a four-grade system into a three-grade system. Angiosarcoma, extraskeletal Ewing's sarcoma, and dermatofibrosarcoma protuberans have been added to the histological subtypes. Desmoid tumors, Kaposi's sarcoma, and infantile fibrosarcoma are no longer included in the histological subtypes for this site. Gastrointestinal stromal tumors are now classified separately. Stage N1 disease has been reclassified from stage IV to stage III. REFERENCE: AJCC Cancer Staging Manual. 7th ed. 2009. p 345.

157. What is the pathologic TNM stage (AJCC, 7th edition) for a 1.5 cm squamous cell carcinoma of the right labia majora with 2 mm of stromal invasion, involvement of 3 of 5 lymph nodes, and each node measuring 4 mm in size when dissected from a right inguinal-femoral lymphadenectomy? A. pT1apN1a (IIIA) B. pT1bpN1b (IIIA) C. pT1apN2a (IIIB) D. pT1bpN2a (IIIB)

answer is D. RATIONALE: The AJCC Cancer Staging Manual (7th edition) has adopted FIGO 2008 staging to reflect the prognostic importance of the number of lymph nodes and the size of lymph node metastases. The only difference in the staging of lymph nodes is that <6 lymph nodes resected in the FIGO staging version would be designated as stage pNx. The patient's carcinoma (based on a size of <2 cm) would be classified as stage T1a, but >1 mm of stromal invasion is staged as T1b. Involvement of one positive lymph node is classified as stage N1 disease, whereas stage N2a relates to 3+ lymph node metastases, with each node smaller than 5 mm in size. REFERENCE: AJCC Cancer Staging Manual. 7th ed. 2009.

127. Which of the following hypofractionated radiation therapy schedules was administered to the whole breast for women with early-stage breast cancer in the Canadian (Whelan, et al.) trial? A. 38.4 Gy in 10 fractions twice daily for 1 week B. 39 Gy in 13 fractions 5 days per week for 2½ weeks C. 40 Gy in 10 fractions 5 days per week for 2 weeks D. 42.5 Gy in 16 fractions 5 days per week for 3 weeks

answer is D. RATIONALE: The Canadian trial compared 42.5 Gy in 16 fractions and 50 Gy in 25 fractions. There was no difference at 10 years in local control. The START A and B trials compared different whole-breast hypofractionation schedules to 50 Gy in 2-Gy fractions over 5 weeks. The START A trial used 41.6 Gy or 39 Gy in 13 fractions but used 5 fractions every 2 weeks to maintain a 5-week treatment length. The START B trial used 40 Gy in 15 fractions 5 days a week for 3 weeks. REFERENCES: Whelan, et al. New England Journal of Medicine. 2010;362:513-20. Lancet Oncology. 2008;9:331-41. Lancet. 2008;371:1098-107.

221. Which of the following forms of leukemia most commonly is associated with the Philadelphia chromosome? A. Acute lymphoblastic B. Acute myelogenous C. Chronic lymphocytic D. Chronic myelogenous

answer is D. RATIONALE: The Philadelphia chromosome t(9;22)(q34;q11) is associated with chronic myelogenous leukemia (CML).

79. A 45-year-old patient undergoes complete resection of a 5 cm, completely encapsulated, type B2 thymoma. According to the SEER analysis, postoperative radiation therapy is: A. indicated due to the tumor size. B. indicated due to the B2 disease. C. indicated with adjuvant chemotherapy. D. not indicated.

answer is D. RATIONALE: The SEER analysis showed clearly no benefit for the use of postoperative radiation therapy (PORT) in patients with stage I (encapsulated) completely resected thymoma. No conclusions were drawn based on tumor size or histology. REFERENCE: Forquer JA, et al. International Journal of Radiation Oncology, Biology, Physics (Int J Radiat Oncol Biol Phys). 2010.

276. According to the Swedish Rectal Cancer Trial, which treatment improved local control and overall survival in patients with resectable rectal cancer? A. Total mesorectal excision B. Preoperative radiation therapy with continuous 5-FU infusion C. Postoperative radiation therapy D. Preoperative radiation therapy

answer is D. RATIONALE: The Swedish Rectal Cancer Trial was the one of the first randomized studies to evaluate the efficacy of this fractionation schema, which randomized 908 patients with stage I-III resectable rectal cancer between 1987 and 1990 to curative surgery alone (n=454) or preoperative radiation therapy (25 Gy in 5 fractions, n=454) followed by curative surgery within 1 week of completion of radiation. With a median follow-up of 13 years, compared to surgery alone, preoperative radiation therapy significantly decreased rates of local recurrence (9% vs. 26%, p<0.001) and increased rates of cancer-specific survival (72% vs. 62%, p=0.03) and overall survival (38% vs. 30%, p=0.008)1. However, there were no significant differences with respect to rates of distant failure between the irradiated and nonirradiated arms. REFERENCE: Improved survival with preoperative radiotherapy in resectable rectal cancer. Swedish Rectal Cancer Trial. New England Journal of Medicine (N Engl J Med). 1997;336:9807. Exam answers and rationales 84 American College of Radiology

303. Chemotherapy should be added to EBRT after radical surgery for patients who have squamous cell carcinoma of the cervix and: A. deep cervical stromal invasion. B. lymphovascular space invasion. C. pathologic tumor size >4 cm. D. positive pelvic lymph nodes.

answer is D. RATIONALE: The addition of concurrent cisplatin-based chemotherapy to pelvic radiation significantly improves progression-free and overall survival for high-risk patients who undergo radical hysterectomy and pelvic lymphadenectomy for carcinoma of the cervix. Based on the Peters' trial, high-risk was defined as positive pelvic lymph nodes and/or positive margins and/or microscopic involvement of the parametrium. REFERENCE: Journal of Clinical Oncology. 2000 April;18(8):1606-1613.

71. The energy of the x-rays exiting the head of a linear accelerator can be changed by: A. plugging the accelerator into an outlet that has a different voltage. B. changing the energy of the electrons leaving the electron gun. C. changing the gain of the monitor chamber in the accelerator head. D. changing the amplitude of the electric field of the microwaves in the waveguide.

answer is D. RATIONALE: The amplitude of the microwaves determines the acceleration of the electrons in the waveguide. The initial energy of the electrons leaving the electron gun is not very important. Changing the gain of the monitor chamber in the accelerator head only changes the output of a linear accelerator (linac).

131. Which of the following radiation dose ranges is most appropriate for treatment of a solitary plasmacytoma? A. 10 to 15 Gy B. 25 to 30 Gy C. 35 to 40 Gy D. 45 to 50 Gy

answer is D. RATIONALE: The appropriate dose for treating solitary plasmacytoma is 45 to 50 Gy. REFERENCE: DeVita VT, Hellman S, Rosenberg, SA (eds). Cancer: Principles & Practice of Oncology. 6th ed. Lippincott: 2001; p 2483. Exam answers and rationales 40 American College of Radiology

294. According to the 18-year Danish 82B and 82C trials, postmastectomy irradiation reduced first-event absolute locoregional recurrence rates in patients with high-risk breast cancer by: A. 10%. B. 15%. C. 25%. D. 35%.

answer is D. RATIONALE: The first-event absolute locoregional recurrence rate was 14% in patients receiving radiation and 49% in patients who did not receive radiation. The difference is 35% (p <0.0001). REFERENCE: Nielsen HM, Overgaard M, Grau C, et al. Study of failure pattern among high-risk breast cancer patients with or without postmastectomy radiotherapy in addition to adjuvant systemic therapy: long-term results from the Danish Breast Cancer Cooperative Group DBCG 82b & C randomized studies. Journal of Clinical Oncology (J Clin Oncol). 2006;24:2268-2275. Exam answers and rationales Radiation Oncology In-Training Exam 2011 89

62. Which of the following tumor sites has the highest risk for metastasis to the Rouvière's lymph nodes? A. Tonsil B. Oral cavity C. Pyriform sinus D. Nasopharynx

answer is D. RATIONALE: The nasopharynx/hypopharynx tumor sites (specifically the posterior pharyngeal wall) are the most likely primary cancer sites to spread to the Rouvière's lymph nodes. Exam answers and rationales Radiation Oncology In-Training Exam 2011 19

296. According to the RTOG 90-01 trial, what was the absolute improvement in overall survival at 8 years for patients with high-risk cervical cancer treated with chemotherapy and pelvic irradiation versus pelvic and paraaortic irradiation? A. 05% B. 10% C. 20% D. 25%

answer is D. RATIONALE: The overall survival rate for patients treated with chemotherapy and pelvic irradiation was significantly greater than that for patients treated with pelvic and paraaortic irradiation (67% vs 41% at 8 years with P <.0001). REFERENCE: Journal of Clinical Oncology (J Clin Oncol). 2004 Mar 1;22(5):872-80.

99. According to the phase III randomized (Whelan, et al.) trial, which of the following 10-year local control and cosmetic outcomes were demonstrated in patients receiving hypofractionated versus conventional fractionated whole-breast radiation therapy for early-stage breast cancer? A. Worse local control and cosmetic outcome B. Improved local control and cosmetic outcome C. No difference in local control but worse cosmetic outcome D. No difference in local control or cosmetic outcome

answer is D. RATIONALE: The phase III randomized trial compared whole-breast radiation therapy of 42.5 Gy in 16 fractions to 50 Gy in 25 fractions. The local recurrence rate at 10 years was 6.7% with standard fractionation and 6.2% with hypofractionation. The cosmetic outcome was good or excellent in 71.3% and 69.8% of patients, respectively. These differences were not significant. REFERENCE: Whelan, et al. New England Journal of Medicine (NEJM). 2010;362:513-20. Exam answers and rationales 30 American College of Radiology

139. Which of the following physical interactions causes silver halide film to overrespond with respect to human tissue (water) at low photon energies? A. Pair production B. Compton scatter C. Incoherent scatter D. Photoelectric effect

answer is D. RATIONALE: The photoelectric effect is the dominant interaction at lower photon energies. The high Z material in film compared to that of tissue (water) causes an overexposure with respect to dose in tissue (water). The mass photoelectric attenuation coefficient is proportional to Z3.

318. A 65-year-old smoker has an extensive mediastinal adenopathy. A fine needle aspiration of the subcarinal lymph node shows an extensive nuclear crush artifact in the specimen. What is the most likely diagnosis? A. Mesothelioma B. Adenocarcinoma C. Squamous cell carcinoma D. Small cell carcinoma

answer is D. RATIONALE: The presence of a nuclear crush artifact, while not pathognomonic, is a highly suggestive morphologic feature for the diagnosis of small cell lung cancer in this clinical setting. REFERENCE: Davenport R, et al. Acta Cytologica (Acta Cytol). 1990 Jul-Aug;34(4):502-4.

178. Which of the following histological subtypes of thyroid cancer has the worst prognosis? A. Hürthle cell B. Classic papillary C. Follicular variant of papillary D. Giant cell variant of anaplastic

answer is D. RATIONALE: The prognosis is similar for the follicular variant of papillary cancer, classic papillary cancer, and classic follicular cancer.

246. Radiation therapy for a patient with stage T1N0 carcinoma of the false vocal cord versus a stage T1N0 lesion of the true vocal cord should involve: A. unilateral treatment. B. a hypofractionated regimen. C. a shorter treatment time. D. treating the regional lymph nodes.

answer is D. RATIONALE: The risk for regional lymph node involvement is 20% to 30% for supraglottic cancer compared to <1% for a stage T1 lesion. REFERENCE: Lindberg, RD. Distribution of cervical lymph node metastases from squamous cell carcinoma of the upper respiratory and digestive tracts. Cancer. 1972;29:1446.

196. What is the specificity of the x-ray interpretation compared to the pathology results shown below? Abnormal x-ray and abnormal pathology 300 patients Abnormal x-ray and normal pathology 30 patients Normal x-ray and abnormal pathology 20 patients Normal x-ray and normal pathology 50 patients A. 93.8% B. 90.9% C. 71.4% D. 62.5%

answer is D. RATIONALE: The sensitivity and specificity are also used in investigating the performance of simpler diagnostic tests. Suppose "gold standard" tests provide whether or not a subject actually has the disease. The number of subjects without disease based on the "gold standard" diagnosis is 30+50. Among those, 50 subjects are classified as not having disease by x-ray examination: Specificity = 50/(50+30) = 62.5%.

298. Langerhans cell histiocytosis most commonly involves which of the following bony sites? A. Ribs B. Long bones C. Vertebrae D. Skull

answer is D. RATIONALE: The skull, including the mandible, is the most common bony site of involvement; it is involved in 50% of patients. Twenty-five percent of patients have lesions in the long bones of the extremities, and the vertebrae are involved in 10% of patients. Fifty percent of patients have a single lesion at diagnosis. REFERENCE: Halperin, Constine, Tarbell, Kun. Langerhans cell histiocytosis. Pediatric Radiation Oncology. 4th ed. pp 487-508.

255. There is no dose rate effect for carbon ion radiation therapy because: A. tumor cell repopulation is inhibited. B. the oxygen enhancement ratio equals 1.0. C. there is no age response through the cell cycle. D. there is no sublethal damage recovery.

answer is D. RATIONALE: The types of DNA damage caused by exposure to high LET, densely ionizing radiation like a carbon ion beam are more complex and severe and less amenable to repair. As sublethal damage recovery is a manifestation of chromosome break rejoining and that, in turn, is a manifestation of DNA damage repair, it follows that there would be little or no sublethal damage recovery during carbon ion radiation therapy. And because sublethal damage recovery during radiation therapy largely accounts for the dose rate effect, there would be no dose rate effect for carbon ion radiation therapy either. REFERENCE: Hall and Giaccia. Radiobiology for the Radiologist. 6th ed. Chapter 7.

5. Which type of salivary gland tumor is most likely to be benign? A. Minor salivary B. Submandibular C. Sublingual D. Parotid

answer is D. RATIONALE: There is an inverse relationship between the size of the salivary gland and the likelihood of malignancy.

187. Which of the following tests should be included in the workup of a 3-year-old child who has a presumed diagnosis of stage 3, high-risk abdominal neuroblastoma? A. MRI of the brain B. Cerebrospinal fluid cytology C. Total body gallium scan D. Urinary catecholamine metabolite levels

answer is D. RATIONALE: This patient should have measurements of urine vanillylmandelic acid (VMA) and homovanillic acid (HVA) levels, which are catecholamine metabolites.

84. Which statement about the importance of angiogenesis in cancer development and treatment is true? A. Ionizing radiation kills tumor cells independent of a tumor's vascular structure. B. Solid tumors treated with anti-angiogenic drugs become more radioresistant. C. Anti-angiogenic drugs permanently normalize tumor blood vessels. D. Abnormal blood vessels are a hallmark of solid cancers.

answer is D. RATIONALE: Tumor blood vessels are abnormal structurally, functionally, and physiologically, due in part to the continuous overstimulation of angiogenesis by tumor cells. The tumor blood vessel normalization effect following treatment with anti-angiogenics is a temporary phenomenon; however, tumors become more sensitive, not resistant, to other cancer therapies during this time. Tumor angiogenesis has now become an important target for novel treatment approaches that combine anti-angiogenic agents with radiation. REFERENCE: Jain RK. Taming vessels to treat cancer. Scientific American (Sci Am). 2008;298:53-63.

26. Which of the following hypoxic cell radiosensitizers or hypoxic cytotoxins correctly matches its description? A. Etanidazole - hypoxia-specific cytotoxin B. Tirapazamine - nitroimidazole radiosensitizer C. Misonidazole - detected in tumors using MR spectroscopy D. Nimorazole - combined with radiation therapy for head and neck cancer

answer is D. RATIONALE: Tumor hypoxia remains a likely cause of radiation therapy failure in many settings. The hypoxic cell radiosensitizer nimorazole has proven clinical efficacy in combination with radiation therapy for a subset of patients with head and neck cancer (DAHANCA 5 trial), yet the combination is still not in widespread use.

80. A 2-year-old patient's tumor ruptures during resection of a large, right-sided Wilms tumor. Chemotherapy is scheduled to start during the second postoperative week. Which of the following radiation treatments would be most appropriate? A. 10.8 Gy to the preoperative tumor volume at the end of chemotherapy B. 10.8 Gy to the preoperative tumor volume starting before chemotherapy C. 10.5 Gy to the whole abdomen with a 10.8 Gy boost to the primary site starting before day 9 D. 10.5 Gy to the whole abdomen starting before postoperative day 9

answer is D. RATIONALE: With a diffuse abdominal spill, 10.5 Gy is given to the whole abdomen on or before postoperative day 9. If there is no spill and there is a need for radiation therapy, 10.8 Gy is given to the primary site by day 9. Exam answers and rationales 24 American College of Radiology

281. Which of the following features is associated with improved overall survival in patients with low-grade gliomas? A. Intact 1p and 19q B. Bilateral involvement C. Pure astrocytic histology D. Age of less than 45 years

answer is D. RATIONALE: Younger age, oligodendroglial histology, 1p and 19q codeletion, resectable disease, and size <6 cm are associated with a better prognosis for patients with low-grade gliomas.

177. Which of the following tumors is NOT associated with von Hippel-Lindau syndrome? A. Retinal angioma B. Pheochromocytoma C. CNS hemangioblastoma D. Papillary renal cell carcinoma

answer is D. RATIONALE: von Hippel-Lindau (VHL) syndrome is associated with clear cell renal cell carcinoma and not papillary renal cell carcinoma. Von Hippel-Lindau (VHL) syndrome also is associated with retinal angioma, pheochromocytoma, and CNS hemangioblastoma.

52. What is the local control rate for unresectable desmoid tumors treated with definitive radiation therapy alone? A. 65% B. 70% C. 75% D. 80%

answer is D. REFERENCE. Micke O, Seegenschmiedt MH. German Cooperative Group on Radiotherapy for Benign Disease. Radiation therapy for aggressive fibromatosis (desmoid tumors): results of national Patterns of Care Study. International Journal of Radiation Oncology, Biology, Physics (Int J Radiat Oncol Biol Phys). 2005;61:882-91.


संबंधित स्टडी सेट्स

Global Sports and National Culture UIowa

View Set

A Game of Polo with a Headless Goat

View Set

Addiction and Substance-Related Disorders

View Set

Bontrager Ch 8 Cervical and Thoracic Spine Workbook

View Set

Straighterline Intro to Nutrition Quiz 2

View Set

Chapter 15: Product Use Testing, AB testing, Ab test, AB Testing, SaaS Key Terms, Saas Terminology, Market Sizing, SAAS metrics, Rob's Decode & Conquer +, Decode and Conquer, Product Metric Frameworks, Interview - Product design, Interview - Product...

View Set